Top Banner
Economist : Although open-source software has gained popularity in the last couple of years, its usefulness is quite limited. This is because the most creative ideas come from professional software designers, not volunteers. Journalist : I, too, was once skeptical about whether open-source software would become a permanent feature of our world. However, I realized that this article could not have been produced without the open-source software volunteers who allowed me to interview them. The journalist’s response to the economist is flawed for which of the following reasons? The journalist has changed his mind about open- source software, making his response invalid. The journalist refused to interview any professional software designers for his article. The journalist does not address the economist’s argument that professional software designers are more creative than volunteers. The journalist has written a series of articles on software development. The journalist does not discuss the popularity of open-source software. Situation: An economist argues that open-source software has limited usefulness because professional software designers are more creative than the volunteers who develop open-source materials. A journalist objects to this, claiming that his article would not have been possible without open-source software volunteers to interview.
294
Welcome message from author
This document is posted to help you gain knowledge. Please leave a comment to let me know what you think about it! Share it to your friends and learn new things together.
Transcript
Page 1: Economist

Economist: Although open-source software has gained popularity in the last couple of years, its usefulness is quite limited. This is because the most creative ideas come from professional software designers, not volunteers.

Journalist: I, too, was once skeptical about whether open-source software would become a permanent feature of our world. However, I realized that this article could not have been produced without the open-source software volunteers who allowed me to interview them.

The journalist’s response to the economist is flawed for which of the following reasons?

The journalist has changed his mind about open-source software, making his response invalid.

The journalist refused to interview any professional software designers for his article.

The journalist does not address the economist’s argument that professional software designers are more creative than volunteers.

The journalist has written a series of articles on software development.

The journalist does not discuss the popularity of open-source software.Situation: An economist argues that open-source software has limited usefulness because professional software designers are more creative than the volunteers who develop open-source materials. A journalist objects to this, claiming that his article would not have been possible without open-source software volunteers to interview.

Reasoning: Which option describes the biggest flaw in the journalist’s response? The economist’s argument is based on a comparison between professional software designers and volunteers, who are primarily responsible for open-source software development. The economist argues that professional designers are more creative, and that, therefore, open-source software developed by volunteers will not be as useful. In order to be valid, the journalist’s response must specifically address the comparison drawn in the economist’s argument.

A. That fact that the journalist has changed his mind does not invalidate his response.

Page 2: Economist

B. This can not be concluded based only on the information in the passage.

C. This option correctly identifies the fact that the journalist has ignored the main point of the economist’s argument: that professional software designers are more creative than volunteers.

D. The journalist’s other writings are not at issue here.

E. Though this is true, it does not identify the flaw in the journalist’s response to the economist.

The correct answer is C.

Roadblocks on major holidays, even if they are set up only to check drivers’ licenses and seatbelts, are spontaneous searches of every driver, even the ones who have committed no crime. Without establishing probable cause, I believe such searches should be illegal.

Without establishing probable cause, I believe such searches should be illegal

Unless establishing probable cause, such searches, I believe, should be illegal

I believe such searches should be illegal, without establishing probable cause

I believe such searches should be illegal, unless officers have established probable cause

I believe without establishing probable cause that such searches should be illegalIn this sentence, the author is arguing that searches at roadblocks should only be conducted if officers have established probable cause for the search, or are convinced that the driver may have committed a crime. According to the wording of the original sentence, however, it is the author’s belief that is heldwithout establishing probable cause. The correct answer choice will eliminate this confusion.

Page 3: Economist

A. The wording of this option is unclear; its meaning literally is I believe this without establishing probable cause.

B. The wording of this option is even more awkward and confusing than the original.

C. This option rearranges the original, but still states that it is the author’s belief that is held without probable cause.

D. This option clears up the confusion of the original by separating the author’s belief that such searches should be illegal from the officers’ need to establish probable cause.

E. This option does not clear up the confusion of the original.

The correct answer is D.It is a mistake to give post office employees individual discretion as to when to inspect or open suspicious packages. If individual employees are allowed to open “suspicious” packages without first following a strict protocol, it is only a matter of time before all packages will arrive having already been opened due to some postal employee’s idle curiosity.

The conclusion above is based on which of the following assumptions?

Postal service managers are the only people with the authority to open suspicious packages.

Suspicious packages are indistinguishable from all other kinds of package.

The efficiency of the postal service will be greatly reduced if more packages are inspected.

There is currently no protocol in place for the inspection of suspicious packages.

Postal employees desire to open packages out of curiosity.Situation: The author of the passage argues that if individual post office employees are allowed individual discretion as to when to open suspicious packages, they will soon open all packages out of curiosity.

Page 4: Economist

Reasoning: Which of the following is an assumption made in the passage? The author of the passage would like to see postal service employees prevented from making their own decisions about when to open packages. The author would prefer that a strict protocol for opening suspicious packagesbe put in place in order to prevent all packages from arriving already open. The author assumes that postal employees will open packages out of curiosity and wants to prevent this.

A. This claim is not made in the passage.

B. What exactly makes a package suspicious is not discussed in the passage, and so it is impossible to tell whether it is indistinguishable from all other kinds of package.

C. Postal service efficiency is not the author’s concern here.

D. The passage does not support this claim.

E. The author of the passage’s fear is that, if postal employees are allowed to decide when to open packages, they will open them all out of idle curiosity.

The correct answer is E.In a recent study examining the relationship between expectation and experience of flavor, researchers conducted a taste test with over 200 participants in which each person tasted two versions of a cola beverage, one of which they were told contained a “secret ingredient.” Because the secret ingredient was white vinegar, researchers were surprised when participants preferred the white vinegar version of the cola beverage by a margin of three to one.

Which of the following, if true, offers the best basis for an explanation of the fact that participants preferred the beverage containing white vinegar?

Most people actually prefer a cola beverage containing white vinegar.

Study participants focused on identifying the beverage with the “secret ingredient”.

Researchers added only a small amount of vinegar to the cola.

Study participants were selected based on their enjoyment of white vinegar.

Page 5: Economist

Most cola recipes contain no white vinegar.Situation: A study of the relationship between expectation and taste has found that a majority of participants preferred a cola beverage containing a “secret ingredient,” even though the secret ingredient was white vinegar.

Reasoning: Which option is the basis for an explanation of why a majority of people preferred the beverage containing white vinegar? The subject of the study is the relationship between expectation and the experience of flavor, or taste. It is reasonable to conclude, then, that the researchers expect that something about the participants’ expectations to influence their responses to the two beverages. Because participants are told that one of the two beverages contains a secret ingredient – but not the identity of the ingredient – it can be concluded that participants focused simply on identifying the beverage with the secret ingredient and, because they likely expected it to be a pleasant flavor, were not able to distinguish that it was white vinegar.

A. This option does not account for the fact that the study is on the relationship between expectation and taste.

B. Because the topic of the study is the relationship between expectation and taste, it is reasonable to conclude that participants preferred the beverage containing white vinegar simply because they were expecting to identify and prefer the beverage with the secret ingredient.

C. The amount of vinegar added to the cola is not addressed in the passage.

D. Though this might be possible, it is not suggested in the passage and, more importantly, would skew the results of the study.

E. This information is irrelevant to the study described in the passage.

The correct answer is B.In the wake of a series of fraud convictions and mismanagement scandals involving members of a five-person city council, a commentator for a local newspaper has suggested that the council create a sixth elected position. The person elected to the new position would be responsible for the daily operation of the city government, leaving the other council members time to talk with their constituents and to write legislation.

Which of the following, if true, points to the biggest flaw in the plan to curb corruption by adding a sixth elected position to the city council?

Page 6: Economist

The council manager, though a voting member of the council, would not be allowed to introduce new legislation.

The council manager would have no control over other board members’ decisions about funds disbursement.

Each council member represents over 15,000 constituents and would not have time to talk with all of them.

The council manager is required to have a background in either accounting or business administration.

The city council must cooperate with the county government, which is also made up of elected officials.Situation: A newspaper commentator suggests adding a sixth position to a city council that has had problems with fraud and mismanagement. The sixth position would be responsible for the city’s daily operations.

Reasoning: Which is the biggest flaw in the plan? The sixth council position has been suggested as a way to prevent further corruption, specifically involving finances. The plan holds that the sixth position would allow council members extra time to work with citizens and legislation, and that the new manager would be responsible for daily operations. If, however, these new duties will not curb financial corruption, the plan is deeply flawed.

A. This provision would not aid the council manager in preventing corruption.

B. If the council manager would have no control over other board members’ financial decisions, then the position’s ability to prevent fraud would be seriously diminished.

C. The number of constituents represented by each council member is irrelevant.

D. This would actually be a strength of the plan.

E. This option does not directly address the function of the sixth position and is, therefore, irrelevant.

The correct answer is B.Which of the following best completes the passage below?

Page 7: Economist

At a recent conference on the evolution of humans, a team of scientists proposed the theory that the millions of sweat glands in human skin evolved as a way to keep humans cool while running over long distances or for extended periods. The scientists theorize that this ability developed as a hunting strategy that depended on out-enduring prey by pushing animals to overheat. One of the first things that must be shown, of course, before this theory can be commonly accepted is that  .

endurance is an advantage for running long distances or for extended periods

animals can be pushed to a point where overheating will make them easier prey

there are enough sweat glands in human skin to keep humans from overheating

early humans were the only hunters to employ this strategy

the scientific community can agree on the meaning of “long distances”Situation: Scientists propose that human sweat glands evolved to aid in hunting animals.

Reasoning: What would have to be shown to make the theory likely to be commonly accepted? The scientists at the conference propose a link between human sweat glands and the ability to hunt animals as prey. Millions of sweat glands, the scientists theorize, would allow humans a great deal of endurance, which would then allow them to out-endure their prey, or run animals to the point of overheating. What must be shown is that an ability to endure longer than animals would be an advantage to hunting.

A. Endurance is the definition of what is needed for running long distances or for extended periods; this does not need to be proved.

B. Scientists theorize that sweat glands evolved as part of a hunting strategy. It must be shown, then, that out-enduring animals would make them easier prey.

C. Everyday experience shows that humans do not routinely overheat; this fact does not require proof.

Page 8: Economist

D. Whether or not any other early hunters used this strategy is irrelevant to a discussion of whether it worked for humans.

E. In the context of this argument, long distances means merely as far as an animal can run. Agreement on a specific measure of distance is not required.

The correct answer is B.The idea behind the Personal Long Letter campaign is that a single impassioned constituent may sway a lawmaker’s opinion, whereas a half-dozen banded together only causes him alarm.

a half-dozen banded together only causes him alarm

only alarm is caused by a half-dozen banded together

only alarm has been caused by a half-dozen banded together

a half-dozen banded together only cause him alarm

a half-dozen have caused him only alarm when banded togetherThe noun half-dozen, though it refers to a collection of six discrete things or people, is grammatically singular. Any verbs that take it as their antecedent, then, must also be singular. This sentence also tests correct verb form, which should be simple present tense because the sentence refers to facts that aregenerally true. The sentence is correct as written.

A. The noun half-dozen agrees with the verb causes, and the correct simple present tense is used to describe facts generally believed to be true.

B. Verb tense is correct in this option, but the phrasing of the sentence is wordy and awkward.

C. The verb form has been caused unnecessarily uses past perfect, and the word order is awkward.

D. Cause does not agree with the singular a half-dozen.

E. This option is awkward and wordy.

The correct answer is A.

Page 9: Economist

When children experience emotional upset or trauma, their problems are sometimes made worse by well-meaning adults who either don’t notice their distress or insist that they express themselves through an adult medium: verbal language. Play therapy, on the other hand, recognizes that imaginative play is the mode of expression most comfortable and natural to children and focuses on helping them express their emotions. A form of psychodynamic treatment, play therapy takes for granted the fact that, once aided in expression, children are able to resolve their anxieties themselves.

The equipment of play therapy is simple and familiar. Dolls, play dishes, soldiers, stuffed animals, and sandboxes all have their place in the play therapy room. The job of the therapist, however, is a bit more challenging, because he or she must refrain from guiding the child in a particular direction or asking pointed questions. The therapist’s task is partly to observe the child’s play and recognize the child’s emotions and help bring them to expression. The second, very important, task is for the therapist to accept the child unconditionally.

A well-known example of successful play therapy is documented in Dr. Virginia Axline’s book Dibs in Search of Self. Dibs is an emotionally disturbed 5-year-old whose teachers fear he may have brain damage. In his play therapy sessions with Dr. Axline, however, a different picture emerges. Dibs’ play with a dollhouse and several dolls reveals that he feels cut off from his family and that he is angry with them for locking him in his room. This theme is repeated in play with animals in the room’s sandbox; he frees them from captivity again and again. During the sessions, Dr. Axline is careful not to express preference for any of Dibs’ activities, and most of her comments are simply reflecting back to him what he has just said. Additionally, through her choices to help him express his anger rather than offering him comfort in it, Dr. Axline fosters in Dibs a sense of emotional independence. At the end of the case study, Dibs has emerged as a content, intelligent, compassionate child.

According to the passage, which of the following is a reason that play therapists help disturbed children express their emotions?

Children express themselves through a different medium than adults.

Children sometimes experience emotional upset or trauma.

The therapist’s task is to accept the child unconditionally.

Therapists observe children’s behavior and recognize the emotions being expressed.

Page 10: Economist

Children can resolve their own anxieties once they have been aided in expression.Since this question uses the phrase according to the passage, the answer is stated in the passage. Aside from the general tone in the passage that play therapy is intended to help children, only this line relate a specific reason: play therapy takes for granted the fact that, once aided in expression, children are able to resolve their anxieties themselves.

A. This is stated in the first paragraph, but is not areason for play therapists to help children express emotion.

B. Though true, this is not directly stated as areason.

C. This information comes from the second and third paragraphs, but is a method not a reason.

D. Though this is an activity of the therapist, it is not a reason for helping the child express emotion.

E. This option correctly identifies that fact that the purpose of helping children express their emotions through play therapy is to allow them toresolve their anxieties themselves.

The correct answer is E.When children experience emotional upset or trauma, their problems are sometimes made worse by well-meaning adults who either don’t notice their distress or insist that they express themselves through an adult medium: verbal language. Play therapy, on the other hand, recognizes that imaginative play is the mode of expression most comfortable and natural to children and focuses on helping them express their emotions. A form of psychodynamic treatment, play therapy takes for granted the fact that, once aided in expression, children are able to resolve their anxieties themselves.

The equipment of play therapy is simple and familiar. Dolls, play dishes, soldiers, stuffed animals, and sandboxes all have their place in the play therapy room. The job of the therapist, however, is a bit more challenging, because he or she must refrain from guiding the child in a particular direction or asking pointed questions. The therapist’s task is partly to observe the child’s play and recognize the child’s emotions and help bring them to expression. The second, very important, task is for the therapist to accept the child unconditionally.

A well-known example of successful play therapy is documented in Dr. Virginia Axline’s book Dibs in Search of Self. Dibs is an emotionally disturbed 5-year-old whose teachers fear he may have brain damage. In his play therapy sessions with Dr. Axline, however, a different picture emerges. Dibs’ play with a dollhouse and several dolls reveals that he feels cut off from his family and that he is angry with them for

Page 11: Economist

locking him in his room. This theme is repeated in play with animals in the room’s sandbox; he frees them from captivity again and again. During the sessions, Dr. Axline is careful not to express preference for any of Dibs’ activities, and most of her comments are simply reflecting back to him what he has just said. Additionally, through her choices to help him express his anger rather than offering him comfort in it, Dr. Axline fosters in Dibs a sense of emotional independence. At the end of the case study, Dibs has emerged as a content, intelligent, compassionate child.

It can be inferred from the passage that the author considers play therapy to be a

sound and useful approach to helping emotionally disturbed children

pleasant and diverting way for children to spend their time

new and exciting way of understanding the experience of childhood

hazardous but necessary method of reaching out to troubled children

universally applicable solution to all problems with children’s behaviorAn inference is drawn from stated information and, in this case, must come from information in the entire passage. The first paragraph relates the reasons for play therapy, and the second and third paragraphs describe its general principles, both abstractly and then using examples. Since the author goes into so much detail about play therapy, it is likely that he or she regards it as something sound and useful.

A. The passage contains only information about what play therapy is and how it can be successful; therefore it is reasonable to conclude that the author regards it as a sound and useful approach.

B. The passage is not concerned with whether play therapy is pleasant or diverting.

C. It is not stated in the passage that play therapy isnew.

D. The passage places confidence in play therapy and does not infer that it is hazardous.

E. The first paragraph states that play therapy is useful mostly for children who have experiencedemotional upset or trauma.

The correct answer is A.Which of the following best describes the relationship of paragraph 3 to paragraph 2?

New information is introduced that qualifies earlier statements.

Page 12: Economist

An exception to previously stated rules is discussed.

Important concepts detailed beforehand are rephrased in briefer form.

General principles introduced earlier are expanded using specific examples.

Concepts essential to further study of the subject are outlined.To answer this question, review paragraphs 2 and 3 to determine their main ideas. Paragraph 2 discusses in general terms the equipment of play therapy and the responsibilities of the therapist. Paragraph 3 covers these same topics, but using the example of Dr. Axline’s work with the child Dibs.

A. New information is introduced, but it does notqualify earlier statements.

B. An exception to rules is not discussed.

C. Important concepts reappear, but they are expanded, not presented in briefer form.

D. The general principles of play therapy equipment and responsibilities of the therapist are expanded using the example of Dr. Axline’s work with Dibs.

E. The purpose of paragraph 3 is not to outlineconcepts essential to further study.

The correct answer is DThe primary purpose of the passage is to

convince the reader that adults often misunderstand children.

discuss the goals and procedures of a form of psychological treatment.

demonstrate the effectiveness of an approach using examples.

relate the history of an emotionally disturbed child.

offer guidance to adults who wish to better understand children.To answer this question, consider the passage as a whole and determine its main point. The first paragraph introduces the need for and goals of play therapy. The second and third paragraphs discuss the equipment and responsibilities of the therapist, first generally, then using the example of a specific case study.

A. This idea is mentioned in the first paragraph, but is not the main idea of the passage.

Page 13: Economist

B. The first paragraph introduces the goals of play therapy, and the second and third paragraphs discuss the procedures.

C. Though the effectiveness of treatment is demonstrated in the third paragraph, this is not the main idea of the entire passage.

D. Dibs’ history is not the main idea of the entire passage.

E. Though this may be possible for those who learn about play therapy, it is not the primary purpose of this particular passage.

The correct answer is BIn addition to adhering to the policies and guidelines of the State Department of Education, the successful instructor will be responsible for instruction in the appropriate teaching environment and evaluates student learning.

will be responsible for instruction in the appropriate teaching environment and evaluates

will be responsible for instruction in the appropriate teaching environment and evaluate

is responsible for instruction in the appropriate teaching environment and evaluate

is being responsible for instruction in the appropriate teaching environment and for evaluating

is responsible for instructions in the appropriate teaching environment and will be evaluatingThe original sentence contains an error in parallel form. The successful instructor will do two things: 1) be responsible for instruction and 2) evaluate student learning. Even though the verb form is responsible is technically possible in this sentence, the answer choices that use it introduce other types of errors or redundancies.

A. The forms will be responsible and evaluates are not parallel.

B. This option displays correct parallel form between the words will be responsible and evaluate, and this answer choice introduces no other errors.

C. Is responsible and evaluate are not parallel.

Page 14: Economist

D. The verb form is being responsible is wordy and unnecessary.

E. Verb forms is responsible and will be evaluating are not parallel, and the addition of an s to instruction introduces content not in the original sentence.

The correct answer is B.Taking note of the success of local farmers’ markets, the university’s food service providers have transitioned into a locally-grown, sustainable operation.

have transitioned into a locally-grown, sustainable operation

have transitioned into sustainable, locally-grown operation

have transitioned into locally-grown, sustainable operations

are transitioned into sustainable, locally-grown operations

are transitioning into locally-grown, sustainable operationThis question tests correct agreement between the plural food service providers and the word operation, which should also be the plural operations. The words are divided by an intervening phrase, but they must still agree. Also tested is the use of the verb form have transitioned to indicate that a change has already occurred.

A. The word a and the singular operation do not agree with the plural food service providers.

B. Operation must become the plural operations; also, placing sustainable before locally-grown is unnecessary and does not change either the structure or the content of the sentence.

C. The plural operations agrees with food service providers and no unnecessary word order changes have been made.

D. The verb form are transitioned does not make sense in the sentence; also, the placement of sustainable before locally-grown is unnecessary.

E. Operation is singular and does not agree with food service providers. Also, nothing in the sentence indicates that the present progressive are transitioning is needed.

The correct answer is C

Page 15: Economist

In surprising medical news, scientists investigating long-term effects of stroke are reporting that injury to a specific part of the brain, located behind the ear, can instantly and permanently break a smoking habit.

brain, located behind the ear, can instantly and permanently break a smoking habit

brain located behind the ear can instantly and permanently break a smoking habit

brain, located behind the ear, can instantly and permanently break the habit of smoking

brain located behind the ear, can instantly and permanently break a smoking habit

brain, the one located behind the ear, can instantly and permanently break the habit of smokingThe original sentence is correct as written. Commas are used correctly to separate the parenthetical expression located behind the ear from the rest of the sentence.

A. The phrase located behind the ear is correctly separated from the rest of the sentence by commas.

B. The lack of commas in this option turns the sentence into a run-on.

C. Though commas are used correctly here, the phrase the habit of smoking is wordy.

D. A comma is needed after the word brain to make the sentence grammatically correct.

E. The phrases the one located by behind the ear and the habit of smoking are both unnecessarily wordy.

The correct answer is A.Which of the following best completes the passage below?

Economic sanctions that forbid trade with countries with dictatorial governments assume that the dictator will change policies unacceptable to the imposing nation

Page 16: Economist

when the country’s citizens lose sufficient access to necessities such as food, water, and clothing. This approach occasionally proves unsuccessful, however, especially when  .

citizens of the country are accustomed to being poor

the dictator has ideological reasons for maintaining national policy

the country imposing the sanctions relies on exports from the subject country

the dictator is not moved by the suffering of his people

citizens of the country do not understand the leader’s policiesSituation: Economic sanctions imposed on countries run by dictators assume that policies will be changed once the country’s citizens begin to run low on necessities.

Reasoning: Which option provides a reason that sanctions imposed in this manner might be unsuccessful? This approach assumes that a dictator will change his policies when citizens begin to suffer due to the lack of necessities that results from the sanctions. If, however, the dictator does not care that citizens are suffering, this approach is very likely to prove unsuccessful.

A. What citizens are accustomed to has little bearing on the reaction of a dictator.

B. This option ignores the connection made in the first sentence of the passage between sanctions and citizens’ access to necessities.

C. Exports are not mentioned in the passage as being key to the success of the strategy of sanctions.

D. This option correctly addresses the connection made in the first section of the passage between desired changes of policies and the citizens’ loss ofsufficient access to necessities. If the dictator does not care about the suffering of his people, the policies will not change.

E. Citizen understanding of the dictator’s policies is not a key to successful sanctions.

The correct answer is D

Page 17: Economist

Federal regulations require that corporations use separate accounting firms for audit and non-audit services. This presents difficulties for many multi-national companies because there are only four large international accounting firms based in the United States. An outspoken group of CEOs has suggested breaking up the “Big Four” firms into smaller operations, so that corporations will have more options for their accounting needs.

Which of the following is most like the CEOs’ plan in logical structure?

The owners of a prosperous, family-run bakery with several food preparation facilities decide to sell various buildings to local restaurants and coffee shops so that they can retire.

A federal court rules that a communications conglomerate is in violation of anti-trust laws and must break into smaller companies to enable competition from local businesses.

A school district superintendent writes a letter to the publisher or his district’s textbooks complaining about lack of variety and suggesting that the publisher add new divisions with different emphases to their corporate structure.

A subsidiary of a large textile manufacturer is bought out in a hostile takeover, and its operations are changed.

The board of a hospital decides to retain only a small number of staff physicians and to rent out remaining space to specialists in private practice.Situation: A group of CEOs has proposed that the Big Four accounting firms be broken into smaller firms so that corporations will have more options for audit and non-audit services.

Reasoning: Which option is most similar to the plan in logical structure? In order to answer this question, first paraphrase the logical structure of the plan in the passage. First, it is important to note that the plan is being proposed, not by leaders of the Big Four firms or by legislators, but by a group of corporate CEOs who have no real power to compel compliance with their suggestions. Second, it should be observed that driving this suggestion is the need for more options, or variety; since corporations are federally required to use separate firms for audit and non-audit services, lack of

Page 18: Economist

variety creates the difficulty. Lastly, note that the proposed solution has to do with restructuring an already functioning operation in order to provide this additional variety.

A. This plan is proposed by the owners of a company, who have the power to make whatever changes they may desire.

B. Here a federal court, which has the power to enforce its decisions, is the entity proposing the plan to break up a larger corporation.

C. This option correctly identifies several similarities with the original passage: the school superintendent does not have power to enforce his suggestion of corporate restructuring, but his complaint does stem from a lack of options in textbook choices.

D. A hostile takeover has already changed operations; this option is not concerned at all with lack of variety.

E. This option does concern the restructuring of a business, but the restructuring is done by those in charge of the business. Also, lack of options is not mentioned.

The correct answer is C.Federal regulations require that corporations use separate accounting firms for audit and non-audit services. This presents difficulties for many multi-national companies because there are only four large international accounting firms based in the United States. An outspoken group of CEOs has suggested breaking up the “Big Four” firms into smaller operations, so that corporations will have more options for their accounting needs.

Which of the following stipulations would be most helpful in assuring the success of the CEOs’ plan to provide more variety in accounting services by breaking up the Big Four firms?

The firms should maintain their multi-national contacts.

CEOs for the new companies should be chosen from inside each firm.

Corporations must keep the same firm for their audit services, but should choose a new firm for non-audit needs.

Page 19: Economist

The new firms should maintain their internal audit procedures.

The Big Four firms should divide so that the audit and non-audit sections are not broken up.Situation: A group of CEOs has proposed that the Big Four accounting firms be broken into smaller firms so that corporations will have more options for audit and non-audit services.

Reasoning: Which added provision will help assure the success of the CEOs’ plan? The CEOs suggest breaking up the Big Four firms so that corporations can have more choices for their audit and non-audit services, which must, by federal regulation, not be performed by the same firm. Anything that further insures that audit and non-audit services will be kept separate in breaking up the firms will also assure that CEOs will get the added variety they are seeking.

A. This option does not directly impact the question of variety.

B. The origin of new CEOs does not deal with variety or with the separating of audit and non-audit services.

C. This provision specifies what decisions corporations may be allowed to make, but it does not insure variety.

D. This option does not directly impact the question of variety.

E. If each Big Four firm breaks into two – one performing audit services, and one performing non-audit services – then the field will have gained the variety sought by CEOs.

The correct answer is E.The 19th century group known popularly as the Martha Washingtonians was a temperance society composed of wives, mothers, sisters, and other female relatives of drunken men. Though some historians would say that the women in this society were progressive simply because they often behaved in ways uncommon to women of the time – taking to the streets to hand out flyers or to publicly exhort individuals – it is more appropriate to consider them a conservative group. They had no desire to change America in any way besides alcohol consumption.

According to its use in the passage above, which of the following is the best synonym for the word “conservative”?

Page 20: Economist

opposed to alcohol

affectionate toward the suffering

politically active

averse to change

respectful of traditionSituation: The author of the passage argues that the Martha Washingtonians were a conservative rather than a progressive society.

Reasoning: Which is the best synonym for conservative as it is used in the passage? To understand the meaning of the word conservative, review its context in the passage. The author is arguing that the women of the group might be considered progressive because they did things that women of the time usually did not do. The author argues that this is not the case, and in the last sentence of the passage gives a reason that the Martha Washingtonians should be considered conservative: they had no desire to change America in any way besides alcohol consumption.

A. It is true that the group was opposed to alcohol, but this is not the meaning of the word conservative.

B. This is not the meaning of conservative in the passage.

C. Martha Washingtonians were politically active, but this is not the meaning of conservative.

D. A contrast is made in the passage between the words progressive and conservative, and conservative is defined in the last sentence of having no desire to change America in any way besides alcohol consumption.

E. This is one possible meaning of conservative, but it is not the one used in the passage.

The correct answer is D.Although quite powerful in his time, the 16th century Russian czar Ivan the Terrible seems to us as a remote and barely visible historical figure.

Page 21: Economist

to us as a remote and barely visible historical figure

to us to be as a remote and barely visible figure of history

to us a remote and barely visible figure of history

to us a remote and barely visible historical figure

to us to be a remote and barely visible historical figureThis sentence tests both the use of the word as and the ability to recognize concise constructions. The word as means in the capacity of, and is therefore not necessary to the meaning of this sentence. In fact, the word makes the sentence unnecessarily wordy, as do several of the incorrect answer choices.

A. As in this option makes the sentence unnecessarily wordy.

B. The words to be are redundant, and the word as is unnecessary; also, the phrase figure of history is wordy.

C. Figure of history is wordy and unnecessary.

D. The unnecessary word as has been eliminated, and the phrase barely visible historical figure is appropriately concise.

E. The words to be are redundant and unnecessary.

The correct answer is D.The field of exobiology seeks to understand the origin, evolution, and distribution of life in the universe. Exobiology as a discipline emerged shortly after a 1953 experiment conducted at the University of Chicago that used electricity and simple gases to produce amino acids, the building blocks of proteins and, therefore, organic life as we know it. The experiment was conducted by a biology graduate student, Stephen Miller, in the lab of Harold Urey, a scientist who was already well-known for his theories on the composition of the early earth’s atmosphere. Miller and Urey put three gases believed to have been present in the early atmosphere – methane, ammonia, and hydrogen – into a closed system with an “ocean” of water at the bottom and ran electricity through the gases to simulate the lightening storms believed common on the early earth.At the end of a week, the scientists observed that basic organic compounds, including amino acids, had formed and dissolved in the water, where some of them had even combined to form more complex, though still non-living, substances.

Page 22: Economist

Though parts of the Miller-Urey experiment have been thrown into question by more recent discoveries, exobiology remains a small, close-knit field of dedicated scientists with backgrounds in areas as diverse as biology, chemistry and physics. Much of the current research supported by NASA, the American space agency, focuses on four major areas. The first two speculate about the evolution of compounds necessary to organic life, whether that be the development in space of “biogenic,” or life-generating compounds, or the planetary evolution of compounds that are considered “prebiotic,” or pre-living. A third area seeks to trace steps of transformation from organic compounds to simple forms of life, and a fourth studies the evolution from simple organisms to the complex and diverse forms of life found on earth today. Investigations in all four areas involve a mixture of approaches: observation of simple life-forms, such as bacteria and viruses; examination of materials from space, such as asteroids, which seem to contain evidence of life; speculation based on data gathered from telescopes and space probes; and experimentation.

The primary purpose of the passage is to

speculate about the origin of life in the universe

overturn an outdated, though popular, scientific theory

contrast two approaches to the study of the origin of life

trace the development of a field of study

discuss the origins and goals of a branch of scienceMain Idea questions require a true statement about information provided in the passage as a whole. The first sentence of the paragraph contains a definition of exobiology, and the remainder of the passage further supports this definition by discussing the origins of the field (paragraph 1) and its goals (paragraph 2).

A. This is a goal of exobiology itself, not this passage.

B. The passage does not suggest a single theory; therefore, a theory can not be overthrown.

C. Two approaches are neither introduced nor contrasted in the passage.

D. This is the main idea only of paragraph 1.

E. This option correctly summarizes information contained in both paragraphs of the passage.

The correct answer is E.

Page 23: Economist

Based on information in the passage, which of the following can be concluded about organic compounds?

Organic compounds  .

are not living, but are the building blocks of life

were discovered in 1953, as part of a famous experiment

can be formed more quickly than inorganic compounds

combine only in the presence of electricity

contain gases such as ammonia, methane, and hydrogenThe phrase based on information in the passage in the question stem reveals that the answer to this question can be found by reviewing what is stated explicitly in the passage. This line states that amino acids are the building blocks of proteins and, therefore, organic life as we know it. Combining this information with that provided inthis line, which further emphasizes that amino acids areorganic compounds, it can be concluded that organic compounds are the building blocks of life even though they are not living.

A. This information can by found by combining relevant sentences in the first paragraph.

B. The paragraph states that the experiment was conducted in 1953, but not that organic compounds were discovered then.

C. This claim is not made in the passage.

D. The passage does not make this claim.

E. Though this is true of the organic compounds discussed in the first paragraph, the passage does not make this claim for all organic compounds.

The correct answer is A.The field of exobiology seeks to understand the origin, evolution, and distribution of life in the universe. Exobiology as a discipline emerged shortly after a 1953 experiment conducted at the University of Chicago that used electricity and simple gases to produce amino acids, the building blocks of proteins and, therefore, organic life as we know it. The experiment was conducted by a biology graduate student, Stephen Miller, in the lab of Harold Urey, a scientist who was already well-known for his theories on the composition of the early earth’s atmosphere. Miller and Urey put three gases believed to have been present in the early atmosphere – methane,

Page 24: Economist

ammonia, and hydrogen – into a closed system with an “ocean” of water at the bottom and ran electricity through the gases to simulate the lightening storms believed common on the early earth.At the end of a week, the scientists observed that basic organic compounds, including amino acids, had formed and dissolved in the water, where some of them had even combined to form more complex, though still non-living, substances.

Though parts of the Miller-Urey experiment have been thrown into question by more recent discoveries, exobiology remains a small, close-knit field of dedicated scientists with backgrounds in areas as diverse as biology, chemistry and physics. Much of the current research supported by NASA, the American space agency, focuses on four major areas. The first two speculate about the evolution of compounds necessary to organic life, whether that be the development in space of “biogenic,” or life-generating compounds, or the planetary evolution of compounds that are considered “prebiotic,” or pre-living. A third area seeks to trace steps of transformation from organic compounds to simple forms of life, and a fourth studies the evolution from simple organisms to the complex and diverse forms of life found on earth today. Investigations in all four areas involve a mixture of approaches: observation of simple life-forms, such as bacteria and viruses; examination of materials from space, such as asteroids, which seem to contain evidence of life; speculation based on data gathered from telescopes and space probes; and experimentation.

The author refers to an “ocean” most likely in order to

suggest that gases are not sufficient to create organic compounds.

emphasize that the experiment sought to replicate conditions of early earth.

shed light on the mindset of scientists conducting the experiment.

disprove the theory that early earth’s oceans were of a substance other than water.

provide an example of a substance common to early earth.The answer to this question depends on drawing a conclusion based on information stated in the passage. To begin, review the highlighted text where the ocean is mentioned. The reference occurs within the context of Miller and Urey’s experiment, in which they attempted to re-create conditions present on the early earth to test whether organic compounds could be formed. It can be concluded, then, that the word ocean is meant to be included in the list of conditions replicating the situation of the early earth.

Page 25: Economist

A. Though water and electricity are also used to create organic compounds in this experiment, this has no bearing on the fact that the author chose the word ocean.

B. This option correctly recognizes the placement of the word ocean within the context of an experiment that sought to replicate conditions present on the early earth.

C. The word ocean is in itself irrelevant to the mindset of Miller and Urey.

D. This theory is neither mentioned nor refuted in the passage.

E. This option is close to being correct, but does not correctly evaluate the word ocean in the context of an experiment which sought to recreate certain conditions.

The correct answer is B.According to the passage, EACH of the following is an area of investigation for exobiology EXCEPT the

development in space of compounds that have potential to generate life.

evolution of simple life forms into more complex organisms.

examination of objects from space to seek evidence of non-earthly life.

potential uses of organic compounds not found on earth.

interpretation of data gathered from outer space research.The answer to this question can be found by checking each statement one at a time against information stated explicitly in the passage.

A. Biogenic, or life-generating compounds, are mentioned in this line

B. This type of evolution is mentioned in this line(and a fourth studies the evolution from simple organisms to the complex and diverse forms of life found on earth today) as one of the areas of investigation for exobiology.

C. Examination of objects such as asteroids is mentioned in this line (examination of materials from space, such as asteroids, which seem to contain evidence of life).

D. The passage does not discuss the potential uses of organic compounds not found on earth.

Page 26: Economist

E. This area of investigation is mentioned in this line(speculation based on data gathered from telescopes and space probes).

The correct answer is DIt can be inferred from the passage that which of the following is true about the field of exobiology?

Exobiology is more concerned with finding life on other planets than with determining the origin of life on earth.

Research in exobiology is funded primarily by world governments.

Research in exobiology combines knowledge from other branches of science such as chemistry and physics.

Exobiology continues to follow the premises introduced by Miller and Urey.

Exobiology is concerned only with life on earth and earth-like planets.An inference is drawn from stated information. To find the answer to this question, review what is stated explicitly in the passage about the field of exobiology. Check each answer choice against a positive statement involving exobiology and determine whether the stated inference can be appropriately drawn.

A. The first sentence states that exobiology seeks to understand the origin, evolution, and distribution of life in the universe, but the passage does not state which areas of life are of most concern to exobiologists.

B. The passage states in this line that exobiological research is funded by NASA in the United States, but this is not sufficient evidence to conclude that all research is funded by world governments.

C. The first sentence of the second paragraph states that scientists working in exobiology can have backgrounds in biology, chemistry, and physics. It is reasonable to conclude, then, that the field combines knowledge and expertise from all of these areas.

D. On the contrary, the first sentence of the second paragraph suggests that Miller and Urey’s experiment has fallen out of favor.

E. This line  states that exobiology is concerned with the development of biogenic compounds in space, and this line states that space data gathered from telescopes and space probes is also essential.

Page 27: Economist

The correct answer is C.President of Venezuela during the mid-1950s, the country’s economy flourished under Marcos Perez Jimenez.

President of Venezuela during the mid-1950s, the country’s economy flourished under Marcos Perez Jimenez.

Venezuela’s economy flourished during the mid-1950s under President Marcos Perez Jimenez.

Under Marcos Perez Jimenez, president of the country during the mid-1950s, Venezuela’s economy flourished.

The country’s economy flourished under the president of Venezuela during the mid-1950s, Marcos Perez Jimenez.

During the mid-1950s, the country’s economy flourished under Marcos Perez Jimenez, president of Venezuela.The entire original sentence is unclear and must be rearranged. The correct answer will maintain the logical relationship of all elements – the name of the country, the name of the president, the time period, the fact that the economy flourished – but will reorder them in such a way that the meaning is more direct.

A. This sentence places the phrase President of Venezuela in an unclear relationship with the phrase the country’s economy.

B. This sentence is clear, and its meaning is straightforward.

C. This option clarifies the meaning of the original, but its wording is rather awkward.

D. The wording of this sentence is bulky and somewhat confusing.

E. The phrase the country’s economy is logically separated from the word Venezuela in such a way that the sense of the sentence is destroyed.

The correct answer is B.In recent years, directors of remedial reading programs have struggled to find a way to provide their students with sustained independent reading practice in texts that challenge students to apply their phonics and word-decoding skills without

Page 28: Economist

overwhelming them. In response to this quandary, Roger Wade, the director of a private corporation known as the Reading Readiness Institute, piloted a system of in-class assessment that would, ideally, have allowed teachers to make individual recommendations for the difficulty level of book where each student could read independently with the most success. The six Institute teachers testing the program were required to build two 10-15 assessment periods into each class session, during which they would use company-issued Reading Rating Sheets to evaluate each student’s performance on a specific section of text, which came from a book of pre-determined difficulty. Teachers would then assign students one of three rankings – A, B, or C – based on their comfort with the text on the Rating Sheets. Parents were also required to complete short Homework Surveys, which provided teachers with additional, but similarly-focused, information about student reading at home.

The program was temporarily discontinued after only 6 weeks, however, when all six teachers protested that the addition of assessment periods had forced them to drop other activities that they considered vital from their curricula. Teachers also complained that time spent assessing individual students took attention from general classroom management. The Reading Readiness Institute has taken teacher complaints into account and is currently adjusting its individual assessment program

Which of the following titles best summarizes the contents of the passage?

Remedial Reading Programs: A New Focus on Phonics and Word-Decoding Helps Students

Parents and Teachers Together: A New Perspective on Homework Feedback

Rejected Outright: Reading Readiness Institute Abandons Individual Assessment

Reading Readiness Institute: Corporate Demands Create Trouble for Teachers

Individual Assessment: A Good Idea Encounters its First ObstaclesTo answer this question about possible titles, the intention of the entire passage must be taken into account. The first couple of sentences introduce a problem faced by remedial reading programs, and the following sentences in the paragraph describe the ways the Reading Readiness Institute’s assessment program is designed to solve this problem. The second paragraph discusses problems in the program’s implementation and states that it is being revised. The title of the passage must take all of this information into account.

Page 29: Economist

A. The passage focuses only on the Reading Readiness Institute’s program.

B. Homework Feedback is mentioned in the second paragraph, but it is not the subject of the entire passage.

C. The words rejected outright and abandons are too strong for the sense of the passage. The last sentence states that the program is being revised.

D. Corporate demands are not mentioned explicitly in the passage.

E. This title correctly identifies the fact that the focus of the passage is on individual assessment as an approach to helping students. The second part of the title acknowledges the fact that the author seems optimistic about assessment, even while admitting to the problems relayed by teachers.

The correct answer is E.The passage suggests that which of the following is a problem faced by current remedial reading programs?

Teachers are too slow to evaluate the needs of individual students.

In-class time is poorly managed.

Parent involvement is not well coordinated.

Practice texts are often overwhelming to students.

Potentially useful programs are too short-lived to be effective.This question requires an inference drawn from information stated explicitly in the passage. The problem faced by remedial reading programs is discussed in the first sentence, where it is stated that program directorshave struggled...to provide their students with independent reading practice without overwhelming them. It is reasonable to conclude, then, that practice texts in remedial reading programs are often overwhelming to students.

A. This conclusion is not supported by the passage.

B. On the contrary, the passage seems to imply that class time is managed well, and that the assessment program has disrupted that.

C. There is no basis for reaching this conclusion from information in the passage.

D. This option correctly identifies the problem stated in the first sentence of the passage.

Page 30: Economist

E. Though the individual assessment program may be classified as short-lived, this is not the problem discussed by the passage.

The correct answer is DIt can be inferred from the passage that a successful revision of the individual assessment program would result in

a way to make individual recommendations for students without over-burdening teachers

the resumption of class activities teachers consider vital to their curricula

a challenge to students of their phonics and word-decoding skills

the omission of at least one in-class assessment period

better cooperation among Institute executives, teachers, and parents of studentsBecause this is an inference question, the answer is not stated directly in the passage, but must be drawn from the information given. To begin, review the sections of the passage where the teachers’ complaints are discussed; if the program has been temporarily discontinued and is awaiting revision, it is reasonable to assume that any adjustments to the program will seek to eliminate the problems of which the teachers complained. Teachers complained that they had to drop other activities and that assessment took time from general classroom management. A successfully revised plan would not overburden teachers in this way.

A. This option correctly identifies both the complaints made by teachers and a way to eliminate the need for such complaints.

B. Though this option provides part of the answer, it does not go far enough to encompass all teacher complaints.

C. Remedial reading programs already do this.

D. This option is too specific to deal with the general problem of teacher complaints.

E. Cooperation is not mentioned as an issue needing attention in a revised version of the program.

The correct answer is A

Page 31: Economist

Roger Wade, the director of a private corporation known as the Reading Readiness Institute, piloted a system of in-class assessment that would, ideally, have allowed teachers to make individual recommendations for the difficulty level of book where each student could read independently with the most success.The author uses the word “ideally” primarily in order to

suggest that the individual assessment program was doomed from the beginning.

introduce the concept of individual recommendations for independent reading.

highlight the good intentions of the assessment program while acknowledging its failure.

forestall argument about the eventual success of the program.

draw attention to the fact that the program was the idea of the Institute’s director.To find the purpose of this word choice, review the context in which it occurs. It occurs in this line (piloted a system of in-class assessment that would, ideally, have allowed teachers to make individual recommendations for the difficulty level of book where each student could read independently with the most success), in the middle of a sentence linking ideas about a problem found in remedial reading programs with a solution proposed by the director of the Reading Readiness Institute. After finishing the passage, it becomes clear that the program was not ultimately successful; the word ideally, then, serves as a signal that performance did not live up to intention. The overall tone of the passage, however, is positive, indicating that the author likely considers individual assessment a good idea.

A. Though the assessment program was not ultimately successful, the word doomed is too strong for the general tone of the passage.

B. The word ideally does not serve to introduce this concept.

C. The word ideally indicates that author’s sense that the program had a noble goal, even while acknowledging that it was ultimately discontinued.

D. There is no argument to be prevented. The passage states that the program was temporarily discontinued.

E. The word ideally does not serve this function in the passage.

Page 32: Economist

The correct answer is CReporting that one of its many obstacles had been a citizens’ group seeking to preserve historic buildings, developers announced a revision of plans to tear down the city’s original post office in order to expand a luxury hotel’s parking lot.

its many obstacles had been

its many obstacles had turned out to be

its many obstacles is

their many obstacles had been

their many obstacles isThis sentence tests agreement between a possessive noun and its antecedent, as well as the correct use of verb tense. The antecedent developers, even though it comes after the underlined possessive pronoun, still requires the plural pronoun their. Additionally, the developers are just announcing a revision of plans based on previous problems, indicating that the correct verb form is had been.

A. The possessive pronoun its does not agree with the plural developers.

B. Its does not agree with developers, and this option is unnecessarily wordy.

C. The verb form is is incorrect in this sentence.

D. The pronoun their agrees with the antecedent developers, and the verb form had been is appropriate to the sentence.

E. Is is incorrect verb form for this sentence.

The correct answer is DCzech-born American tennis player Martina Navratilova not only reached the Wimbledon singles finals an impressive 12 times, holding a record for the most women’s singles titles won at Wimbledon: 9.

holding a record for the most women’s singles titles won at Wimbledon: 9

holding a record of 9, the most women’s singles titles won at Wimbledon

Page 33: Economist

but she also holds a record of 9, the most women’s singles titles won at Wimbledon

but she also, having won 9 women’s singles titles, holds a record at Wimbledon

but she also holds a record for the most singles titles won at Wimbledon: 9The idiom being tested in this sentence is the construction not only...but also. The not only reached in the first part of the sentence requires a parallel but also + past tense verb in the second part. The original sentence’s failure to use this construction correctly has also resulted in an incomplete sentence.

A. This option does not correctly use the idiom not only...but also, and the word holding at the beginning of the clause makes the sentence incomplete.

B. Holding at the beginning of the clause forms an incomplete sentence because it does not correctly use the idiom not only...but also. Additionally, this option is wordy.

C. The phrase she also holds a record of 9 does not make sense.

D. This option correctly uses not only...but also, but it is unnecessarily wordy.

E. The idiom not only...but also is correctly used here, and the verb phrase she also holds forms a complete sentence.

The correct answer is EOur reading program teaches third graders to use phonics to sound out multi-syllabic words by pronouncing prefixes and suffixes separately from base words, and then reading the word parts all together. Since children are still primarily reading aloud at this age, this approach ensures comprehension even in books that contain a high percentage of multi-syllabic words.

The approach of the summer reading program assumes which of the following about third graders’ comprehension of multi-syllabic words?

Students will recognize multi-syllabic words when they hear them.

Multi-syllabic words are all composed of prefixes and suffixes.

Page 34: Economist

Third graders struggle more than other students with multi-syllabic words.

Comprehension is less important than sounding out words.

Other reading programs do not focus on multi-syllabic words.Situation: A reading program teaches third-graders to sound out multi-syllabic words in order to understand them.

Reasoning: Which identifies the assumption underlying the program’s approach? The reading program teaches students a way of breaking down words and sounding them out. This strategy, together with the fact that children are still primarily reading aloud at this age, makes it clear that children are expected to recognize the words once they hear them.

A. The program’s emphasis is on using sounds to figure out long words, making clear the assumption that children will recognize the words if they can only pronounce them.

B. The passage does not make this claim for all multi-syllabic words.

C. Third graders are not compared with other students in the passage.

D. In fact, the passage states that words are sounded out so that comprehension can be ensured.

E. Other reading programs are not compared in the passage.

The correct answer is AAlthough there is a serious shortage of potable water in the area where the new sub-division is being built, the developer has contracted a company to deliver 400 gallons a day to assure that the workers remain fully hydrated.

Although there is a serious shortage of potable water

Because of a serious shortage of potable water

Because of a potable shortage of water

In the event of a serious shortage of potable water

Page 35: Economist

Even though there is a serious shortage of potable waterIn this sentence, the first clause is dependent, and the second is independent. In order to choose the best introduction to the first part of the sentence, the correct relationship between the two clauses must be established. Here the first clause establishes the reasons for the action taken in the second clause; therefore, an introductory word demonstrating causation will best complete the sentence.

A. The word although does not make sense in this sentence because no contrast is being made.

B. The word because correctly demonstrates the relationship between the shortage of water and the need to have it delivered to workers.

C. The word potable is misplaced and modifies shortage rather than water.

D. In the event suggests a contingency plan and does not make sense in this sentence.

E. Even though draws an unnecessary contrast.

The correct answer is BThe art installation in Central Park, which featured over a hundred men and women dressed in army fatigues and lying on the ground as if they had been killed in battle, represented neither an anti-war nor a pro-military statement, but rather attempted to be commenting creatively on troubled times.

but rather attempted to be commenting creatively

but attempted a rather creative comment

but rather was an attempt to comment creatively

but commented rather creatively, or at least attempted to

but commented creatively, or, rather, attempted to commentThis question tests the appropriate revision of the incorrect verb form to be commenting in the original sentence. Also tested is the use of the word ratherto show contrast with ideas in the first part of the sentence, not as a modifier.

A. The verb form to be commenting is incorrect and nonsensical.

Page 36: Economist

B. Rather is misused as a modifier rather than a word that shows contrast.

C. Rather is correctly used to show contrast, and the fact that the word attempt has changed from a verb to a noun has also enabled the verbal phrase comment creatively to be used correctly.

D. Not only is the word rather used incorrectly, but this option is wordy.

E. The wording in this option is awkward and unclear.

The correct answer is CThe board of directors of the city art museum has begun discussing ways to deal with a large and unexpected budget shortfall. Several prominent members of the board suggest suspending free admission days, which occur every Tuesday, with the additional revenue to go directly into day-to-day operating costs.

The answer to which of the following questions is LEAST directly relevant to the board members’ consideration of whether suspending free admission days will correct the shortfall in the museum budget?

How many people are expected to pay for admission on days that were previously free?

Do most members of the board agree that this is the best solution?

What percentage of the budget shortfall are day-to-day operating costs?

Did the free admission days have any impact on creating the budget problems?

Will the money generated from the plan be sufficient to cover day-to-day operating costs?Situation: Members of the board of directors of a museum propose suspending free admission days to correct a budget shortfall.

Reasoning: The answer to which question is irrelevant in determining the success of the plan? To answer this question, consider what information can be gained by answering each individual question, then determine whether that information is relevant. The plan is to eliminate free entrance and to use money gained from

Page 37: Economist

charging admission to pay for daily operating costs. Any information that will suggest the success or failure of the plan is relevant, and that answer choice can be eliminated.

A. The answer to this question will provide information about how much money will be raised by suspending free admission days. It is relevant.

B. The answer to this question will not provide any directly relevant information about whether the plan will be successful. If the plan is not sound, it will fail even if most members do agree to implement it.

C. The answer to this question will tell how much of the budget shortfall can be corrected by using additional revenue to cover operating costs. It is relevant.

D. The answer to this question will allow board members to determine whether the free admission days were part of the budget problem. It is relevant.

E. Though covering daily operating costs may not completely correct the budget shortfall, knowledge of how much positive impact this aspect can have on the overall plan is relevant.

The correct answer is BAfter being upset by the conditions of animals she saw on tours of a cattle ranch and meat-packing facility, it was decided that Marta would become a vegetarian.

it was decided that Marta would become a vegetarian

it was decided by Marta that she would become a vegetarian

the decision that Marta made was to become a vegetarian

Marta’s decision was to become a vegetarian

Marta decided to become a vegetarianWho decided that Marta would become a vegetarian? According to the logic of the sentence, it is Marta herself who makes the decision, and the correct answer choice will make her the subject of the clause. Making Marta the subject of the clause will also change the sentence from passive voice to active voice, a marked improvement.

A. The phrase it was decided is vague and inappropriate.

Page 38: Economist

B. This option is excessively wordy, and the phrase it was decided by Marta does not attribute the action to Marta directly enough.

C. This option abuses passive voice to the point of almost not making sense.

D. Though this option makes Marta the subject of the clause, it is unnecessarily wordy.

E. This option correctly and succinctly makes Marta the subject of the clause and changes the original sentence into active voice.

The correct answer is E.It was nearly impossible to diagnose the girl’s illness even after three hospitalizations, several months of therapy, and a near mental breakdown because no one medical professional – neither her therapist nor her doctors – were able to see the full scope of her symptoms.

neither her therapist nor her doctors – were able to see

not even her therapist nor her doctors – were able to see

including her therapist and her doctors – were able to see

not even her therapist nor her doctors – was able to see

neither her therapist nor her doctors – was able to seeThis sentence tests the correct use of the phrase neither...nor as well as subject-verb agreement when subject and verb are split across a parenthetical phrase. The words no one before the hyphen indicate that the parenthetical expression must contain a negative expression, the word neither. Neither is idiomatically paired with the word nor. Also, the subject of the clause – no one medical professional – requires the singular verb was.

A. Were is a plural verb that does not agree with the singular no one medical professional.

B. Not even cannot be correctly idiomatically paired with nor, and were is an incorrect verb.

Page 39: Economist

C. Were is a plural verb and does not agree with the singular subject of the clause; also, the word including does not sufficiently respond to the wordsno one, which come before the hyphen.

D. The words not even are not idiomatically correct when paired with the word nor.

E. The phrase neither...nor is idiomatically correct, and the singular verb was agrees with the subject no one medical professional.

The correct answer is EThe city will lose more than one million dollars in much-needed revenue when the shopping center on Beacon Drive will close sometime in the next year and move to a different county.

will close sometime in the next year and move to a different county

closes sometime in the next year and moves to a different county

will close in the next year sometime and move to a different county

closes sometime in the next year and will move to a different county

will close sometime in the next year and then move to a different countyThis sentence requires the recognition that the word when creates a conditional tense that requires the (e)s form of the verb, and also that both verbs in the following phrase must use parallel form. The complete and correct verb phrase, without supporting words, should read: when the shopping center closes and moves.

A. Though the verbs in this option are parallel, they are not in the e(s) form required by the word when.

B. Closes and moves are the parallel verb forms required by the word when.

C. The verb forms will close and move are not the conditional form required by the word when, and the word sometime is awkwardly placed.

D. Verb forms closes and will move are not in parallel form.

E. Then is unnecessary, and the verb forms will close and move are not in the e(s) form required by the word when.

Page 40: Economist

The correct answer is B.A citizens’ group in State X is pushing legislation to have mobile phone use banned during driving. Leaders of the group are concerned that talking and text messaging while operating a motor vehicle has led to many injuries and to damage of state property.

EACH of the following is a piece of evidence the group could use in support of its position EXCEPT

statistics on how many licensed drivers own mobile phones

police accident reports stating that mobile phone use was a cause of a collision

scientific studies stating the effect of divided attention on motor vehicle safety

pamphlets from other citizens’ groups urging drivers to turn off mobile phones in traffic

testimonials from drivers who have been injured in accidents involving mobile phone useSituation: A citizens’ group wants to legally ban mobile phone use while driving because it is dangerous.

Reasoning: Which is not a piece of evidence that would be useful to the group? The citizens’ group’s claim is that mobile phone use has led to many injuries and that it has damaged state property. Any evidence that directly supports this claim would be useful. To answer this question, examine each potential piece of evidence and determine its usefulness.

A. This information would be useful in speculating how many drivers are likely to be injured if mobile phone use is not banned.

B. Police accident reports would directly confirm that mobile phone use has resulted – and is likely to continue to result – in accidents.

C. This information provides insight into how mobile phone use can cause motor vehicle accidents.

Page 41: Economist

D. This piece of evidence is not likely to support the cause of the citizens’ group because, first, it does not provide evidence to support the group’s claim and, second, because it does not directly address the issues that are the target of legislation.

E. Testimonials would provide valuable insight into how mobile phones can cause motor vehicle accidents.

The correct answer is D.Mount Washington, in the White Mountains of northern New England, routinely sees some of the coldest temperatures and highest winds in the United States each year, even though its elevation is a mere 6,288 feet. This is due partly to the fact that storms moving east to west across the country are directed by prevailing winds into the area. The mountain range is also positioned almost at a right angle to cold air sweeping down from Canada, which is compressed between two sections of the range and funneled directly onto Mount Washington.

Based on information from the passage, which of the following can be concluded about Mount Washington?

The highest wind speeds ever recorded in the US were documented on Mount Washington.

Few climbers visit Mount Washington, due to its cold temperatures and high winds.

Mount Washington receives the most snow of any peak in the White Mountains.

Mount Washington’s weather patterns are determined more by geography than by elevation.

Mount Washington is the highest peak in the eastern United States.Situation: The author discusses Mount Washington and the reasons it sees cold temperatures and high winds.

Reasoning: Which can be concluded about Mount Washington from information in the passage? The passage begins by stating the location of Mount Washington and that it experiences cold temperatures and high winds. The rest of the passage discusses

Page 42: Economist

weather patterns on Mount Washington in terms of where the mountain lies relative to other geographical features and wind directions.

A. Though the passage mentions high winds, it does not state that the highest winds speeds ever recorded in the US were recorded on Mount Washington.

B. This claim is not made in the passage.

C. The passage does not compare snowfall on Mount Washington to snowfall on other mountains in the range.

D. This option correctly identifies the passage’s focus on Mount Washington’s relative position in determining weather patterns.

E. This conclusion is not supported by the passage.

The correct answer is D.Utility companies in the United States are generally privately-owned, for-profit business ventures. One notable exception is the Tennessee Valley Authority, which was formed under the New Deal in the 1930s and continues, under federal control, to provide electricity to parts of the Southeast. Given the current pro-business climate in the region, however, it is safe to say that TVA is long overdue to be privatized.

The argument that the ownership of the Tennessee Valley Authority should be privatized rests on which of the following assumptions?

A pro-business climate demands the private ownership of utilities.

New Deal programs are outdated and should be discontinued.

Electricity in the Southeast will be more efficiently distributed if utilities are privately owned.

The price of electricity is currently higher than it would be if TVA were privatized.

Most residents of the Southeast prefer that TVA enter private ownership.Situation: The author argues that ownership of TVA, an electric utility formed during the 1930s, should be privatized due to a pro-business climate in the region.

Page 43: Economist

Reasoning: Which of the following is the underlying assumption? The author begins by stating that most US utilities are privately-owned, for-profitbusinesses. The Tennessee Valley Authority is contrasted to this, as a utility that is federally controlled. The reason, however, that the author argues TVA should be privatized is the pro-business climate in the region. It can be concluded, then, that the author assumes a pro-business climate should prompt privatization of the utility.

A. Since the author connects private ownership with a pro-business climate, it can be concluded that he believes TVA should be privatized for this reason.

B. New Deal programs are not the subject of this passage and are only relevant as they provide information about TVA.

C. This idea is neither present nor implied in the passage.

D. The price of electricity is not discussed in the passage.

E. The statement in the passage is that there is a pro-business climate in the Southeast; this does not necessarily mean that most residents would prefer private ownership of TVA.

The correct answer is A.

gMat 4

Soul music first rose to prominence in the United States in the 1960s, when artists were combining subtle African rhythms with gospel harmonies and themes, as well as influences from the newly emergent genre of rock-and-roll.

when artists were combining

during which many artists were combining

at which point artists began to combine

when artists began to combine

when artists who began to combine

Page 44: Economist

The content of the sentence concerns a point in time when soul music first became popular, making the use of the word began and simple past tense necessary.

A. The use of past progressive tense in this option makes it sound as if the activities described in the sentence continued throughout the 1960s; the option does not correctly discuss a starting point for soul music.

B. During which is redundant and unnecessary.

C. The phrase at which point is wordy and unnecessary.

D. This option correctly identifies the need to use simple past tense to identify a specific point in time when soul music began to be popular.

E. The word who in the option makes the sentence incomplete.

The correct answer is D.The support group for mothers of soldiers emphasizes in its introductory literature that acts of heroism have been performed every day by young men and women born of American families.

have been performed every day by young men and women born of American families

every day have been performed by young men and women, born in American families

are performed, every day, by young men and women born of American families

are performed every day by young men and women born in American families

are performed every day by young men and women born of American familiesThis sentence is testing two skills, one at the beginning of the underlined portion, and one at the end. The first requires that the sentence be kept in simple present tense. There is nothing to indicate a past action; rather, the support group emphasizes...acts of heroism that are ongoing. Secondly, the sentence tests the correct idiomatic use of the preposition in to indicate being part of a family through birth.

Page 45: Economist

A. The preposition of is misused, and past perfect tense – have been performed - is unnecessary.

B. Every day at the beginning of this option makes the sentence awkward, and the comma is unnecessary.

C. Of is misused, and the commas surrounding every day are unnecessary.

D. This option correctly uses the phrase are performed to indicate present tense, as well as identifies the correct use of the preposition in with the verb born.

E. The preposition of is misused.

The correct answer is D.The city council has installed alarm systems in some of the neighborhood’s vacant houses to keep out people hoping to steal lights and other fixtures, drug users and squatters.

people hoping to steal lights and other fixtures, drug users and squatters

drug users hoping to steal lights and other fixtures, and squatters

drug users, squatters, and people hoping to steal lights and other fixtures

people hoping to steal lights, other fixtures, drug users, and squatters

drug users and squatters, and people hoping to steal lights and other fixturesThe way the original sentence is phrased, it sounds like the people who will break into vacant houses are hoping to steal a series of things: lights, fixtures, drug users, and squatters. The correct answer choice will regroup items in the list to make it clear that the sentence intends to list the people it wants to keep out of the houses, rather than the items they wish to steal.

A. Because the phrase drug users and squatters is misplaced, it sounds as if these people are in danger of being stolen.

B. The sentence does not suggest that drug users will steal from vacant houses.

C. This option correctly rearranges the items in the series to make it clear that there are people who should be kept out of vacant houses.

Page 46: Economist

D. The comma between lights and other fixtures creates an option that does not make logical sense.

E. The and between drug users and squatters makes this option unnecessarily wordy.

The correct answer is C.Many conservative voters who had previously supported the candidate changed their minds when they learned that, in addition to his strong stance against sexual offenders, he is a lifelong collector of pornographic books and art.

in addition to

in spite of

as a consequence of

in opposition to

concurrently withThe fact that voters changed their minds about the candidate in question indicates that they find something objectionable about him. The voters’ reactions indicate that the underlined word should contrast the candidate’s strong stance against sexual offenders and the fact that he is a lifelong collector of pornographic books and art.

A. In addition to indicates a continuation of an idea, not the contrast called for by the logic of the sentence.

B. The idiomatic phrase in spite of correctly identifies the contrast perceived by voters between a situation the candidate opposes and a lifelonghobby.

C. The sentence does not suggest that the candidate’s collection is a consequence of his opposition to sexual offenders.

D. The phrase in opposition to expresses contrast, but does not conform to the logic of the sentence.

E. Concurrently with does not express the contrast required by the sentence.

The correct answer is B.

Page 47: Economist

Which of the following best completes the passage below?

The 1906 San Francisco Earthquake destroyed over half the city and changed the development of the California economy, but much of the damage was actually caused by fire. It has been estimated that as much as 90% of the total destruction was a result of fire damage rather than movement of the earth. This figure is likely exaggerated, however, because the nearly universal practice of insuring San Francisco properties against fire but not earthquake damage all but guaranteed that  .

most damage to the city was blamed on fire

the city would eventually be rebuilt

insurance companies were forced to offer earthquake coverage

residents subsequently moved to other cities where earthquake coverage was available

buildings not damaged by fire were never repairedSituation:Although much of the damage during the 1906 San Francisco Earthquake was caused by fire, the estimate of 90% fire damage is likely an exaggeration.

Reasoning:Which gives a reason for exaggerating the amount of damage caused by fire? The passage maintains throughout a contrast between earthquake and fire damage, although both were part of the destruction of San Francisco. The passage claims that much damage was actually caused by fire, but also states that the figure of 90% fire damage is likely exaggerated. The word because in the third sentence indicates that the blank must be filled by a reason San Francisco residents would claim more fire damage than actually occurred; insuring indicates that the claim had something to do with insurance coverage.

A. If property owners could only claim insurance money for damage caused by fire but not earthquake, they would be likely to report any damage to their property as fire damage.

B. Rebuilding of the city is outside the scope of the passage.

C. The passage does not discuss changes in insurance coverage policies.

D. Residents’ eventual movements are outside the scope of this passage.

Page 48: Economist

E. The repair of certain buildings is not a concern of the passage.

The correct answer is AIn an effort to stop the recent wave of foreclosures in middleclass neighborhoods, two counties outside Atlanta have launched anti-foreclosure campaigns, while several towns south of Chicago are forcing titleholders to either improve empty houses or repay the government for doing so.

forcing titleholders to either improve empty houses or repay

either forcing titleholders to improve empty houses or to repay

forcing either titleholders to improve empty houses or repaying

either improving titleholders’ empty houses or forcing them to repay

forcing the improvement of titleholders’ empty houses or repayingThis sentence tests the use of the phrase either…or to create a sentence that is both grammatically and logically correct. The sentence is concerned with the a recent wave of foreclosures and the steps that certain cities are taking to deal with the consequences. According to the passage, two counties outside Atlanta have begun anti-foreclosure campaigns; the logic of the sentence requires that the underlined portion also discuss ways of dealing with foreclosures: either requiring that owners fix up their houses or letting the government fix them up and requiring owners to repay the costs. The options available to homeowners must also be in parallel form.

A. This option places the phrase either…or so that the sentence correctly describes the choices available to titleholders in towns south of Chicago: they can either improve empty houses or repay the government for making improvements on their behalf.

B. Either is misplaced, making this option logically incorrect, and the words forcing and to repay do not display needed parallel form.

C. The word either is misplaced, making this option logically incorrect.

Page 49: Economist

D. Though grammatically correct, this option does not make logical sense, because it is the titleholders, not several towns south of Chicago who must improve empty houses.

E. This option is wordy and confusing.

The correct answer is A.As part of a unified scholarly approach, Symeon, an Eastern Orthodox saint often called “The New Theologian,” collected texts describing existing traditions and relayed them to his readers in a tone of great devotion and authority and borrowing from Asia some aspects of mystical practice.

authority and borrowing from Asia some aspects of mystical practice

authority, and borrowed from Asia some aspects of mystical practice

authority along with the aspects of mystical practice he borrowed from Asia

authority, borrowing from Asia some aspects of mystical practice

authority borrowing some aspects of mystical practice from parts of AsiaThe sentence describes Symeon’s scholarly process in a combination of verb forms. The first two are past tense forms: collected and relayed. The form of the third verb, borrow, can grammatically be either the past tense borrowed, used without a comma but with the word and, or the –ing form borrowing, which must be preceded by a comma and used without and. Given the answer choices, only the latter option is correct.

A. The combination of the words and and borrowing is confusing and incorrect.

B. The comma before and is not necessary, since the second part of the sentence is not a dependent clause.

C. This option displaces the verb borrowed and is unnecessarily wordy.

D. This option correctly uses the –ing form borrowing, which follows a comma to create a grammatically correct sentence.

E. This option is missing a comma before the word borrowing, and the phrase from parts of is wordy and unnecessary.

Page 50: Economist

The correct answer is D.Many students are surprised to find that objects viewed through a pseudoscope, an optical instrument that reverses depth perception, appears inside out; a box, an example, would seem to be a square-shaped hole in the floor.

appears inside out; a box, an example

appears inside out, for example; a box

appears inside out; a box, for example

appear inside out, for example; a box

appear inside out; a box, for exampleThis sentence tests both the idiomatic usage of the phrase for example and the ability to recognize correct subject-verb agreement when the subject and verb are split by a parenthetical expression. In this sentence the plural subject objects requires the plural verb appear.

A. The singular verb appears does not agree with the plural subject objects, and an example is incorrect idiomatic usage.

B. The phrase for example is misplaced and does not make sense in the sentence.

C. Objects as a plural subject requires the plural verb appear.

D. For example is misplaced and does not make sense in the sentence.

E. The plural verb appear agrees with the plural subject objects, and the phrase for example is used correctly.

The correct answer is E.According to a recent study linking high social standing to a habitually stern facial expression, models who do not smile adequately in magazine advertisements give an impression of status and exclusivity, even if they are not wealthy or influential themselves.

models who do not smile adequately in magazine advertisements give an impression

Page 51: Economist

models in magazine advertisements who do not smile give an impression adequately

models who do not smile in magazine advertisements adequately give an impression

it is the models, in magazine advertisements, who do not smile and adequately give an impression

in magazine advertisements, the models who do not smile adequately give an impressionThis sentence requires attention to the placement and meaning of the word adequately. In the sentence, what is adequate is the ability of the models to give an impression of high social standing. The original sentence and several incorrect answer choices misplace adequately in a way that refers to the models’ smiles.

A. The position of adequately in this option makes it sound as if the word incorrectly modifies the way the models smile.

B. Adequately is misplaced, making the sentence awkward.

C. The word adequately is correctly placed and clearly refers to the ability of the models to give the desired impression of status.

D. The phrase it is the models is wordy, as is the phrase in magazine advertisements when set off by commas.

E. In magazine advertisements is misplaced and awkward.

The correct answer is C.Although the class of anti-inflammatory drugs known as COX-2 inhibitors has proven successful at reducing pain and swelling, a series of clinical trials has shown that these drugs come with potentially harmful side effects. Anti-inflammatory drugs act on prostaglandins, hormone-like substances with a general regulatory role in the body – controlling blood flow to the kidneys or prompting contraction of the uterus during childbirth, for example – in addition to bearing the primary responsibility for causing pain and inflammation. Prostaglandins are the result of a process by which a type of acid in the cell membrane is transformed by two enzymes both known as cyclooxygenase, or COX-1 and COX-2. Older anti-inflammatory medications such as aspirin and ibuprofen stop pain by blocking the action of both COX enzymes. Researchers have long been aware, however, that the simultaneous repression of both

Page 52: Economist

COX-1 and COX-2 halts production of all prostaglandins, including those not responsible for pain.

Researchers who developed COX-2 inhibitors hoped to avoid negative side effects by blocking only one of the two COX enzymes. After a series of trials, drugs such as Vioxx and Celebrex entered the market in the 1990s. This approach, unfortunately, soon presented its own drawbacks. Although COX-2 inhibitors do in fact prevent production of prostaglandin E2 (PGE2), which is responsible for causing pain and inflammation, they also drastically reduce the amount of various other substances in the body, including prostacyclin (PGI2), which dilates blood vessels and prevents clots. On the other hand, COX-2 inhibitors do not prevent the production of thromboxane, another prostaglandin that constricts vessels and helps blood platelets clump. There is widespread agreement that the next step for researchers is to develop anti-inflammatory drugs that target specific prostaglandins rather than the enzymes that produce them

Which of the following best describes the organization of the passage?

Two options for anti-inflammatory drugs are compared.

The history of a type of medication is outlined.

The function of and problems with a type of medication are presented.

The usefulness of a popular prescription is debated.

New information is used to undermine old assumptions.In order to answer this question, analyze the structure of the passage. The first sentence of the passage introduces the topic of COX-2 inhibitors, paragraph 1 discusses their function, and paragraph 2 presents more detailed information about the problems they present. The organization of the passage is intended to discuss function and problems with COX-2 inhibitors.

A. Two options are not compared.

B. Although historical information is given, the purpose of the passage is not to discuss history.

C. Problems of COX-2 inhibitors are discussed in the first sentence and second paragraph, and general function of COX-1 inhibitors is overviewed in paragraph 1.

Page 53: Economist

D. Popular prescriptions Vioxx and Celebrex are mentioned only in the second paragraph.

E. None of the information in the passage is presented as new.

The correct answer is CAccording to the passage, a problem with COX-2 inhibitors arises for which of the following reasons?

COX-2 inhibitors  

often conflict with older types of anti-inflammatory medication.

cause the production of an excessive amount of prostaglandin E2.

do not suppress an excessive amount of thromboxane, which causes blood clots.

do not account for the complexity of prostaglandin function.

block the production of necessary regulatory substances.The phrase according to the passage in the question stem is a signal that this question can be answered using specific details from the passage. In order to answer the question, review significant lines or sentences in the paragraph to determine which is accurate. The correct answer will be a paraphrase of information stated in the passage.

A. The interaction of COX-2 inhibitors with other anti-inflammatory medications is not discussed in the passage.

B. Actually, COX-2 inhibitors successfully blockproduction of this substance. See here.

C. Though this is true, the larger problem with COX-2 inhibitors is that they do block the substance that keeps thromboxane in balance.

D. This is a distortion of details from the passage.

E. The passage states that COX-2 inhibitors block the production of PGI2, which dilates vessels and prevents clots.

The correct answer is EBased on information in the passage, which of the following can be concluded about the function of the substance thromboxane?

Thromboxane is responsible for several types of circulatory disorder.

Page 54: Economist

Prostaglandins are made in different cells than thromboxane.

Aspirin does not affect the production of thromboxane.

Thromboxane balances the effects of PGI2.

Thromboxane plays an undetermined role in regulating inflammation.An inference drawn from specific details from the passage will be necessary to answering this question. To begin, review what is stated in the passage about thromboxanein the second paragraph, as well as in sentences surrounding the thromboxane reference. The lines state that thromboxane and PGI2 have opposite effects on the cardiovascular system. It can be concluded, then, that their functions balance each other.

A. Circulatory disorders are not discussed in the passage.

B. The passage does not discuss the location of the production of substances. Additionally, the passage states that thromboxane is a type of prostaglandin.

C. Aspirin’s affect on the production of thromboxane is not discussed in the passage.

D. According to the passage, thromboxaneconstricts vessels and helps blood platelets clump, while PGI2 dilates blood vessels and prevents clots.

E. On the contrary, the passage contains information about the exact functioning of thromboxane.

The correct answer is D.It can be concluded from the passage that COX-2 inhibitors are in danger of accidentally disrupting which of the following bodily processes?

constriction of blood vessels

inflammation

cell division

kidney function

enzyme productionThe answer to this question depends on making an inference. More specifically, the question requires the connection of information in the first and third paragraphs. It is

Page 55: Economist

stated in the first paragraph that prostaglandins regulate certain healthy functions, such as blood flow to the kidneys. In the second paragraph it is stated that COX-2 inhibitors disrupt the production of various prostaglandins, which could reasonably include those that control kidney function.

A. These lines  state that COX-2 inhibitors do notblock thromboxane, which constricts blood vessels.

B. COX-2 inhibitors are anti-inflammatories intended to disrupt inflammation.

C. Cells are mentioned in the passage, but not cell division.

D. Since this line states that prostaglandins regulate blood flow to the kidneys and this line states that COX-2 inhibitors interrupt the production of various prostaglandins, it can reasonably be concluded that COX-2 inhibitors could disrupt kidney function.

E. COX-2 inhibitors deliberately interrupt the function of COX enzymes, but their productionis not discussed in the passage.

The correct answer is DFeng Shui, the ancient Chinese art of harmonious design, first became fashionable in

the United States in the 1960s, when decorators heavily influenced by New Age sensibilities began selling Feng Shui to their clients as a chance to increase health and wealth as well as to have a more beautiful home. The Chinese practice of Feng Shui

depends on directing the healthy flow of Chi, or life energy, by aligning the home itself according to the four cardinal directions and objects in the home in such a way

as to receive and maintain this positive energy. In ancient practice, the most important objects in the home include the front door, or “Mouth of Chi,” where

most life energy is received; the stove, which represents wealth and abundance; and the bed, whose positive position controls relationship energy. American popularized versions of Feng Shui, by contrast, show hardly any concern for directional alignment or key objects, preferring instead the excessive use of crystals, mirrors, and table-top

water features, which, though perhaps entertaining, display precious little respect for the real influence of Chi.

Which of the following best describes the author’s tone in the passage?

Both supportive and critical

Alternately warm and sarcastic

Wavering between disagreement and support

Page 56: Economist

Neither satisfied nor dissatisfied

Variously shocked and proudThe correct answer to this question will depend on an understanding of the author’s attitude toward Feng Shui when practiced in both China and the United States in the 1960s. In the passage, the author uses positive words such as healthy, and maintain when discussing the Chinese practice of Feng Shui, but critical words such asselling...to their clients, excessive, and precious littlewhen reviewing its applications in the United States. It is also worth noting that all of the answer choices contain two adjectives that are related to each other. The correct answer choice will identify both the author’s attitude toward different practices of Feng Shui and the fact that the attitudes are contrasted within the passage.

A. Supportive does not apply as a description of tone.

B. The author is alternately warm when discussing the Chinese practice of Feng Shui and sarcasticwhen detailing its applications in the United States in the 1960s.

C. The author’s tone does not waver.

D. This option implies that the author’s tone in the passage is neutral, when in fact traditional Feng Shui is clearly preferred to its applications in the United States.

E. Shocked does not describe the author’s tone in any part of the passage.

The correct answer is B.The author most likely uses the adjective “popularized” in order to emphasize that  

many Americans in the 1960s used Feng Shui in their homes.

directional alignment and key objects are no longer common features of Chinese Feng Shui.

decorators influenced by New Age sensibilities changed the practice of Feng Shui by adding crystals, mirrors, and water features.

the practice of Feng Shui in the United States was considerably more shallow than its practice in ancient China.

crystals, mirrors, and water features were as popular in the United States in the 1960s as Feng Shui was in ancient China.

Page 57: Economist

Understanding the author’s word choice requires grasping both the author’s overall purpose and the function of the word itself in context. The author clearly prefers the ancient Chinese practice of Feng Shui to fashionableversions of it in the United States. This opinion is developed in the first sentence, when decorators are described as selling to their clients the advantages of harmonious design. In the final sentence, the words by contrast indicate that the author views Chinese and American Feng Shui as very different. Additionally, the author describes the use of objects not common to the ancient practice of Feng Shui as excessive and says thatAmerican popularized versions of Feng Shui displayprecious little respect for Chi, the most important aspect of its traditional practice.

A. This is a distortion of a detail in the first sentence.

B. This is true only of American versions of Feng Shui.

C. The passage says that American versions makeexcessive use of these objects, but does not suggest that they were absent from Chinese Feng Shui.

D. This perspective is maintained throughout the passage; the author’s use of the wordpopularized in this sentence emphasizes the contrast between ancient practice and distorted versions fashionable in the US in the1960s.

E. Crystals, mirrors, and water features are not directly compared to Feng Shui; this option is a distortion of details.

The correct answer is D.Mark-to-market accounting, a bookkeeping technique by which estimated future revenue is counted as money in hand, can be used to make a company appear more profitable. This is especially true for large corporations in the utility and energy sectors, where multi-million dollar contracts are often signed for services that will not be delivered for several years. Corporate executives are then able to quote large annual revenue figures to stockholders, even when actual cash flow is almost nonexistent.

Based on the information above, which of the following could most properly be concluded about companies that use mark-to-market accounting?

Executives routinely exaggerate the net worth of these companies by millions of dollars a year.

Mark-to-market accounting, though dangerous, is a necessity in corporations which contract for services that will not be delivered until a later date.

Page 58: Economist

Executives of these companies are all dishonest and seek to deceive shareholders.

Information in addition to quoted annual revenue figures is needed in order to tell how profitable a company really is.

These companies will eventually collapse when the difference between reported annual revenue and cash flow has grown too great.Situation:Mark-to-market accounting can be used to make a company’s profits seem larger than its cash flow; it is especially common in large corporations in the utility and energy sectors.

Reasoning:Which conclusion can be drawn from information in the passage? Mark-to-market accounting is a bookkeeping technique that opens up the possibility that annual revenue will seem larger than actual cash flow, making a company seem more profitable than it actually is. In order to calculate that actual net worth of a company, then, information in addition to annual revenue figures, such as the company’s tax return or expenditures statements, will be necessary.

A. The passage does not suggest that executives routinely exaggerate the net worth of companies.

B. Mark-to-market accounting may be used frequently, but the passage does not state it is a necessity.

C. The words all dishonest and seek to deceive take a strong position not suggested by the passage.

D. If annual revenue figures of companies that use mark-to-market accounting can make the company seem more profitable than it actually is, then other information will be needed to determine its actual net gain.

E. This is not supported by the passage.

The correct answer is D.Making use of a study identifying how often and for what reasons the average person’s mind wanders, a government agency which studies the causes of traffic accidents has announced the creation of a series of mental exercises designed to increase concentration while driving. An agency spokesperson has stated that, once

Page 59: Economist

the exercises have been published, fatal accidents are expected to decrease by 30% in the first year alone.

The agency’s expectation rests on which of the following assumptions about the causes of most fatal traffic accidents?

Most fatal accidents involve the average person.

Most fatal accidents are reported immediately to the government agency studying them.

Most fatal accidents are caused by lack of concentration while driving.

Fatal accidents can only be prevented once the mental exercises have been published.

Fatal accidents occur with a frequency that can be predicted by the government agency.Situation:A government agency plans to prevent fatal traffic accidents through the use of mental exercises designed to increase concentration.

Reasoning:Which option identifies the assumption underlying the agency’s expectation? The first sentence of the passage states that the government agency will use data from a study on mind wandering to develop exercises that improve concentration while driving. The agency expects that fatal traffic accidents will decrease as drivers’ concentration increases. Therefore, the assumption underlying the expectation is that most fatal traffic accidents are caused by lack of attention.

A. The average person is mentioned in the first sentence as the object of a previous study, not as the cause of fatal accidents.

B. This claim is not made in the passage.

C. The government will use the results of a study on mind-wandering to help improve the concentration of drivers; if drivers’ concentration is improved, the agency expects, fatal traffic accidents will decrease.

D. This is a distortion of a detail in the second sentence.

Page 60: Economist

E. Though an expected amount of decrease in accidents is quoted, the passage does not claim that the government agency can predict the future frequency of fatal traffic accidents.

The correct answer is C.Which of the following best completes the passage below?

A recent downturn in the national economy has led retailers to expect lower sales during the upcoming holiday season. In an attempt to prevent losses during the most important quarter of the year, the owner of Suzy’s Gift Shop has decided to offer even deeper discounts than usual during the annual holiday sale, hoping that  .

the economy will actually improve during the holiday season

shoppers also suffering from the economic downturn will recognize the generous gesture and patronize the store

the amount of money spent advertising the sale can be recovered by an increase in purchases

lower prices will attract enough extra shoppers to make up for the recent downturn in the economy.

shoppers will enter the holiday spirit and spend more money in the store than they have in previous monthsSituation:The owner of a gift shop hopes to make up for a recent economic downturn by offering deeply discounted sale prices.

Reasoning:Which option best completes the passage? Because the economy has recently been slow, retailers, such as the owner of Suzy’s Gift Shop, expect sales to be lower during the important holiday shopping season. The owner of the store hopes that more drastic discounts during the annual holiday sale will prevent losses. The correct answer choice will describe a way in which lower sale prices can actually lead to greater profit: by attracting more customers.

A. The passage does not make this suggestion.

B. Whether shoppers view lower sale prices as generous does not affect the soundness of the plan.

Page 61: Economist

C. Advertising costs are not addressed in the passage.

D. If lower sale prices attract enough extra shoppers, the overall profit margin of the store will increase.

E. Though the owner does likely hope that shoppers will spend more money, the passage does not state that the holiday spirit is part of the plan.

The correct answer is D.In general, honest people are as uncomfortable misleading others as they are lying. In politics, however, things must often go unsaid or be spun a certain way in order to keep the greater peace. Therefore, it is reasonable to conclude that honest people should not get into politics.

Which of the following is most like the passage above in logical structure?

Diabetics should not become pastry chefs, because over-exposure to sugar will be harmful to their health.

People who love to read should avoid science professions, because science and the arts are mutually exclusive.

Professional basketball players should avoid playing against amateurs, because people who are less talented than they are can only weaken their skills.

Compulsive liars should avoid becoming religious leaders, because most religions stress the importance of honesty.

Those who do not believe in corporal punishment should avoid becoming parents, because children’s behavior will often make spanking necessary.Situation:Honest people should not get into politics because they are uncomfortable misleading others.

Reasoning:Which is most like the passage in logical structure? The argument might be rephrased in the following way: Honest people should avoid politics, because in politics one must lie. The logical structure of this argument depends on the assumption that all honest people must become dishonest in order to succeed in politics; conversely, if they want to remain honest, they should avoid the political situation altogether. The general assumption is that all people will act the same way in a given

Page 62: Economist

situation, rather than choosing to change the terms of their participation. The correct answer will rest on a similar assumption about the necessity of avoiding a certain situation rather than making different choices while in that situation.

A. Though this option does assume that all diabetic pastry chefs will over-consume sugar, diabetes is a health condition, not a choice on the same level as honesty.

B. The because statement in this option introduces an opinion about the relationship between two fields of study, not about decision-making in a given situation.

C. The choice in this option lies in whether or not to play against amateurs, not, as in the passage, in whether to be changed by the given situation.

D. A compulsive liar has little choice about whether to lie and so would not be free to choose not to lie.

E. This option assumes that all parents will find corporal punishment necessary and does not allow for the fact that some parents may choose to avoid spanking.

The correct answer is E.Several internet-based companies that use open-source software for their day-to-day operations have been surprised at the degree to which volunteer contributions increase productivity. At the same time, the overall profits of most of these companies have increased, since a much smaller percentage of overall revenue per working hour must be directed toward salaries. Beginning internet companies should imitate this successful model by hiring only a small core staff and leaving the rest of the tasks to volunteers.

Which of the following, if true, would most weaken the argument that a beginning internet company can best increase its profitability by relying on volunteer labor?

Internet companies can also increase profitability by paying lower salaries to more full-time employees.

Cash flow in internet companies is very different from cash flow in traditional companies, so that not all bookkeeping procedures apply.

Page 63: Economist

Internet companies often perform tasks that are highly specialized, and volunteers are not always able to complete necessary portions of projects.

The average number of hours worked by a volunteer at an internet company is already greater than the average number worked by volunteers at traditional companies.

Volunteers often hold other full-time jobs, making their actual financial contributions to internet companies difficult to calculate.Situation:In order to increase profits, beginning internet companies should hire few staff and rely on volunteers for the bulk of tasks.

Reasoning:Which most clearly shows a problem with relying on volunteer labor? The advantages of volunteer labor come in terms of both productivity and profit. If, however, the presence of volunteers can be shown to actually damage one of these goals, then the soundness of the model proposed by the plan will be seriously undermined.

A. This option does not speak directly to the question of whether volunteers are best for a beginning company.

B. Bookkeeping procedures are not mentioned in the passage; this statement is out of scope.

C. This option directly undermines the suggestion that volunteers are good for all starting internet businesses by describing a situation in which volunteers could actually undermine productivity.

D. The average number of hours worked by volunteers has no impact on the soundness of the plan. Also, traditional companies are not mentioned in the passage.

E. The passage is concerned primarily with the productivity contributions of volunteers, which translate into financial contributions. The fact that volunteers hold other jobs would have more impact on the volunteers themselves than it would on the finances of the internet company.

The correct answer is C.Used either in conjunction with or in addition to warfare and diplomacy, embargo is a foreign policy tool designed to coerce governments into taking actions the originating

Page 64: Economist

nation deems necessary or appropriate. An embargo, strictly defined, halts both imports to and exports from the target nation, but it can also prohibit only certain kinds of trade or levy heavy taxes on a nation’s goods. More a political than a military tool, embargo is a form of economic warfare that uses a forced shortage of goods – and therefore cash flow – to place internal pressure on a country’s leaders in the hope of forcing policy or regime change.

In the twentieth century, the United States government has employed embargo against Cuba since the early 1960s, when Fidel Castro brought a Communist government to power, and also more recently against certain African countries, such as Liberia and Zimbabwe, where a series of civil wars have been funded by the diamond trade. The US embargo of Cuba began in 1961, when US leaders’ discomfort with having an ally of the Soviet Union so close to the United States – Cuba is an island only 90 miles from the tip of Florida – led them to explore economic tools as a way to dismantle Castro’s regime. In Africa, by contrast, embargoes on diamonds have sought, not to undermine a particular political ideology, but rather to protect common citizens by preventing warlords and dictators from trading diamonds for guns that they use to perpetuate civil war.

Although embargo imposed from outside a country is most common, an interior embargo is also possible. Called “autarky,” from the Greek words “self” and “sustain,” an autarky is a closed economic system. The autarkic country essentially embargoes all surrounding nations by allowing trade only among its own citizens and companies. Although the United States experienced a brief period of autarky from 1807 to 1809, when President Thomas Jefferson thought it best that the newly-formed nation break all economic ties with Great Britain, such extreme isolationism has proven hard to maintain

The author of the passage is primarily concerned with

debating the merits of a foreign policy tool

undermining a common misconception

providing information about a government practice

advocating the use of a type of economic punishment

bringing attention to a little known conceptThe words primarily concerned with in the question stem indicate that the correct answer can be found by considering the passage as a whole. Paragraph 1 defines embargo, which paragraph 2 discusses instances when embargo has been used by the

Page 65: Economist

US government. The third paragraph presents an alternate view, of interior embargo. It can be concluded, then, that the goal of the passage is to provide information on the topic embargo.

A. The merits of embargo as a tool are not debated.

B. A common misconception is not discussed, much less undermined.

C. Each paragraph in the passage presents a new idea about embargo, so it can be concluded that the main goal of the passage as a whole is to provide information.

D. Economic punishment is not discussed, and the passage does not have a tone of advocacy.

E. The passage does not state that embargo is alittle known concept.

The correct answer is CWhich of the following best describes the relationship of the third paragraph to the passage as a whole?

A new idea and contrary evidence are introduced.

Exceptions to the rules of economic embargo are discussed.

The main idea of the passage is refuted using historical evidence.

A third perspective on the coercive nature of embargo is outlined.

The main idea of the passage is further discussed from a different perspective.In the passage, each paragraph offers new information about the main topic embargo. Paragraph 1 provides a definition, paragraph 2 gives examples of use, and paragraph 3 continues discussion of the main topic from a different perspective.

A. Details introduced in paragraph 3 are meant to provide further information, not contrary evidence.

B. The passage discusses neither the rules of embargo nor any exceptions to those rules.

C. The main goal of the passage is to provide information on embargo, and, as such, it can not be refuted by further historical evidence.

Page 66: Economist

D. The coercive nature of embargo is not the main idea of the passage, and so the final paragraph can not present a third perspective.

E. The main goal of the passage is to provide information about embargo, and the third paragraph continues the discussion with details about a type of interior embargo.

The correct answer is E.The author mentions EACH of the following as a goal of embargo EXCEPT

to wage economic warfare

to punish a country’s leadership for inappropriate policies

to break economic ties with a parent nation

to force a shortage of goods and cash flow

to protect common citizens from violenceThis question requires the selection of the detail notmentioned in the passage. In order to select the correct answer choice, test each one and look for a line in the passage that supports it.

A. In this line, embargo is mentioned as a form of economic warfare.

B. The passage does not discuss embargo as apunishment; this option is a distortion of ideas in the first paragraph.

C. This line  discusses the 1807-09 period of autarky in the US as an attempt to break economic ties with Great Britain.

D. Shortage of goods and restriction of cash flow are mentioned in these lines.

E. Embargoes against certain African countries are intended to protect citizens, as discussed in these lines.

The correct answer is BThe author would most likely agree with which of the following statements about the United States government’s embargo of Cuba?

The embargo of Cuba  .

was begun with the intention of undermining a political ideology

Page 67: Economist

is easier to enforce than embargoes of African countries, since Cuba is closer

has forced Cuba to become an autarky with a closed economic system

was a tactic of economic warfare used against the Soviet Union

has put significant economic pressure on Cuba’s leadersThe answer to this question depends on making an inference. First, review paragraph 2, where the US government’s reasons for initiating embargoes are discussed. This line indicates that an embargo against Cuba began when a Communist government came into power, and these lines state that a goal of the US embargo was to weaken the Communist leader, Castro. In the following line, the passage states that the US did not seek to undermine a political ideology in Africa, implying that it did have such a goal in Cuba. The wordsby contrast further emphasize this idea.

A. These lines , taken together with the first sentence of the paragraph, imply that undermining Castro’s Communist ideology was a main goal of the embargo of Cuba.

B. This is a distortion of a detail found here.

C. The passage does not state that Cuba has cut off all imports and exports from the interior, which would make it an autarky.

D. This is a distortion of a detail, which states only that Cuba was an ally of the Soviet Union.

E. This is the goal of embargo, but the passage does not state whether or not it was successful.

The correct answer is A.Since the majority of customers are not complaining when they are dissatisfied, even those organizations most concerned with customer service have only a single opportunity to establish brand loyalty.

Since the majority of customers are not complaining

Since the majority of customers have not complained

Since the majority of customers do not complain

Page 68: Economist

When the majority of customers do not complain

In the event that the majority of customers are not complainingCorrect verb form and use of the word since to indicate a causal relationship are being tested in this sentence. Because the sentence refers to events occurring in present time, the use of simple present tense and the verb form do not complain are required. Additionally, the parts of the sentence are logically connected in a causal relationship: because most customers do not complain, companies do not have a second chance to establish brand loyalty. This logical relationship requires the use of the word since.

A. Are not complaining is present progressive tense and is not correct in the sentence.

B. Have not complained incorrectly uses past perfect tense.

C. This option correctly uses do not complain in present tense and the word since to indicate a logical causal relationship between parts of the sentence.

D. The word when introduces an unnecessary chronological element into the sentence.

E. The phrase in the event that is wordy and does not express the necessary logical relationship among parts of the sentence.

The correct answer is C.If one believes what Sigmund Freud writes about forgetting, being unable to remember a name is an act of willful repression, like amnesia, in which one is unaware of having forgotten.

like

whereas

similar to

unlike

apart from

Page 69: Economist

This sentence contrasts an act of willful repression with a condition in which one is unaware of having forgotten. The contrast should be clear in the middle of the sentence, where amnesia is mentioned as an example of the second condition. Unlike correctly expresses this contrast.

A. Like does not suggest a contrast; its comparison of the two conditions is illogical here.

B. Whereas expresses contrast, but inserted into the underlined portion, it makes the sentence incomplete.

C. Similar to does not suggest needed contrast.

D. Unlike is correct in this sentence because it emphasizes the contrast between willful repression and amnesia.

E. Apart from does not make sense in this sentence.

The correct answer is D.Several internet-based companies that use open-source software for their day-to-day operations have been surprised at the degree to which volunteer contributions increase productivity. At the same time, the overall profits of most of these companies have increased, since a much smaller percentage of overall revenue per working hour must be directed toward salaries. Beginning internet companies should imitate this successful model by hiring only a small core staff and leaving the rest of the tasks to volunteers.

The argument above that internet companies will increase profits by hiring fewer full-time employees is based on which of the following assumptions?

Internet companies that use open-source software are better suited to function efficiently with only a small core staff than traditional companies.

The amount of administrative work to be performed in internet companies is small enough that it can reasonably be completed by fewer full-time employees.

Full-time employees and volunteers will have different levels of access to most projects undertaken by the company.

Page 70: Economist

Internet companies will hire more full-time employees as they become more profitable due to volunteer contributions.

Companies that rely on both full-time employees and volunteer labor increase their overall revenue per working hour.Situation:In order to increase profits, beginning internet companies should hire few staff and rely on volunteers for the bulk of tasks.

Reasoning:Which is the assumption underlying the conclusion that fewer full-time employees will result in greater profits? The model presented in the passage imagines that all work necessary to success can be performed when it is divided between full-time staff and volunteers. However, the total amount of work must necessarily include administrative tasks to be performed by full-time employees. The passage assumes, therefore, that the amount of administrative work to be performed can reasonably be accomplished by a small number of core staff.

A. Internet companies and traditional companies are not directly compared in the passage.

B. The passage assumes that a small number of full-time employees can successfully perform all needed administrative tasks.

C. Outside the scope. The passage does not mention levels of access.

D. Plans for hiring more full-time employees are not discussed in the passage.

E. This is not an assumption underlying the conclusion

The correct answer is B.A study attempting to link fertility with increased sexual availability concluded that women unconsciously dress more attractively during ovulation. The study was conducted using pairs of photographs of 200 women and volunteer viewers who chose the photograph in which they thought the woman appeared more attractive. Researchers found that 67% of the time viewers chose the photo taken when the woman was ovulating.

The methodology of the study above is flawed for which of the following reasons?

The study fails to specify what is meant by the word attractive, leaving it up to the opinion of the individual viewer.

Page 71: Economist

Because all the viewers were male, the results of the study were skewed by individual responses to attractive women.

The study fails to account for the psychological complexity of sexual desire in women

The study assumes that women will dress more attractively when they have greater sexual desire.

Most women do not know when they are ovulating, and so would not dress more attractively on that day.Situation:A study has concluded that women dress more attractively when they are most fertile. Reasoning:Which points to the most serious flaw in the study’s methodology? Researchers approached the question of the link between dress and fertility using pairs of photographs of 200 women. One photograph was taken while the woman was ovulating, and one was taken when she was not. Researchers then showed these photographs to volunteer viewers, who decided in which photograph they thought the woman was more attractive. However, because the study did not define what should constitute “attractive,” the potential margin of error among the opinions of viewers is too great for the study’s conclusions to be beyond question. In order for the photographic approach to work, researchers would need to give criteria for “attractive dress,” such as shorter skirts, low-cut blouses, high heels, etc.

A. Because the study does not specify what is meant by attractive dress, individual viewers may simply choose the photograph in which they find the woman more attractive for reasons that have nothing to do with the study.

B. The passage does not state that all viewers were male.

C. Though this may be true, an understanding of psychological complexity is not the goal of the study.

D. Because this is the premise being tested, the assumption is necessary to the study.

E. The passage states explicitly that women unconsciously dress more attractively during ovulation, and so deliberately dressing more attractively is not an issue.

The correct answer is A.

Page 72: Economist

During the Great Depression, Roosevelt’s New Deal expanded federal authority by creating several new government agencies designed to provide and administer relief to the country, which had been devastated by the 1929 stock market crash. Many agencies created under the New Deal were discontinued in subsequent decades, however, when policymakers grew uncomfortable with the amount of power wielded by the federal government. Additionally, a large number of economists at the time felt the market had recovered to the point that federal regulation had become more a hindrance to than a provider of economic stability.

The economists’ support of the discontinuation of New Deal programs rests on which of the following assumptions about the role of the federal government in the market?

Interference by federal government in the market can never create economic stability.

Federal regulation of the market is an emergency measure and, as such, should be temporary.

Agencies created under the New Deal routinely exercised control beyond what was stated in their charters.

Policymakers who discontinued New Deal programs were not the same policymakers who originally implemented them.

New Deal programs designed to provide economic relief actually perpetuated market instability.Situation:Policy makers and economists in the decades following the Great Depression agreed that many New Deal agencies should be discontinued because expansive federal regulation and power were no longer necessary to market stability.

Reasoning:Which identifies the assumption on which the economists’ support rests? According to the passage, the New Deal represented an expansion of federal authority in response to the crisis of the 1929 stock market crash. In subsequent decades, however, after the Depression had ended, policymakers had room to be uncomfortable with the amount of federal control by New Deal agencies because the state of emergency has passed. This idea is supported in the third sentence, which states that economists felt the market had recovered from the crash, and that it had

Page 73: Economist

reached a point at whichfederal regulation had become...a hindrance, even though it had been necessary earlier.

A. According to the passage, the economists thought that federal regulation had become...a hindrance to economic stability, not that it must always be one.

B. The economists’ support of discontinuing New Deal agencies rests on a belief that federal regulation of the market had outlived its usefulness; the underlying assumption in the third sentence is that market regulation had previously served a necessary function.

C. Neither New Deal agency charters nor the actual amount of control they exercised is discussed in the passage.

D. This is not stated in the passage.

E. On the contrary, the economists’ assumption was that federal regulation had once been useful but had become...a hindrance of economic stability.

The correct answer is B.In an experiment involving taste, researchers had participants try two versions of a cola, one regular and another, called “cola extra,” to which a secret ingredient had been added. The researchers found that, when the tasters did not know that the secret ingredient was white vinegar, they preferred “cola extra” by a margin of almost two to one. However, when tasters were told the identity of the secret ingredient before drinking, they preferred the cola without vinegar 98% of the time.

Which of the following, if true, forms the best basis for an explanation of this outcome?

Most consumers are excited by the possibility of tasting a “secret ingredient” and will always prefer that choice.

For most consumers, expectation strongly influences the ability to accurately distinguish flavors.

Most cola drinkers prefer beverages without vinegar, even when they do not know vinegar is present.

Page 74: Economist

Researchers skewed results by telling participants in the second group what the secret ingredient was.

The first group of taste testers was disproportionately composed of people who enjoy the taste of vinegar.Situation:In a taste test, participants who did not know one of the colas contained vinegar preferred the cola with vinegar. When participants did know there was vinegar in one of the colas, they preferred the cola without vinegar.

Reasoning:Which option best explains the outcome of the experiment? The experiment described in the passage is performed with two groups of tasters. Both groups taste both regular cola and “cola extra,” which contains a secret ingredient. The only difference between the two groups is that the first group does not know before drinking that the secret ingredient is white vinegar. The second group, however, knows about the vinegar before drinking. Since the first group prefers “cola extra” and the second prefers regular cola, it is reasonable to assume that each group’s expectations affected their experience of the taste of each cola.

A. This is a partial explanation for the responses of the first group, but does not address the study as a whole.

B. For both groups of tasters, expectation of either a positive or a negative taste changed their perceptions of each cola. The first group preferred “cola extra” because they expected to prefer the secret ingredient, and the second group preferred regular cola because they expected vinegar to have an unpleasant taste.

C. This option fails to explain why the first group preferred cola extra.

D. Knowledge of the secret ingredient was a key component in the researchers’ approach and did not skew results.

E. This claim is not made in the passage.

The correct answer is B.Two new commentaries on the life and work of early twentieth century short story writer Katherine Mansfield may be aimed at completely different audiences, but each uses well-known facts in new ways. The first, a popular biography by Virginia Smith, nods at Mansfield’s origins at the edge of the British empire – she was born in 1888 in Wellington, New Zealand – but is ultimately much more interested in the ways

Page 75: Economist

Mansfield’s simultaneously defiant and needy personality made her one of the most important, though often overlooked, groundbreakers of literary modernism. Mansfield’s relationships with fellow writers D.H. Lawrence and Virginia Woolf, as well as with her critic-husband John Middleton Murray, are examined for evidence of Mansfield’s influence on them, rather than vice versa. Ms. Smith compellingly presents Mansfield as a social chameleon skilled at imitation and adaptation; this innate flexibility, the book argues, is the very trait that made only Mansfield capable of infusing the English literary scene with the influence of Russian writer Anton Chekhov. Though in the end Ms. Smith offers few new biographical details, the truly impressive aspect of this biography is her ability to shift easily between worn fact and compelling narrative.

Whereas the few shortcomings of Ms. Smith’s book may be attributed to lack of publishing experience, a second commentary from long-time professor Jim Jeffries can make no such excuse. Mr. Jeffries’ contribution is an often tedious biographical essay that introduces a new critical edition of Mansfield’s short stories. Jeffries’ work plods point-by-point along Mansfield’s biography, attempting to attribute the inspiration for each sparkling, artfully constructed story to a traumatic event in her life. The result is not so much a portrait of Mansfield’s work as it is an ornate yet overly-simplified timeline. This offense is only compounded by Mr. Jeffries’ shallow interpretations of Mansfield’s most subtle and complex symbolism, and his continuing references to various critics and philosophers make the essay no more interesting than name-dropping at a cocktail part

The passage above is primarily concerned with

establishing an unknown writer as an important literary figure.

comparing and contrasting two recent biographical works.

arguing for a new interpretation of the life of a literary figure.

disputing the credentials of a well-known literary critic.

analyzing different uses of commonly-known biographical facts.The correct answer choice depends on an understanding of the passage as a whole and must take all parts of the passage into account. The first sentence previews the rest of the passage by stating that the topic is two new commentaries about Katherine Mansfield. Though the author clearly prefers one commentary over the other, thegeneral purpose of the passage is to compare and contrast the two works.

A. It is not claimed in the passage that Mansfield isan unknown writer.

Page 76: Economist

B. Virginia Smith’s popular biography is compared and contrasted with Jim Jeffries’ introduction to a critical edition of short stories.

C. Though the authors of both works being reviewed reinterpret Mansfield’s life, this is not a goal of the author of the passage.

D. A well-known literary critic is mentioned only in paragraph 2, and his credentials are not disputed.

E. This is one aspect of the author’s review of each biographical work, but it is not the main goal of the passage.

The correct answer is B.Which of the following forms the best conclusion to the second paragraph?

However, Mr. Jeffries does manage to say considerably more about Mansfield’s relationship with Virginia Woolf than does Ms. Smith.

In light of these facts, one can predict that Mr. Jeffries’ book will be far less popular than Ms. Smith’s.

In short, though Mr. Jeffries had at his disposal the same biographical information as Ms. Smith, he manages to do considerably worse with it.

Because of the many shortcomings of Mr. Jeffries’ book, it is likely that Ms. Smith will soon eclipse him as the predominant scholarly authority on Mansfield.

Notwithstanding these many shortcomings, Mr. Jeffries has his long-standing critical reputation to recommend him to readers.Because this question does not deal directly with information from the passage, it can best by answered by making an inference. A review of the second paragraph reminds us that the author of the passage dislikes Jeffries’ work on Mansfield because it is both over-simplified and arrogant. An apt conclusion to the second paragraph will maintain the critical tone of the paragraph and summarize the author’s reasons for dislike.

A. The positive tone of this option does not fit with the critical tone of the rest of the paragraph.

B. Popularity of either book is outside the scope of both the second paragraph and the passage.

Page 77: Economist

C. This option maintains the critical tone of the second paragraph, summarizes the criticism, and refers back to the first sentence of the passage, in which the thread common to both works – biographical information – was discussed.

D. The second paragraph does not speculate on the relative authority of Smith and Jeffries.

E. A compliment to Jeffries’ critical reputationdoes not fit with the critical tone of the second paragraph.

The correct answer is CWith which of the following statements about Mansfield’s relationships with other modernist writers would the author of the passage most likely agree?

Fellow writers such as Lawrence and Woolf learned imitation and adaptation from Mansfield.

The fact that Mansfield was both defiant and needy made her relationships with other writers difficult.

Mansfield preferred her friendship with Chekhov to relationships with English writers Lawrence and Woolf.

Mansfield’s personality was flexible enough to accommodate relationships with critics as well as with fellow writers.

Mansfield’s influence on fellow writers, though often overlooked, is as significant as their influence on her.The phrase most likely agree in the question stem indicates that the correct answer depends on making an inference. First, review these lines, where Mansfield’s relationships with fellow writers are discussed. In this sentence, the author italicizes the word them to emphasize that relationships of influence among modernist writers are assumed to run the other way, with writers such as Woolf and Lawrence impacting Mansfield. The phrase rather than vice versa at the end of the sentence further emphasizes this assumption and reveals the writer’s opinion that Mansfield’s influence on fellow writers was at least as important as their influence on her.

A. Imitation and adaptation are mentioned as traits possessed by Mansfield, not as traits learned by fellow writers.

B. This is a distortion of a detail from this line.

C. The passage does not state that Mansfield had a friendship with Chekhov.

Page 78: Economist

D. This is a distortion of details from this line, which mentions that Mansfield’s husband was a literary critic, and this line, which applauds the flexibilityof her imagination.

E. This option is a paraphrase of these lines, which make it clear that the author considers Mansfield’s influence on fellow writers to be of often underestimated importance.

The correct answer is EThe author most likely uses the word “plods” in this line in order to

emphasize the tedious nature of the essay.

criticize the essay’s excessive use of detail.

highlight the essay’s method of connecting biography to literary output.

mark a distinction between narrative and factual elements in the essay.

draw attention to a preferred approach to biography.The question requires a consideration of the author’s word choice. The previous two sentences begin a general criticism of Jeffries’ essay that includes the accusation that it is tedious. The next sentence continues this idea, providing more details and further criticizing Jeffries’ work for heavy use of a timeline approach.

A. The word plods underscores the claim in the previous sentence that Jeffries’ essay is tedious, and prepares the rest of the sentence to argue more specifically that the timeline approach is particularly offensive to the author.

B. The second paragraph does criticize the essay’s excessive use of detail, but the word plods in this sentence has a more specific function.

C. Plods does not refer directly to the essay’s method of connecting biography and output.

D. The word plods does not function this way in the sentence.

E. The author does prefer a certain approach to biography, but that approach is described in the first paragraph, not the second.

The correct answer is ARust, the name applied to the oxidation of iron and its alloys, including stainless steel and wrought iron, is the result of an exothermic reaction, or a reaction that produces heat.

Page 79: Economist

and its alloys, including stainless steel and wrought iron, is

alloys such as stainless steel and wrought iron is

and alloys including stainless steel and wrought iron, are

alloys, including those of stainless steel and wrought iron, is

and its alloys, including stainless steel and wrought iron, areThis correct sentence uses commas to set off the two names of specific iron alloys and uses is as the sentence’s key verb, agreeing with the subject rust.

A. This option correctly sets of stainless steel and wrought iron as two examples of iron alloys and also illustrates subject-verb agreement with the singular subject rust.

B. Without commas to separate the examples of iron alloys, this sentence is incomplete.

C. The plural verb are does not agree with the singular subject rust.

D. The word of in the middle of the phrase is unnecessary; also, there is no reason to drop the word its from the original sentence.

E. The singular subject rust requires a singular verb is.

The correct answer is A.As part of a protest against what he considered unlawful military action, the performance artist dressed in complete British Army camouflage fatigues, with commando boots wearing a red beret, and lay down in a street in the city center.

fatigues, with commando boots wearing a red beret

fatigues, with commando boots and a red beret

fatigues and commando boots, with a red beret

fatigues and commando boots and a red beret

fatigues, commando boots, a red beret

Page 80: Economist

As written, the original passage makes it sound as if the artist’s boots are wearing the red beret, rather than the artist himself. The correct answer choice will clear up this confusion and list the items the artist was wearing as part of his complete uniform in correct parallel form.

A. This option is confusing, making it sound as if the commando boots are wearing the beret.

B. This option correctly identifies the fact that commando boots and a red beret are part of the complete uniform of a soldier, and expresses each part in parallel form.

C. Commando boots and red beret should be grouped together as elements of a complete uniform.

D. Though this option is grammatically correct, the list of items does not express the idea of a complete uniform called for by the sentence.

E. This option makes the sentence incomplete.

The correct answer is B.Corporate Financial Officer: The proposal for our firm’s new office building is unnecessarily extravagant, overly spacious, and ridiculously expensive. The absolute maximum budget for this project is $9 million.

Building Contractor: Our architects have redrawn the blueprints to remove the atrium and sculpture garden, and our landscape artists have reduced the complexity of the surrounding gardens. We will now be able to complete the project at one-tenth the original estimated cost.

Which of the following, if true, most undermines the soundness of the contractor’s proposal to complete the project at a fraction of the original cost?

The original estimated cost of the project was $100 million.

The contracting firm was originally hired based on the originality of its atrium design.

Corporate officers have already paid a deposit of $2 million.

Page 81: Economist

The firm expects to hire 100 new employees by the time the building project is completed.

Architects working on the revised plans did not discard all expensive elements.Situation:A building contractor offers revised blueprints to a client who claims the original plans were too expensive.

Reasoning:Which is the most serious flaw in the contractor’s proposal? The main concern of the financial officer is that the plans are larger, and therefore more expensive, than his firm can afford. Because the financial officer sets the maximum budget at $9 million, any evidence that the contractor’s new plans will be more expensive than this figure will greatly undermine the soundness of his proposal.

A. If the original cost of the project was $100 million, then one-tenth of this cost will be $10 million, which is $1 million more than the company can afford.

B. The elimination of the atrium does not undermine the soundness of the contractor’s plan, no matter if that design was originally appealing to the corporate officers.

C. The amount of the deposit has no bearing on the final total cost.

D. This option is tempting because it makes it seem as if the firm will need a larger building, but it actually has no bearing on the soundness of the contractor’s plan.

E. The corporate officer did not ask that all expensive elements be discarded, but only that the project cost no more than $9 million.

The correct answer is A.The author of a book on the women of the Temperance Movement suggests that the single trait common to the most influential members was a vision of her public work like an extension of the safe-guarding of moral integrity in the private household, which was considered the responsibility of wives and mothers.

her public work like an extension of the safe-guarding

their public work as an extension of the safe-guarding

Page 82: Economist

public work for her as an extension of the safe-guarding

their public work like an extension of the safe-guarding

her public work as an extension of the safe-guardingBecause the author of the passage is discussing a vision common to a group of women, the pronoun leading the correct answer choice will be the pluraltheir. As is needed to refer to the way women understood their work; like in this sentence violates the correct idiomatic construction.

A. The singular her does not meet the requirements of the sentence, and the word like is not idiomatically correct.

B. The pronoun their correctly reflects the antecedent the most influential members, and as is idiomatically correct in the phrase as an extension.

C. For her is misplaced, disturbing the logic of the sentence.

D. The word like is an idiomatically incorrect way to refer to the way the women understood their work.

E. Her, a singular pronoun, does not properly reflect the plural antecedent most influential members.

The correct answer is BAn annual survey ranked City B as the sixth most dangerous city of 371 surveyed, using uniform crime statistics records kept by the FBI. City B’s mayor disputed the report’s conclusions, noting both the recent reduction in the city’s crime rate and the FBI’s cautions against using uniform statistics to make comparisons between metropolitan areas.

Which of the following, if true, represents the best piece of evidence to be used in support of the mayor’s objection?

According to the same survey the previous year, City B was ranked as only the tenth most dangerous.

The survey examined statistics for entire counties, but ranked the most dangerous according to the name of the most populous city.

Page 83: Economist

The mayor is planning to run for re-election and has personal reasons for wanting City B to seem less dangerous.

Statistics demonstrate that metropolitan areas are more dangerous than rural areas.

City B’s crime statistics are reported to the FBI on a quarterly, rather than an annual, basis.Situation:The mayor of a city recently ranked sixth most dangerous has disputed the survey’s methodology.

Reasoning:Which is the best piece of evidence to use in support of the mayor’s objection? The survey’s conclusions are based on the use of FBI uniform crime statistics, about which the FBI itself has issued cautions about using to compare metropolitan areas. The correct answer will support the mayor’s objection by giving evidence that the use of FBI crime statistics may have skewed the survey’s results to make City B look more dangerous than it actually is.

A. A change in City B’s ranking is not the most effective piece of evidence to support the mayor’s objection.

B. If the survey ranked City B on the basis of crime statistics for its entire county, and crime statistics in the general metropolitan area are higher than in the city limits, then it is possible that the survey exaggerates the danger present in City B.

C. Neither the mayor’s re-election campaign nor his reasons for running are evidence to support his objection.

D. This may be true, but it is beside the point.

E. The frequency of the reporting of the statistics has not direct bearing on the finding of an annual report, and so can not be used as evidence to support the mayor’s objection.

The correct answer is BFlyer: Appleseed Books is pleased to announce that our Summer Bookclub and Signing Series will feature six of the best new novels by local authors. Club registration fee includes all books, refreshments at meetings, and admission to each signing event.

Page 84: Economist

Based on the information above, which of the following must be true?

Summer Bookclub will be popular among fans of local authors.

Most local authors write novels rather than poetry.

Book signings will not be held during regular meetings.

Only Bookclub members will be admitted to signing events.

It must be decided which new novels are the best.Situation:A bookstore announces the components of its summer book club.

Reasoning:Which option must be true? In order to answer this question correctly, it is necessary to distinguish between information that might or couldbe true, and information stated directly in the passage. Since the flyer claims the club will read the best new novels, it is reasonable to conclude that someone has decided which novels qualify as the best.

A. This may or may not be the case.

B. Genres in which local authors work are not discussed in the passage.

C. Though the passage lists book signings and meetings as separate items in a series, it does not state that they will be held at different times.

D. The passage does not make this claim.

E. If the book club will be reading six of the best new novels, it is reasonable to conclude that some member of the bookstore or reading series staff has made a decision as to what constitutes the best.

The correct answer is E.Whereas pure science is mainly concerned with the discovery of new truths and makes few, if any, attempts to discern the best options between their potential uses, applied science involves the application of existing truths to concrete problems.

best options between their potential uses

best options between its potential uses

Page 85: Economist

best of the options among their potential uses

best options among its potential uses

best options among their potential usesThis sentence tests the correct use of the word among to refer to more than two items, as well as the ability to identify the antecedent of the pronoun their. The potential uses discussed in the sentence are uses for new truths; the pronoun their is correct.

A. Between refers to two objects and is used incorrectly in this sentence.

B. Its incorrectly suggests a singular antecedent, and between is used incorrectly.

C. The word of is wordy and unnecessary.

D. The pronoun its is misused because it suggests a singular antecedent.

E. Both among and their correctly refer to the plural new truths discovered by pure science.

The correct answer is E.

Gmat 3

A survey of charitable giving in the state found that the average dollar amount contributed annually by residents of urban areas to programs for the homeless was $15 greater than the amount contributed by residents of rural areas. The survey’s creators, an urban-promotion group known as Live in the City, concluded that city dwellers are on average more generous than residents of rural areas.

EACH of the following, if true, casts doubt on the conclusion of the survey’s creators EXCEPT

An unrelated survey of annual charitable giving finds that residents of rural areas give 3% less to charity than residents of urban areas.

There are more homeless people in urban areas than rural areas, making it more likely that urban residents would contribute to those charities.

There are more charities in general operating in urban areas than in rural areas.

Page 86: Economist

The survey is calculated based on dollar amount rather than percentage of income, and does not account for the fact that incomes in urban areas are often higher.

The group Live in the City has been known to alter survey results for marketing purposes.Situation:A group that promotes urban living has published survey results that conclude city dwellers give more money to charity than residents of rural areas.

Resoning:Which option supports the conclusion of the survey’s creators?The question stem informs us that all the answer choices except one will undermine the survey’s conclusion. The correct answer will present independent confirmation of the survey’s finding that city dwellers are more generous than residents of rural areas.

A. Though phrased differently, this option presents the same conclusion as that reached by the Live in the City survey – that residents of rural areas statistically give less than residents of urban areas. Both surveys are concerned with annual giving, and the 3% in this option echoes the average dollar amount of the original survey.

B. This statement undermines the original survey’s conclusion by giving a reason that city dwellers give more money to charities for the homeless: there are more homeless in cities.

C. This statement undermines the original survey’s conclusion by making it clear that there are more opportunities in general for charitable giving in cities than in rural areas, making it likely that the annual dollar amount given would be greater in urban areas.

D. This statement undermines the original survey’s conclusion by pointing out that the conclusion does not account for a difference in average incomes in the two areas.

E. This statement undermines the original survey’s conclusion by casting doubt on Live in the City’s integrity.

The correct answer is A.Unfortunately for the commuters who lost their lives when the bridge collapsed, the engineer was not honest   at the preliminary public safety commission hearing, when he was questioned about the structure’s weight-bearing abilities.

Page 87: Economist

was not honest

was not

was honest

had not been honest

had not been being honestThis sentence requires past perfect verb form to indicate that the engineer’s dishonesty occurred at a definite point in the past. The point in the past is indicated by the phrase at the preliminary safety hearing.

A. Was not honest is past tense, not past perfect, and does not sufficiently indicate the order of events discussed in the sentence.

B. The word honest is missing from this answer choice, changing the meaning of the sentence.

C. This answer choice is the opposite meaning of the underlined portion.

D. The words had not been honest correctly indicate past perfect tense and demonstrate that the engineer’s dishonesty occurred at a definite point in the past.

E. The word being is redundant and unnecessary.

The correct answer is D.The artist’s most recent work, a series of canvasses splashed liberally with acrylic paint in primary colors, is extraordinarily popular with interior designers specializing in hotels.

primary colors, is

primary colors is

primary colors, are

primary colors are

Page 88: Economist

colors, are primarilyThe sentence is testing the ability to connect subjects and verbs separated by parenthetical phrases. In this case, the singular subject work requires a singular verb is, and the parenthetical phrase must be set off by commas.

A. The verb is agrees with the singular subject work, and the entire parenthetical phrase is set off by commas.

B. The parenthetical phrase is not correctly set off by a comma after colors.

C. The verb are is plural and does not agree with the singular subject work.

D. Are is a plural verb that does not agree with the subject work, and there is no comma after colors separating the parenthetical phrase.

E. This phrase uses commas correctly, but the verb are does not agree with the subject work, and the sense of the sentence has been changed.

The correct answer is A.Static electricity results when negatively-charged electrons are discharged, in the absence of a conductor, toward a positively-charged substance or material. Lightning is a powerful kind of static electricity created when electrons build up on the bottom of a rain cloud. Since like charges repel, electrons below are driven away, leaving the ground with a positive charge. When the charges in the cloud and the ground have gained sufficient strength, a bolt of lightning is generated.

If the statements above are all true, which of the following can properly be inferred on the basis of them?

The positive charge in the cloud and the negative charge on the ground must be equal in order for lightning to be generated.

Since scientists understand the movement of positive and negative charges, they can use lightning technology to generate electricity for consumer use.

A lightning bolt is generated when electrons from the cloud are discharged toward the positively-charged ground.

Lightning bolts are more likely to occur when it is raining, since water can act as a conductor.

Page 89: Economist

Trees are more likely to be struck by lightning than open ground, since they naturally tend toward a greater positive charge.Situation: The passage describes how lightning, as a type of static electricity, is generated by the interaction of positive and negative charges in the atmosphere.

Reasoning:What inference can be drawn from this information? According to the first sentence, static electricity results from the movement of negatively-charged (-) electrons toward a positively-charged (+) substance or material. Since this movement of charges is the basic mechanism at work in static electricity, lightning as a form of static electricity could be expected to behave the same way. In this case, the movement is from a negatively-charged cloud (-) toward the positively-charged (+) ground.

A. The charges here are opposite to the conditions that generate a bolt of lightning: the cloud has a negative charge and the ground has a positivecharge. Also, the passage does not mention whether the charges should be equal.

B. The passage does not mention potential uses for lightning technology.

C. The passage describes lightning as a type of static electricity, which would mean that, when lightning occurs, negative charges (electrons) are moving from a cloud toward the positively-charged ground. This answer choice correctly describes the process.

D. The passage does not discuss whether lightning is more likely to occur in the presence of rain. Also, the passage explicitly states that static electricity functions without a conductor.

E. The passage does not discuss whether trees or the ground or more likely to be struck, nor does it mention the positive or negative charge of trees.

The correct answer is C.The reason the factory did not meet its deadline is because a snowstorm in the northeast delayed delivery of the necessary components by more than a week.

The reason the factory did not meet its deadline is

The factory was not able to be meeting the deadline

The factory did not meet its deadline

Page 90: Economist

The deadline of the factory was not met

The deadline, of the factory, was not metIn this sentence, the underlined phrase is wordy, and the phrases reason and because are redundant. The correct answer choice will express the main idea of the sentence succinctly

A. Reason and is because make the sentence awkward and redundant.

B. The verbal phrase to be meeting is awkward.

C. The redundant phrase reason has been eliminated, and the resulting phrase is concise and direct.

D. The phrase the deadline of the factory is passive voice and incorrect.

E. This option uses passive voice and the phrase of the factory does not need to be separated by commas.

The correct answer is C.The new mayor is certainly more progressive than her predecessor, but she is not nearly as efficient.

but she is not nearly as efficient

but she, however, is not nearly as efficient

but she is nearly not as efficient

even though she is not nearly as efficient

despite being not as efficientThis sentence is correct and concise. She clearly refers to the new mayor, and the statement of her efficiency contains no unnecessary words.

A. The relationship between the subjects of clauses in this sentence is clear; the second clause is concise.

B. However is unnecessary.

C. Nearly is misplaced, making the clause awkward.

Page 91: Economist

D. The phrase even though implies an incorrect logical relationship between the new mayor’s progressive stance and her efficiency.

E. Despite implies an incorrect logical relationship between the mayor’s progressive stance and her efficiency.

The correct answer is AThe author of a new art history book argues that students whose paintings come to be received as masterpieces achieve greatness by perfecting the technique and vision of their teachers. To support this theory, the author provides a detailed comparison of the works of 15 student-teacher pairs, in each case demonstrating that the teacher’s work is superseded by the student’s.

Which of the following, if true, is most damaging to the conclusion reached by the author of the art history book?

The book does not include any side-by-side illustrations comparing the work of students to the work of their teachers.

The author does not consider the fact that, in student-teacher pairs, both artists can produce either masterpieces or low-quality works.

The author does not give sufficient attention to the historical context of each masterpiece produced.

The author chose for inclusion only those artists whom he considered to have surpassed their teachers, and then examined the relationship of their respective paintings.

The author concentrates almost exclusively on the techniques developed by the teachers, and does not give sufficient attention to the execution of technique by the students.Situation:The author of an art history book claims that artists achieve greatness by surpassing the skills of their teachers. The author cites pairs of students and teachers as proof of this conclusion.

Reasoning:Which option is most damaging to the author’s conclusion? In order for this conclusion to be valid, the author should first clarify his definition of “masterpiece” and then proceed to examine which painters of qualifying masterpieces

Page 92: Economist

first followed and then surpassed the skills of their teachers. Given the reasoning outlined here, however, it is reasonable to expect that the author selected from among student-teacher pairs only those pairs that fit the stated conclusion, choosing to examine only those pairs in which the student produced a vaguely-defined “masterpiece.”

A. While illustrations might be a serious omission, the main weakness of the argument is the author’s ability to choose only those pairs which fit the conclusion.

B. This option does not directly address the author’s concern with masterpieces produced by students.

C. Historical context is not mentioned in the passage.

D. The choice of student-teacher pairs is made specifically to support the author’s pre-formed conclusion, obscuring any real test of the theory of art students surpassing their teachers.

E. The author of the passage does not discuss technique in detail.

The correct answer is D.Apparently tipped off, the police were not able to capture the suspect, who was fleeing the house before they arrived.

the police were not able to capture the suspect, who was fleeing the house before they arrived

the police, have not been able to capture the suspect, who fled as they arrived

the suspect fled the police, who are not able to capture him at the house

the suspect arrived before the police were fleeing the house

the suspect fled the house before the police arrivedThis sentence requires the selection of simple past tense for the verb to flee, as well as a clarification of the antecedent of the introductory phraseapparently tipped off.

Page 93: Economist

A. This option incorrectly implies that the police were tipped off, and the progressive verb form was fleeing does not sufficiently indicate the time period before they arrived.

B. The phrase the police is incorrectly indicated as the antecedent of the phrase tipped off; the comma is unnecessary.

C. The verb are is present tense when past tense is required.

D. The suspect is identified as the antecedent of tipped off, but the sentence implies it was the police who were fleeing the house.

E. The suspect is correctly identified as the antecedent of the phrase tipped off, and the verb fled is the required past tense.

The correct answer is E.Consumer Safety Advocate:The most recent series of holiday string lights is dangerous because the plastic coating over the wires is thin, making it more likely that the wires will be exposed and cause a fire.

Manufacturer:Our company has taken extensive precautions with the manufacturing process of the glass for the bulbs to assure that each one can withstand high temperatures without cracking. Because of this process, the danger of fire is extremely low.

The manufacturer’s response is flawed as a refutation of the consumer safety advocate’s argument because it  

Does not offer to issue a recall order on a product that is clearly dangerous.

Does not address the consumer safety advocate’s concern about the dangers posed by thin wiring.

Does not give specific information on the process by which the glass is strengthened.

Focuses too much attention on the process that strengthens the glass, and gives too few details about the manufacture of the wiring.

Page 94: Economist

Focuses too much attention on the manufacturer’s precautions and not enough on the consumer safety advocate’s concerns.Situation:A consumer safety advocate complains that the wiring of holiday string lights is likely to cause a fire; the products manufacturer answers with a statement about the precautions taken with the preparation of glass for bulbs.

Reasoning:Which response represents the most serious flaw in the manufacturer’s response? The consumer safety advocate is concerned that thin wiring in the holiday string lights is likely to break and expose bare wires, making a fire likely. In order to refute this concern, the manufacturer would need to directly address concerns about the thin wiring.

A. While a recall order may be important if the product is deemed dangerous, this response does not directly address the consumer safety advocate’s concerns about wiring.

B. This option correctly identifies the flaw in the manufacturer’s argument: the manufacturer has failed to address the consumer safety advocate’s concern that thin wiring will cause a fire.

C. Even if specific information on the process were given, that information would not address the consumer safety advocate’s concern about thin wiring.

D. The manufacturer does not focus on the process that strengthens the glass.

E. While it is true that the manufacturer does not focus on the consumer safety advocate’s concerns, the most serious flaw in the response is that the manufacturer discusses glass while the consumer safety advocate is most concerned with wiring.

The correct answer is B.Along Highway 1, traffic is very tightly controlled by traffic lights placed every half mile to cut down on speeding. Motor vehicle accidents, however, are more common along Highway 1 than on any other road in the city. Clearly, this problem can only be solved by adding more traffic lights to the highway, reducing speeding even more.

Which of the following, if true, casts the most doubt on the effectiveness of the solution proposed above?

Page 95: Economist

The average speed on Highway 1 is already 35 miles per hour, making more traffic lights unnecessary.

The city council does not have sufficient funds budgeted for improvements along Highway 1.

Highway 1 is the longest road in the city, making motor vehicle accidents statistically more likely.

Most motor vehicle accidents along Highway 1 are rear-end collisions that occur when one of the parties is unable to stop safely at a traffic light.

There are more traffic-light controlled intersections on Highway 1 than there are on- and off-ramps.Situation:Additional traffic lights are proposed along Highway 1 to cut down on speeding. It is assumed that speeding is the cause of most motor vehicle accidents along the highway.

Reasoning:Which of the following casts the most doubt on the proposed solution? The solution proposed in the passage assumes both that most accidents along Highway 1 are caused by speeding and that traffic lights are the most effective method of reducing speeding. If, however, it can be demonstrated that most motor vehicle accidents have a cause other than mere speeding, the proposed solution will be seriously undermined.

A. The speed limit on the road is irrelevant because drivers can easily choose to break it.

B. Though this fact might make implementation more difficult, it does not directly address the effectiveness of the proposed solution.

C. This may be an explanation for the frequency of motor vehicle accidents, but it does not address the effectiveness of the proposed solution.

D. This option casts the most doubt on the effectiveness of the proposed solution by offering an alternative explanation for the causes of the frequent accidents along Highway 1. If most accidents are caused by an inability to stop safely at traffic lights, then more lights will only add to the problem.

Page 96: Economist

E. This information is irrelevant to determining the effectiveness of the proposed solution.

The correct answer is DAlthough structural damage from the earthquake was severe, less than ten people were injured.

from the earthquake was severe, less than ten

from the earthquake was severe; less than ten

from the earthquake was severe, fewer than ten

of the earthquake was severe, fewer than ten

was severe, less than tenThis sentence depends on the correct use of the preposition from to describe damage caused as a result of a natural disaster, such as an earthquake. Also, it must be recognized that the word fewer should be used to denote a number of persons who can be counted.

A. Although the idiom is correct, the phrase less than ten to refer to people is incorrect.

B. The introductory clause should be set off by a comma, not a semicolon.

C. The phrase from the earthquake is idiomatically correct, and the word fewer correctly denotes the number of persons who were injured.

D. The preposition of does not correctly indicate damage caused by the earthquake.

E. This choice eliminates from the earthquake, which is necessary to the sentence.

The correct answer is CEven though the foreign exchange student was from a completely different culture, it was not long before she felt comfortable between us.

between us

between most of us

Page 97: Economist

around us

among us

in this culture where the rest of us are comfortableThe sentence is testing the correct use of the preposition among to refer to the position of persons or things in a group of other people or objects.

A. The word between should be used only for groups of two and does not sufficiently establish the idea of a culture

B. Between should be used only for groups of two; most of is unnecessary

C. Though technically correct, the phrase around us does not give the sense of the student being in a completely different culture

D. The word among correctly locates the student in the midst of a completely different culture.

E. The phrase is awkward and unnecessary.

The correct answer is D.The cost of gas-burning grills has dropped nearly 40% in the last year, prompting an increase in sales. Hank’s Hardware responded to the popularity of gas grills with large displays and deeply-discounted sale prices. Sales of gas grills at Hank’s Hardware went up by a third, but at the end of the quarter managers were shocked to find they had lost money on gas grills.

Which of the following, if true, is the best explanation for why Hank’s Hardware’s profits on gas grills are suffering?

Sales volume has been miscalculated, surprising managers with a loss in profits.

The further discounts offered by sale prices account for a larger dollar amount than the increase in sales, leading to a loss of profits.

The reduction in cost is caused by shoddy construction, which is why many people who purchased gas grills are returning them, leading to a loss in profits.

Page 98: Economist

Wood-burning grills are more popular than gas-grills; since Hank’s carries mostly gas grills, their profits are suffering

Customers only bought gas grills when they were on sale; once the sale was over, they stopped buying the grills, leading to a loss in profits.Situation:After sales of gas grills have increased by a sizable percentage, the managers of Hank’s Hardware are surprised to notice a loss in overall profits.

Reasoning:Which of these is the best explanation for the loss in overall profits?The increase in the number of grills sold is only one factor to keep in mind. The price of grills has gone down, and this loss in income has been compounded by sale prices on the grills. If the managers of Hank’s Hardware offered sale prices that were too deeply discounted, they would lose money even if more grills were being sold.

A. Outside the scope of the question. While it is possible managers miscalculated sales volume, it is not discussed in the passage.

B. This option correctly identifies the potential of money lost to discounts outweighing money gained by larger sales volume.

C. The passage does not mention either that construction is shoddy or that customers are returning the grills.

D. The passage does not compare the relative popularity of wood- and gas-burning grills.

E. It may be true that customers stopped buying grills after they were no longer on sale, but this answer choice does not offer an explanation for the loss in profits.

The correct answer is BWhen Sven went to the bank last week, he asked not only for the balance on his account, but also if he could get information about a savings account.

if he could get

if they would give him

to be given

about

Page 99: Economist

forThis sentence tests knowledge of the idiom to ask for, as in to ask for information. This particular idiom should be used in order to maintain parallelism with the phrase he asked...for the balance on his account, which appears in the first half of the sentence. The words not only...but also indicate that the phrase he asked...for has been extended to the second half of the sentence.

A. If he could get is awkward and does not maintain the parallelism required by the sentence.

B. The phrase if they would give him is wordy, does not maintain parallelism, and introduces the pronoun they without specifying its antecedent.

C. To be given creates an inappropriate passive construction and does not maintain parallelism with the first half of the sentence.

D. While the idiom to ask about is correct in certain contexts, the phrase not only...but also embedded in this sentence indicates that the needed preposition is for. Also, when read back into this sentence, about is redundant.

E. This option correctly satisfies the parallelism required in the sentence by the phrase not only...but also and uses correctly the idiom to ask for.

The correct answer is EWhich of the following best completes the passage?

Other supermarkets may contain delis or coffee bars, but the powerful blend of specialty food market with café that has made Duetto Market so popular sets the standard for what is sure to be a national trend.Customers dining in the café sample expensive specialty foods in expertly prepared entrees and appetizers before purchasing raw materials in the market. Owners consolidate costs by ordering larger quantities of spices, meat, produce, and cheeses, some of which are then used in preparing food for the café. As more café and supermarket owners follow this pattern of blended options, they can expect not only to please a wider variety of customers, but also to  .

Increase their profit margins by consolidating the costs of food products they order from wholesalers.

Page 100: Economist

See the value of their stocks rise as more investors participate in a fashionable marketing concept.

Face the expense of renovating existing spaces to accommodate the combination of prepared and raw foods.

Receive more attention from media who publish restaurant reviews.

Gain more customers who are interested in dining as well as shopping.Situation: A blend of specialty food market and cafe offers benefits to both customers and owners.

Reasoning:Which option best completes the passage? The passage is focused exclusively on the benefits of blending the two types of business: customers can sample expensive foods before purchasing them, and owners can consolidate costs. The blank will be filled by a positive outcome and will reflect this dual concern with benefiting both customers and owners.

A. This option correctly fills the blank with a further benefit to owners of the Duetto Market model. Just as the second and third sentences of the passage alternately discuss benefits to customers and then to owners, the final sentence paraphrases those benefits.

B. While this is a potential positive outcome, the passage does not mention Duetto Market stock.

C. This option presents a potentially negative outcome, when the tone of the rest of the passage is overwhelmingly positive.

D. This could be true, but is outside the scope of the passage.

E. This option does not fit the passage’s focus on both customer and owner benefits.

The correct answer is ABefore the early eighteenth century, forms of communication among the deaf were often not regarded as authentic language. Accepted wisdom held either that the deaf were making simple gestures to communicate basic needs and ideas, or, in a more generous view, that sign language was a physical method of expressing the words and syntax of spoken language. Once it was recognized, however, that forms of signing

Page 101: Economist

constituted valid, though non-oral, systems of language, sign systems were standardized and disseminated in educational programs for the deaf.

American Sign Language was one of the first standardized sets of signs to develop in response to this new understanding of communication among the deaf. ASL had its beginnings in 1817 when Thomas Hopkins Gallaudet, an American minister, recruited Frenchman Laurent Clerc to co-found what is now the American School for the Deaf in Hartford, Connecticut. Clerc had been an instructor at the premiere educational institution for the deaf in France, where a standardized system of signs had been in use in for nearly a century. The language Clerc developed for the school was based heavily on the French Sign Language with which he, being deaf, routinely communicated. At the school, French Sign Language mingled with various and idiosyncratic “home signs” brought into the classroom by students from Martha’s Vineyard, an island off Massachusetts where there were an unusually large number of deaf children.Within only a few years, the mixture of French Sign Language with Home Sign had reached equilibrium and was taught in other institutions for the deaf as American Sign Language. Today, though it continues to be spontaneously adjusted to suit local needs, ASL is the third most common language in the United States, the dominant sign language of North America, and a popular form of inter-dialect communication in parts of Africa

The author likely relates the history of American Sign Language in order to

Entertain the reader with historical facts.

Advocate greater flexibility in the development of sign languages.

Compare two perspectives on types of communication used by the deaf.

Provide a historical example of a sign language that was first standardized then disseminated.

Explore cooperation among nations in the development of languages for the deaf.To answer a question about logical structure, consider the role the section plays in the passage as a whole. The first paragraph introduces a historical development, beginning with the idea that methods of communication among the deaf were not always considered authentic language. According to the passage, however, accepted wisdomchanged: once communication among the deaf was accepted as language, systems of signs werestandardized and disseminated. The history of American Sign Language is a historical example of this shift in thought.

Page 102: Economist

A. The passage is not meant to entertain.

B. The passage does not discuss, much less advocate, flexibility in the development of sign languages.

C. Two perspectives are not compared.

D. The section on American Sign Language provides a historical example of the pattern of sign language development discussed in the passage.

E. Though individuals from various nations cooperated in the development of American Sign Language, the focus was not on the nations themselves.

The correct answer is D.It can be inferred from the passage that American Sign Language borrowed liberally from French Sign Language for which of the following reasons?

Many students at the school could read and write French, making the transition to French Sign Language simple.

French Sign Language was held by educators of the deaf in France to be almost flawless.

French Sign Language was a standardized system of signs already familiar to one of the founders of the school.

French Sign Language was equally as expressive as the “home sign” languages that students from Martha’s Vineyard brought to the school.

The founders of the school believed French Sign Language to be superior to British Sign Language.An inference is drawn from information stated in the passage. To answer this question, review what the passage says about French Sign Language and the ways it came to form the base of American Sign Language. French Sign Language appeared at the school contained in the knowledge of Laurent Clerc, who, like his students, was deaf. According to the passage, Clerc used French Sign Language to communicate. Based on this information, it is reasonable to infer that Clerc based American Sign Language on French Sign Language because it was the system of signs with which he was most familiar.

A. The passage does not discuss whether students at the school could read and write French.

Page 103: Economist

B. Although French Sign Language was used by educators in France, the passage does not discuss whether or not they considered it to be flawless.

C. The passage implies that Clerc based American Sign Language on French Sign Language simply because that is the system of signs with which he was most familiar.

D. The expressiveness of French Sign Language and “home sign” is not compared in the passage.

E. British Sign Language is not mentioned in the passage.

The correct answer is C Accepted wisdom held either that the deaf were making simple gestures to communicate basic needs and ideas, or, in a more generous view, that sign language was a physical method of expressing the words and syntax of spoken language.In the highlighted lines the author most likely says that the second traditional understanding of sign language as a physical method of expressing the words and syntax of spoken language was a more generous view because

It considers deaf people incapable of using words and syntax in ways that imitate spoken language.

It was the more commonly held view among people who gave the most money to the deaf.

This position, though not indisputable, is a paraphrase of the point the author is making in the rest of the passage.

This perspective, though not completely accurate, was nearer the understanding that sign language is a valid system of language.

This viewpoint is the same as that eventually espoused by Gallaudet and Clerc.This question references specific lines, indicating the question can be answered by reviewing information in that section of the passage. The lines are contained in a sentence contrasting two ways conventional wisdomunderstood methods of communication used by the deaf; the phrase a more generous view makes it clear that the author prefers the second. The following sentence further clarifies the author’s position, as it becomes apparent that the author is concerned with establishing sign languagesas valid, though non-oral, systems of language.

A. This is the opposite of the author’s position in the passage.

Page 104: Economist

B. The word generous is misconstrued in this option to mean giving money rather than having understanding.

C. Sign language as a physical method of expressing...spoken language is not discussed in the passage. In fact, the author argues that sign language is quite distinct from spoken language.

D. This option correctly identifies the author’s overall concern in the passage: to establish sign languages as valid language systems, even though they are distinct from spoken language.

E. Galloudet and Clerc’s position on this opinion is not discussed in the passage.

The correct answer is DBased on information in the passage, the author of the passage would most likely agree with which of the following?

Gallaudet would have been unable to found a school for the deaf without Clerc’s assistance.

French Sign Language and Home Sign were equally important contributions to the development of American Sign Language.

American Sign Language eventually surpassed the usefulness and flexibility of French Sign Language.

Centuries of misunderstood methods of communication among the deaf made the development of standardized sign languages inevitable.

The deaf living on Martha’s Vineyard would not have formed their own standardized system of signs if they had not been introduced to French Sign Language.The question’s use of the phrase would most likely agree indicates that the answer depends on making an inference. To answer the question, look for support for each answer choice in the passage. These lines state that French Sign Language and Home Sign were a mixture that had reached equilibrium before American Sign Language could be taught in other institutions. It is reasonable to assume, then, that the author would view both French Sign Language and Home Sign as important components of American Sign Language.

A. Gallaudet is mentioned only as a founder of the school; the passage does not speculate about his ability to found a school without assistance.

Page 105: Economist

B. Correct. These lines discuss French Sign Language and Home Sign as components of a mixture that eventually reached equilibrium, suggesting that the author sees the two traditions as equally important contributors to American Sign Language.

C. The passage does not compare the usefulness or flexibility of French and American Sign Language.

D. No evidence is provided to support the assertion that the development of standardized sign languages was inevitable.

E. The potential of Martha’s Vineyard residents to form as standardized sign language is not mentioned.

The correct answer is BSince taking control of the capital, the BBC reporter says security forces have arrested about 40 insurgents of the majority political group, killing one of them.

Since taking control of the capital, the BBC reporter says security forces have arrested

The BBC reporter says that, since taking control of the capital, security forces have arrested

The BBC reporter, since taking control of the capital, says security forces

Security forces, since taking control of the capital, have, the BBC reporter says, arrested

Security forces have arrested the BBC reporter since taking control of the capitalThis question requires clarification as to who took control of the capital. In the original sentence, it seems as if the BBC reporter took control; to fix the sentence, the acting party –security forces – must be clarified.

A. This option splits the acting party, security forces, from the action, taking control of the capital.

B. This option places the BBC reporter at the beginning of the sentence as the observing party and makes it clear that security forces have taken the capital.

Page 106: Economist

C. This option resolves the ambiguity of the original sentence, but incorrectly credits the BBC reporter with taking control of the capital.

D. This option is unnecessarily wordy and even more confusing than the original.

E. Insurgents, not the BBC reporter, were arrested by security forces.

The correct answer is BAlthough it would be unexpected for an attacking dragonfly to gain advantage by presenting himself openly, scientists observed that in 8 out of 15 inter-species battles between male dragonflies, the attackers did not prevent detection of their presence as they would from predators. Instead, the dragonflies employed a technique called “motion camouflage” by which they attacked openly, but in a straight line from a fixed object. The reaction times of their adversaries, who did not seem to realize they were approaching, were slowed significantly, and the attackers were victorious in each case.

Which of the following, if true, forms the best basis for at least a partial explanation of the success of motion camouflage in dragonfly battles?

The dragonflies attacking in a straight line moved more quickly than other dragonflies observed in battle, causing their adversaries’ reaction times to seem slow by comparison.

The adversaries of the dragonflies, like scientists, do not expect to be attacked openly.

Scientists record a lower success rate for dragonflies who attack using a zig-zag pattern, rather than a straight line attack.

Scientists theorize that by remaining on a straight line with some landmark point, the attacking dragonfly remains stationary from the point of view of his adversary.

Scientists discover inaccuracies in the devices used to measure reaction times of adversaries.Situation:Contrary to what might be expected, male dragonflies attack openly and are successful. Scientists call their technique “motion camouflage.”

Page 107: Economist

Reasoning:Which point provides a basis for explaining this phenomenon?Discussion of male dragonflies in this passage centers around their use of camouflage, a technique used to manipulate what an adversary’s vision. Dragonflies use camouflage to hide from predators, but in the inter-species battles discussed in this passage, they do not hide, but present themselves openly. The correct answer will offer the best explanation for the way in which an open attack still manipulates an adversary’s vision.

A. This option does not account for the success of the “motion camouflage” technique.

B. The expectation discussed in the passage is that of the scientists; the expectations of the dragonflies would be impossible to determine, and are outside the scope of the passage.

C. A zig-zag pattern is not discussed in the passage; this observation would add little to an explanation of the success of an open attack.

D. The reason they can’t tell the attackers are moving is because they stay in basically the same place relative to a background object.

E. .This option would not negate the fact that dragonflies are consistently successful when they attack in the open.

The correct answer is DArchaeologists should make greater use of satellite images in their search for Mayan ruins in the jungles of Guatemala. Though satellite photographs have proven unhelpful because of their inability to penetrate the dense tree canopy, infrared imaging translates heat signatures into distinct colors, making previously-undiscovered ruins visible.

The conclusion above is based on which of the following assumptions?

Archeologists in the field have not been able to discover many ruins because the jungle is impenetrable.

Archaeologists use both satellite technology and local folklore to theorize the location of ruins.

Page 108: Economist

Archaeologists require special training in order to read maps produced by infrared imaging.

Satellite photography has not produced images of sufficient detail to locate ruins in the jungle.

Infrared imaging is helpful because the heat signature of ruin sites is different from the heat signature of the surrounding jungle.Situation:The author of the passage argues that archaeologists should use satellite infrared imaging to locate Mayan ruins.

Reasoning:Which of the following is the assumption underlying the author’s conclusion? The author first dismisses satellite photography because photographic images capture only the tops of the trees and not ruins that may lie beneath. Infrared imaging, however, penetrates the jungle and detects heat signatures on the ground, translating them into colors that can be mapped. The passage assumes that the ruins will have a distinctive heat signature, setting them apart from the surrounding jungle.

A. The passage states that the jungle is impenetrable to satellite photographs, not to archaeologists.

B. The passage does not mention the use of local folklore.

C. The passage does not mention a need for special training to read maps produced by infrared imaging.

D. The problem with satellite photography is not its lack of detail, but its inability to penetrate the dense tree canopy.

E. This option correctly identifies the assumption in the passage that infrared imaging will be successful because ruins and jungle have different, distinctive heat signatures.

The correct answer is EBeing that completing a master’s degree opens many lucrative job options, many people are willing to balance full time professional work, family life, and graduate classes to gain access to a higher-paying career track.

Being that

Page 109: Economist

If

Because

When

Notwithstanding the fact thatThis sentence establishes a direct relationship between positive job options made possible by graduate education and the sacrifices of time and money made by some students. The link is one of causation, and should be clear from the first word of the sentence.

A. Being that establishes a causal relationship, but is wordy.

B. If establishes a conditional, not a causal relationship.

C. Because correctly establishes the relationship between benefit and sacrifice in the sentence. Because more options are available to people with graduate degrees, many people are willing to sacrifice to achieve them.

D. When suggests a temporal, or temporary, relationship and does not establish cause.

E. Notwithstanding the fact that is awkward and establishes contrast, not cause.

The correct answer is CThe developers of Overlook Ridge built 200 luxury homes for the subdivision last year. Despite beautiful views and modern amenities, however, at least two-thirds of the homes remain unsold. For their own sake, developers should scrap plans to add another 100 homes to the subdivision this year.

Which of the following, if true, would provide the greatest support for the author’s conclusion?

The most recent census indicates that mean income in the area has been steadily declining over the last five years, making new home purchases unlikely.

An opinion poll in the local newspaper found that most home-buyers prioritize easy access to parks over beautiful views.

Page 110: Economist

Homeowners currently residing in the subdivision have registered several complaints about the noise of ongoing construction.

The eventual sale of another 100 homes will add greatly to the community’s tax base.

The architectural style of the proposed additional homes will be slightly more adventurous, in order to lure potential buyers to the subdivision.Situation:The developers of a local subdivision have built more homes than they have been able to sell. The author of the passage argues that they should not build any more homes.

Reasoning:Which option will provide the greatest support for the argument against building more homes?Most of the luxury homes built in Overlook Ridge have not been sold, in spite of amenities developers consider attractive to potential home-buyers. The correct answer will provide information about the likelihood of current and future houses in Overlook Ridge being sold.

A. This option correctly identifies a vital piece of information: if mean incomes in the area have been decreasing, it is unlikely that the homes currently standing empty will be sold. If houses that have already been constructed will likely not sell, it is clearly in the developers’ best interest not to build more homes they can be reasonably certain will not sell.

B. Nothing in the passage indicates that Overlook Ridge does not include easy access to parks; this option does not speak directly to the likelihood of additional homes being sold.

C. This option does not speak directly to the likelihood of additional homes being sold.

D. This statement may be true, but it would undermine the author’s conclusion, not support it.

E. Even if buyers purchased the more adventurous homes, the two-thirds of empty homes already built would likely remain that way.

The correct answer is AThe author of a book tracing the relationship of work-related stress to productivity loss in small business argues that most workers underestimate the harmful physical

Page 111: Economist

effects of unprocessed anxiety. Powerful hormones such as cortisol and epinephrine, which are released in stressful situations, raise heartbeat and blood pressure, priming the body for physical activity. However, since most business is conducted in a relatively restrained environment, workers find few opportunities to respond to stressful situations as their hormones seem to demand.

If all of the above statements are true, it can be most directly inferred from the passage that

Most productivity loss in small business is due to unprocessed stress hormones.

Increased physical activity would be helpful in processing stressful situations and increasing productivity.

Hormones such as cortisol and epinephrine stimulate critical thinking as well as raising blood pressure.

The restraint of business environments contributes to the stress experienced by workers.

Workers in more physically demanding jobs have fewer problems with work-related stress.Situation:A book argues that there is a strong connection between work-related stress and productivity loss because of the harmful physical effects of stress.

Reasoning:Which of the following can be most directly inferred from the passage?The first sentence introduces the idea that work-related stress can have harmful physical effects, and the second sentence discusses the functioning of stress hormones such as cortisol and epinephrine. The third sentence points to the fact that restraint in business environments may prevent workers from reacting physically to stressful situations, in which case the harmful physical effects of stress would lead to productivity loss.

A. The passage does not make this claim.

B. Since the author is concerned with productivity loss as a side-effect of harmful physical effects that result from lack of physical activity, the author would view increased physical activity as a solution.

Page 112: Economist

C. The effect of these hormones on critical thinking is not discussed in the passage.

D. In the passage, restraint does not contribute directly to stress, but rather to a barrier in processing it.

E. The passage does not discuss workers in more physically demanding jobs.

The correct answer is BThe majority of patrons of the local library, though enthusiastic about a new book mobile service, is unable, or either unwilling , to volunteer time to staff it.

is unable, or either unwilling

is either unable or unwilling

are unable, or either unwilling

are either unable or unwilling

are unable or unwilling, either way,The sentence is testing the ability to recognize majority as a plural word requiring agreement with a plural verb, as well as the recognition of concision in aneither...or phrase.

A. The verb is does not agree with the plural subject majority, and the phrase or either unwilling is unnecessarily wordy.

B. Is does not agree with the plural subject majority.

C. The phrase or either unwilling is wordy and unnecessary

D. The verb are agrees with the plural subject majority, and the rest of the phrase displays appropriate concision.

E. The verb are agrees with the plural subject majority, but the use of either way as an additional phrase is unnecessary and wordy.

The correct answer is D

Page 113: Economist

The section’s new manager is limited in his effectiveness because he is convinced that, of all the possible methods for monitoring progress, the most effective is checking on his employees every few minutes.

is checking on

is checking in on

are checking in on

are checking upon

are checking onThe sentence requires the singular verb is to agree with the most effective (method), and the correct use of the idiom check on. The sentence is correct.

A. The verb is correctly identifies the most effective (method); the idiom checking on is used correctly.

B. The word in is unnecessary in the idiom.

C. Are is an incorrect plural verb; the word in is unnecessary.

D. The verb are is plural and incorrect; the word upon is not correct for the idiom.

E. Are is an incorrect plural verb.

The correct answer is A.Which of the following best completes the passage?

Hydraulic rescue tools, colloquially known as Jaws of Life, are used by emergency response personnel to remove crash victims from damaged vehicles. Hydraulic tools have often completely replaced traditional tools, such as crowbars and circular saws, which could cause additional structural damage or create sparks. Emergency response workers prefer hydraulic tools, not only because they are less frightening to crash victims, but also because  

One hydraulic rescue tool can perform the functions of both a crowbar and a circular saw.

Hydraulic tools are quieter and do not cause crash victims to panic.

Page 114: Economist

Hydraulic tools are unlikely to complicate an already dangerous situation in the way that more traditional tools often did.

Hydraulic tools are able to quickly and efficiently repair structural damage to vehicles involved in collisions.

Emergency response workers have greater leverage when using hydraulic tools than when using traditional tools, such as crowbars and circular saws.Situation:Emergency response personnel prefer hydraulic rescue tools to traditional tools because they do not cause additional dangers in a rescue situation.

Reasoning:Which answer choice best completes the sentence? The passage compares hydraulic rescue tools, commonly known as Jaws of Life, to traditional tools, such as crowbars and circular saws. According to the passage, traditional tools are likely to cause additional structural damage or to create sparks, both of which would sufficiently increase the level of danger in a rescue situation. Hydraulic tools are preferred because they do not introduce new dangers.

A. This is not stated in the passage.

B. This phrase repeats the first half of the sentence.

C. This option correctly identifies the clearest advantage of hydraulic tools over traditional tools: hydraulic tools do not themselves add danger to a rescue situation.

D. Though traditional tools may cause structural damage, the passage does not discuss whether hydraulic tools are used to repair such damage.

E. The passage does not discuss which tools give rescue workers greater leverage.

The correct answer is CBefore the storm destroyed much of the harbor, this town, with its sandy beaches and multiple opportunities for sports, had been one of the most popular resort destinations on the East Coast.

had been

has been

Page 115: Economist

had been being

was being

wasThis sentence requires simple past tense form to indicate the entire period of time before the storm destroyed much of the harbor.

A. Had been is wordy and unnecessary and creates past perfect tense, when simple past tense is more appropriate.

B. The word has is incorrect; it indicates that the condition of the town continues to the present.

C. This option compounds the error of the wordy phrase had been by adding another unnecessary word, being.

D. Was being is grammatically incorrect; the word being is unnecessary.

E. The verb was is sufficient to create simple past tense and to give the best meaning to the sentence.

The correct answer is EThe Chicago School of economics gained ascendancy in the 1950s and became the prevailing way of thinking about capitalist economics, first in the United States and eventually in many Western countries. A dominant subset of “Neoclassical economics,” the Chicago School fine-tuned many of the fundamentals of the larger movement and articulated tenets now generally taken for granted in economic thought.

A basic principle of Neoclassical economics is the concept of utility, or the assertion that a correct economic decision is one that yields “the greatest good for the greatest number.” Neoclassical economics vociferously defends a laissez-faire, or “hands off,” approach to government regulation. According to Neoclassical theory, in the absence of state interference, individual participants in the market will make rational economic decisions that maximize their satisfaction. For individuals, the principle of utility translates into buying quality goods at the lowest possible price; for companies, it means making decisions that will maximize profits. The interplay of free market forces, according to Neoclassical economists, will result in greater good – in the form of high quality products and services – for a greater number of people.

The Chicago School accepts the principles of utility and laissez-faire regulation, but tailors its understanding of these terms to focus on the related principle of

Page 116: Economist

“efficiency.” For Chicago School thinkers, “efficiency” subsumes Neoclassical thought on the mechanisms of utility, which concentrates on individuals making individual rational decisions, into a more sophisticated awareness of the balance between individual decisions and production. The chief cause of inefficiency for Chicago School thinkers is government regulation, which prevents the free interplay of market forces. In an economy unencumbered by state interference, not only are individuals and companies free to maximize their satisfaction, but production itself is likely to become more efficient by producing the highest quality goods at the lowest possible price

According to the passage, which of the following best articulates the relationship of utility to efficiency?

Utility is concerned with maximizing satisfaction, whereas efficiency is concerned with greater production.

Utility is the basic principle, whereas efficiency is a more refined version of the basic principle.

Utility and efficiency are equally important concepts in Neoclassical economic theory.

Utility and efficiency, though opposite in meaning, are both useful in describing the statistical behavior of free markets.

Neither utility nor efficiency, as a principle, is able to completely predict the behavior of a free market.To answer this question, an understanding of the termsutility and efficiency as they are used in the passage is required. The second paragraph discusses the definition of utility as an economic decision that yields the greatest good for the greatest number. The first sentence of the second paragraph states that the Chicago School accepts this definition of utility, but has refined it and added other subtleties to it, so that the School’s major apparent concern is with efficiency.

A. According to the passage, utility and efficiencyare aspects of the same concern with maximizing satisfaction through greater production; this answer choice presents a redundant sentence.

B. This option correctly identifies the connection made in the passage between the ideas of utility, the basic principle associated with Neoclassical economics, and the same principle refined by the Chicago School and discussed as efficiency.

Page 117: Economist

C. Efficiency is not discussed in the passage as being a major component of Neoclassical theory.

D. Utility and efficiency are not opposite in meaning.

E. Whether or not utility and efficiency can predict the behavior of a market is outside the scope of this passage.

The correct answer is BWhich of the following best describes the primary purpose of this passage?

To discuss the relationship of the Chicago School to Neoclassical economic theory.

To demonstrate the fundamental disagreement between the Chicago School and Neoclassical economic theory.

To argue for the application of the principle of utility in the United States economy.

To trace the development of the concept of efficiency in American economic theory.

To outline a plan for producing the highest quality goods at the lowest possible price.The primary purpose of the passage can best be determined by considering the passage as a whole. The first paragraph establishes the Chicago School as an important subset of Neoclassical economic theory. The second paragraph reviews the principles of Neoclassical economics, and the third discusses the innovations of the Chicago School on the fundamentals of Neoclassical theory.

A. This option correctly recognizes the informational tone of the passage as a whole and accounts for the entire logical structure of the passage.

B. A fundamental disagreement between the Chicago School and Neoclassical economic theory is not discussed.

C. The principle of utility is assumed to have already been applied to the United States economy.

D. Trace implies a concern with chronology that is not supported by the passage.

E. Though this idea is mentioned, it is not a focus of the passage as a whole.

The correct answer is A

Page 118: Economist

The interplay of free market forces, according to Neoclassical economists, will result in greater good – in the form of high quality products and services – for a greater

number of people.The author of the passage most likely states that greater good comes in the form of high quality products and services for which of the following reasons?

To underscore the fact that Neoclassical economists are more concerned with the greater good than other types of economists.

To remind readers that, for the principle of utility to truly apply, goods and services must be put to appropriate use.

To emphasize the fact that products and services are good, but high quality products and services are better.

To distinguish between economic good (quality products and services) and economic evil (lack of quality products and services).

To make clear an implied distinction between moral good (virtue) and economic good (quality products and services).The words most likely states in the question stem are a signal that the correct answer will deal with material implied but not directly stated in the passage. The relationship of laissez-faire regulation with the principle of utility is being discussed in the second paragraph. The line in question is making a connection between the benefits of limited government regulation and achieving the goal of the principle of utility. This sentence also clarifies a possible misunderstanding in the meaning of the word good.

A. The passage does not make this claim.

B. Appropriate use of goods and services is not discussed in the passage.

C. This is an inaccurate definition of the term greater good.

D. This distinction is not made in the passage, nor iseconomic evil mentioned.

E. This option correctly identifies the sentence’s purpose in clarifying an implied distinction between uses of the word good.

The correct answer is EAccording to the passage, all of the following are benefits of a laissez-faire economy EXCEPT

Individual participants will make decisions that maximize their satisfaction.

Page 119: Economist

Companies will make decisions that maximize their profits.

Production of goods will become more efficient.

Government agencies will be available to correct inflation.

Quality goods will be produced at the lowest possible price.The words according to the passage in the question stem indicate that the correct answer can be chosen by reviewing details from the passage. Go through each answer choice and confirm that it can be supported by a specific line in the passage. The one that IS NOT supported by the passage is the correct answer.

A. These lines  discuss how individuals will maximize their satisfaction.

B. There is an indication that companies will maximize their profits.

C. This passage  discusses efficient production of goods.

D. The passage states explicitly that a laissez-faireeconomy means little interference from government.

E. There is a discussion of the production of quality goods at a low price.

The correct answer is DPublisher Big Books’ proposal to encourage new authors by founding an imprint, or brand name, exclusively for first novels is doomed to artistic failure. New authors have not had the experience necessary to refine their writing styles, and a wistful desire to publish hoards of new authors will only lower quality standards and flood the market with mediocre prose.

Which of the following, if true, would most weaken the argument against the publisher’s proposed imprint?

Authors hoping to be published will share the cost of publication.

First novels will be marketed only by in-store displays, not with posters, which are more expensive.

All books published under the new imprint are held to rigorous, pre-determined standards of quality.

Page 120: Economist

Many novels that are now considered classics were the first novels published by their authors.

Manuscripts will be solicited from authors who have already proven successful at writing short stories.Situation:The author of the passage claims that a proposed imprint for first novels will result in the publication of works of little artistic merit.

Reasoning:Which point will most weaken the author’s argument?First, notice that the author of the passage is concerned with the artistic quality, not financial success, of first novels. The correct answer will speak directly to this concern and will assure that first novels published under the imprint will be evaluated for high artistic quality.

A. The author of the passage is not explicitly concerned with costs of publication.

B. The author of the passage is not explicitly concerned with marketing expenses.

C. This choice correctly answers the author of the passage’s concern that first novels be of high artistic quality.

D. While this may be true, it does not directly address the situation of seeking to publish more novels that are the first novels published by their authors.

E. Since novels and short stories are different types of writing, authors who may have been successful with short stories are not guaranteed to be as successful with novels.

The correct answer is CWhen scientists discuss the point in the evolutionary process at which a sense of fairness arose in human beings, one of two theories general prevails. The first theory holds that a sense of fairness – or, more generally, morality – is a late and mostly intellectual addition to the evolutionary process. In this view morality is defined as a set of rules agreed upon by a society, permitting human beings to “rise above” the raw violence and competition found in “nature.” In certain circles this idea is known as “veneer theory,” because it posits human morality as a thin crust on a violently churning interior.

The second theory, by contrast, does not set nature and morality in direct opposition, but rather holds a sense of fairness to be as much a part of human “nature” as violence and competition. Advocates of this theory most often appeal to the behavior of other primates to argue that humans possess an innate sense of fairness common to other species of a shared evolutionary past. Orangutans, for example, have been observed to

Page 121: Economist

throw temper tantrums when teased by being shown an apple they are not allowed to have. Experiments with monkeys offering them two kinds of rewards for performing simple tasks have shown that they will refuse the less-preferred reward if they see their companions receiving the preferred reward for performing the same task

Which of the following best describes the primary purpose of this passage?

To speculate about the origins of violence in humans and other primates.

To refute the idea that human nature is inherently violent.

To illustrate the ways in which primates demonstrate an inherent sense of fairness.

To compare and contrast two theories about the development of a sense of fairness in human evolution.

To criticize the assertion that human morality is intended to rise above the violence and competition found in nature.Examining the passage as a whole helps to identify the primary purpose or main idea. In this passage, the first sentence contains the key to understanding the primary purpose. The first sentence gives the general topic of the passage, then states that one of two theories generally prevails. This is a clue that the passage that follows will discuss each of the two theories, comparing and contrasting them.

A. The origins of violence are not discussed in the passage.

B. Though this idea is implied in the first theory, this answer choice does not speak to the passage as a whole.

C. A sense of fairness in primates is discussed in the second paragraph, but this option does not speak to the passage as a whole.

D. The first sentence of the passage introduces the fact that two theories will be discussed, and the logical structure of the passage follows this expectation.

E. Though this idea is mentioned in the first paragraph, the passage does not criticize it.

The correct answer is D.According to the passage, the common concern of the two theories presented in the passage is

Whether orangutans and other primates possess an innate sense of fairness.

Page 122: Economist

Whether a sense of fairness should be considered an earlier or more recent development in the evolution of human beings.

Whether a sense of fairness can rightly be calledmorality.

Whether morality is the basis of human social patterns, or is a surface concern.

Whether morality is innate, or the result of a series of social interactions and agreements.This question requires a determination of which option in the answer choices is common to both theories. The first theory, called veneer theory, posits that a sense of fairness, or morality, is a later development in the evolutionary process that allows humans to rise aboveraw nature. The position of the second theory is that a sense of fairness developed much earlier, not in humans only, but in primates generally. Both of these concerns are anticipated in the first sentence of the passage.

A. This concern is addressed only by the second theory.

B. This option correctly identifies the concern set out by the first sentence of the passage. Though the theories differ in their answers to the question, the common concern is the timing of the development of a sense of fairness.

C. The passage assumes, but does not directly address, whether a sense of fairness can aptly be called morality.

D. This concern is addressed explicitly only in the first paragraph.

E. The passage does not discuss this question directly

The correct answer is BWhich of the following, if added to the end of the passage, would make the best conclusion to the discussion of the second theory?

For advocates of this theory, the behavior of primates offers evidence that a sense of fairness evolved before primate species differentiated, making morality just as “natural” to humans as violence and competition.

In conclusion, both orangutans and monkeys were observed to raise strenuous objections when they felt that a received standard of fairness had somehow been violated.

Page 123: Economist

In contrast to veneer theory, then, the second theory’s use of observation and experiment lends it a credibility that can not be denied.

Because orangutans and monkeys, at least in these examples, demonstrated a clear sense of fairness, it can reasonably be concluded that morality comes more naturally to them than it does to humans.

Despite the suggestion that primates such as monkeys and orangutans possess an innate sense of fairness, however, most scientists continue to accept veneer theory.To answer this question, it will be necessary to understand the general point the second paragraph is attempting to prove. The passage as a whole is concerned with the point in the evolutionary process at which a sense of fairness arose; the second paragraph will deal with this question in a specific way, using the examples of observations of primates as supporting detail. The best answer choice will summarize only the second paragraph, and will do so in a way that re-connects the paragraph to the concern of the passage as a whole.

A. This answer choice summarizes the main points of the second paragraph in the phrases advocates of this theory and behavior of primates, and it restates the general concern of the passage, as stated in the first sentence.

B. Though the words in conclusion appear in this option, the sentence summarizes only the previous sentences about scientists’ observations of primates and does not sufficiently conclude the discussion of the second theory.

C. This option stresses too clear a preference for the second theory and is not appropriate within the overall purpose of the passage.

D. The idea that morality comes more easily to orangutans and monkeys is not discussed in the second paragraph.

E. Because of the assertion that most scientists prefer veneer theory, this sentence does not adequately conclude the second paragraph.

The correct answer is AThe passage suggests which of the following about the condition of “nature,” according to veneer theory?

Societies agree to rules of morality in order to recondition themselves to resist the violence and competition of nature.

Page 124: Economist

Nature is inherently violent and competitive and may be contained, but can not be changed.

The violence and competition found in nature must be transformed by morality in order for humans to live peacefully.

Human beings constitute a thin crust living on the outside of a chaotic core of nature.

Morality is as much a part of nature as violence and competition.The word suggests indicates that this question must be answered by making an inference based on information in the passage. First, review these lines In these sentences, it is stated that raw violence and competition are found in“nature” and that human morality is a crust or veneerthat does not change nature, but covers or controls it.

A. This option is tempting, but the first paragraph does not discuss a desire of societies to recondition themselves, and in fact implies that true reconditioning is impossible.

B. The first half of this sentence is a paraphrase ofline, and the second half is an inference drawn directly from the fact that veneer theory understands morality as only a crust that does not essentially alter the interior of nature.

C. It can not be inferred from the discussion of veneer theory that morality must transform nature.

D. This option is a misreading of line, which states that human morality is a thin crust on nature.

E. This idea is a paraphrase of the argument presented in paragraph two, not of veneer theory.

The correct answer is BAccording to the passage, which of the following is evidence that the development of a sense of fairness occurred early in the evolution of human beings?

History shows that violence and competition have not been a significant part of human behavior.

Societies establish laws designed to enforce agreed-upon ideals of fairness, and to protect members from violence.

Page 125: Economist

Observation demonstrates that this sense is shared by other primates such as orangutans and monkeys.

Orangutans became visibly upset when they were unfairly denied something they wanted.

Experiments involving monkeys prove that primates feel entitled to rewards when they have successfully completed simple tasks.This question is based on information stated explicitly in the passage. More specifically, this information will be found in the second paragraph, where the theory arguing that a sense of fairness was an early evolutionary development is discussed. The discussion of this theory assumes that all primates share a common evolutionary ancestor; the examples involving orangutans and monkeys provide evidence that a sense of fairness evolved very early, before the species had even differentiated.

A. This statement is outside the scope of the passage, which does not discuss the role of violence and competition in history.

B. This idea appears in the passage as part of the discussion of veneer theory, which argues that morality is a late development in human evolution.

C. This option correctly identifies the purpose of the examples involving orangutans and monkeys in the second paragraph: to prove that a sense of fairness developed early in human evolution, before primate species had differentiated.

D. This option is too narrow and concerns only orangutans, not humans.

E. This sentence, though drawn from the passage, does not directly address the question of the evolutionary timing of a sense of fairness in human beings.

The correct answer is CA proposed law would require people to bring photo ID to polling places when they vote to curb potential fraud.

places when they vote to curb potential fraud

places when they vote, to curb potential fraud

places, when they vote to curb potential fraud

places, when they vote, to curb potential fraud

Page 126: Economist

places, to curb potential fraud by votingThe sentence is testing correct placement of modifiers. As the sentence is worded in the question stem, it seems as if a vote is being taken to decide whether or not to curb potential fraud. The meaning of the modifying phrase to curb potential fraud should be clarified.

A. The phrase when they vote to curb potential fraud is misleading

B. The comma placed between vote and to curb correctly separates the two ideas, making it clear that photo ID is the item intended to curb potential fraud.

C. The comma is incorrectly inserted after places and does not sufficiently clarify which part of the sentence to curb potential fraud is intended to modify.

D. The commas setting off the phrase when they vote are unnecessary and do not help clarify the meaning of the sentence.

E. The comma in this phrase is incorrectly placed, and gives inadequate sense to the sentence.

The correct answer is BA developer wants to build a shopping mall in a mostly rural area of a mid-sized city. In the letter to the city council requesting a building permit, the developer argues that the permit should be granted because the shopping mall, if successful, will ultimately benefit the city by creating jobs in a place where the unemployment rate is slightly higher than the national average.

Which of the following, if true, will be most helpful in strengthening the developer’s petition before the city council?

A citizens’ group has circulated a petition collecting signatures in support of the developer’s plan.

The city code requires that proposals for all new structures be voted upon and approved by the city council.

The developer has already obtained verbal agreements from several retailers who plan to open stores in the future shopping mall.

Page 127: Economist

The city’s largest percentage of unemployed people lives in the rural area where the proposed shopping mall would be built.

The city will benefit financially from the new shopping mall, because all sales tax revenues will go to the city.Situation:A developer seeking permission to build a shopping mall argues that the mall will create jobs in an area where unemployment is high.

Reasoning:Which option will most strengthen the developer’s petition? The developer’s letter to the city council focuses on the benefits available to the city if the mall is constructed. The city will benefit if those currently unemployed begin working and spending money to increase the city’s tax base. The correct answer will follow the line of reasoning that the mall will be of benefit to the city.

A. This may demonstrate that local residents support the mall, but it does not necessarily aid the developer’s position before the city council.

B. City building codes are not relevant to the discussion of whether a new shopping mall will benefit the city financially.

C. This may boost confidence in the potential success of the shopping mall, but it does not follow the line of reasoning set forth in the developer’s letter.

D. Correct. This strengthens the developer's petition because of potential reduction of unemployment rate.

E. This answer choice, though provides a good argument, falls outside of the scope of the original statement

The correct answer is DGmat 2

For the first time since influenza vaccines became popular, the number of people expected to want the vaccine exceeds the amount of vaccine available. In order to manage the impact of this shortage on the public health, the governor has issued a directive ordering that the vaccine be made available first to groups most vulnerable to serious complications from the illness. The directive states that all children in the sixth grade and younger, and all people over 68 years of age, be provided the first

Page 128: Economist

option of receiving the vaccine through special 2-day "flu clinics" set up in elementary schools and senior citizens' centers.

If all of the above statements are true, it can be most directly inferred from the passage that

There will be rioting in the state in reaction to vaccine shortages.

The majority of un-vaccinated people will be working adults.

Some people in nursing homes will refuse the vaccine, making more available for working adults.

Students in the seventh through the twelfth grade will be next in priority for receiving the vaccine.

Some residents who are not in either priority group will be unable to find an available dose of vaccine.Situation:The governor of the state has designated priority groups to manage the impact of an expected shortage of flu vaccine.

Reasoning:What inference can be drawn from information in the passage? The question requires drawing distinctions between groups of people in the state. The two designated priority groups include all children younger than sixth grade and all adults older than 68 years. Vaccine remaining after the "flu clinics" will be distributed among the remaining population, consisting of residents older than sixth grade and younger than 68 years. Because there is an expected shortage of vaccine, it can be inferred that some people in this middle group will not be able to find an available dose.

A. Rioting is not feared nor hinted at in the passage.

B. Older children (grades 7-12), college students, and other adults younger than 68 years are also excluded from priority groups.

C. Though this option is possible, it is not a necessary conclusion of the passage.

D. The passage does not discuss secondary priority groups.

Page 129: Economist

E. Since the number of people expected to desire the vaccine is higher than the amount of vaccine available, some residents of the state seeking an available dose will be unable to find one.

The correct answer is EOur lethargic city center will not be adequately revitalized simply by expanding residential space in the form of high-priced condominiums, but must be aided by incentives for investment in small businesses. The revenue generated by condominium sales may indeed be helpful, but our expectations for economic growth can not be one-sided, nor can the residents of these so called "luxury living spaces" be expected to subsist without service industries operating within a reasonable distance. The city council must be aggressive in drawing new restaurants, laundries, childcare facilities, and the like to the city center - or the revitalization project will certainly fail.

Which of the following, if true, would most strengthen the argument that incentives for small businesses must be part of the city's revitalization project?

A case study of a similarly-sized city that traces the positive contributions of small businesses to urban renewal.

A petition from a citizens' group supporting incentives for small business in the city center.

A 100% sale rate for the most expensive condominiums in city center neighborhoods.

An article arguing that abundant, comfortable housing for young professionals is necessary for urban renewal.

A professor at a local university argues that encouraging small business in the city center will drain the resources available for renewal.Situation:Someone interested in renewing the city center argues the city council should encourage small business in the area.

Reasoning: Which is the strongest point in support of encouraging small business? The passage argues that, not only is small business essential to the renewal of the city center, but also that the condominium projects currently underway will not

Page 130: Economist

succeed without service industry small business. For the argument to be strengthened, evidence is needed that small business does in fact contribute to urban renewal.

A. A case study from a city of similar size where small business has been shown to aid in urban renewal will strengthen the argument that a similar goal should be set in this city.

B. This may demonstrate that small business is important to local residents, but it does not demonstrate that small business will aid urban renewal.

C. This item confirms the popularity of condominium developments, but does not directly address the question of small business.

D. The author argues explicitly that housing can not be the only concern of an urban renewal project.

E. This is actually an argument against encouraging small business.

The correct answer is AAdministrators at a local college recently passed a mandatory recycling rule for all faculty and staff. Campus offices will be provided with bins for sorting paper, plastic, and glass, and employees will be subject to reprimand if recyclables are routinely found in their trash bins. The college president said in a statement that he hopes not only to make the campus more environmentally friendly, but also to set a good example for students, so that they will develop a habit of recycling even when they are not on campus.

Which of the following is most like the college president’s stated reasons for the recycling rule in logical structure?

Parents punish their children when they fail to brush their teeth because they want the children to begin habits that will lead to healthy teeth and gums.

Health departments enforce rules that restaurants must post signs in restrooms reminding employees they are required to wash their hands before returning to work, protecting the public health both by assuring that employees practice good hygiene and by encouraging members of the public to do the same.

Page 131: Economist

Library policies state a penalty for returning borrowed books past their due date in order to assure that as many people as possible have access to the books they wish to read.

City sanitation crews post notices near sewers that drain rainwater to the local river announcing a penalty for dumping waste or motor oil.

A state legislature passes a law requiring local factories to reduce the amount of pollutants released into the atmosphere or the local watershed area in order to prevent damage to wildlife habitats.Situation A university has instituted a rule that it hopes will encourage students to recycle even when not on campus.

Reasoning Which argument has the same logical structure as the argument about the recycling rule? The university has issued a recycling rule that will make violators subject to reprimand. The actual goal of the rule, however, has two parts: to make an immediate, positive environmental impact by enforcing a rule on campus; and to exemplify environmentally-friendly habits that students will continue away from campus.

A. The emphasis here is on reprimand and immediate impact, not example.

B. A rule intended for one group of people (food service employees) has two goals: to positively impact the immediate public health through good hygiene, and to encourage members of the public to develop the same habits.

C. The rule about due dates, though it is meant to serve the public, is not intended to be instructive.

D. Though these signs may be intended to protect the public health, there is no attempt to set an example, nor is a specific group of people targeted for the rule.

E. Though this rule targets a certain group (factory owners and managers), it is difficult for it to serve as an example to the public, since individuals do not produce industrial wastes.

The correct answer is BThe host completed preparations for the party by hanging lights, polishing the silver, and then he mixed the punch.

Page 132: Economist

and then he mixed the punch.

and then he was mixing the punch.

and mixing the punch.

and mixing up the punch that would be drunk by the guests of the party.

and the punch.The original sentence contains a list of actions taken by the host to prepare for a party. In order to display proper parallelism, all verbs in the list must be in the same form, in this case present progressive (-ing).

A. Mixed is a past-tense verb form, when the sentence calls for a progressive form ending in –ing.

B. The words he was are redundant and unnecessary.

C. In this sentence, all verbs should display parallel, progressive (-ing) form. Mixing is the correct verb form.

D. Though the word mixing uses correct verb form, the words that would be drunk by the guests of the party are wordy and unnecessary.

E. This option does not contain a verb.

The correct answer is CBy the time the children get home from track practice, their mother will have finished putting new wallpaper in their bedroom.

will have finished

will be finishing

will finish

would have finished

would be finishing

Page 133: Economist

This sentence requires the correct verb form – future perfect tense – in order to demonstrate that an action will be completed at a definite point in the future.

A. In this sentence, the words will have finished correctly indicate that at a definite point in the future – by the time the children get home – the process of wallpapering the room will be complete.

B. The words will be finishing indicate future progressive tense, and suggest that the wallpapering will be an ongoing, rather than completed, process at the definite future point.

C. The phrase will finish is simple future tense and does not indicate a necessary relationship with the future point by the time the children get home.

D. The words would have finished indicate a conditional tense that does not make sense in the sentence.

E. The phrase would be finishing indicates a conditional tense that does not make sense in the sentence.

The correct answer is A.Like earth, the atmosphere of Jupiter is composed of a mixture of swirling, diverse gases.

Like earth, the atmosphere of Jupiter

Like earth’s, Jupiter’s atmosphere

Like earth, Jupiter’s atmosphere

As earth, the atmosphere of Jupiter

As earth’s, Jupiter’s atmosphereThe intent of the sentence is to compare the atmospheres of the earth and Jupiter; both are made up of a mixture of swirling, diverse gases.

A. This construction incorrectly compares the entire earth to only part of Jupiter, the atmosphere. The phrase the atmosphere of Jupiter is wordy.

B. This sentence correctly compares the atmospheres of two planets, earth and Jupiter, and uses concise language.

Page 134: Economist

C. This construction incorrectly compares the entire earth to only part of Jupiter, the atmosphere.

D. As generally means “in the capacity of” and is used incorrectly in this sentence. The phrase the atmosphere of Jupiter is wordy.

E. As generally means “in the capacity of” and is used incorrectly in this sentence.

The correct answer is BThough there are some exceptions, most adult Olympic athletes are hardly neophytes at the sports in which they compete; they likely enjoyed the same activities as a child.

they likely enjoyed the same activities as a child.

they enjoyed the same activities as a child.

they likely enjoyed the same activities as children.

he or she likely enjoyed the same activities as a child.

he or she likely enjoyed the same activities as children.This question concerns correct agreement between pronouns and antecedents. In this sentence the plural adult Olympic athletes in the sentenc's first clause indicates that the pronoun in the second clause should be the plural they. The antecedent, likewise, should be the plural children.

A. The plural pronoun they does not show agreement with the singular child.

B. Though this version is more concise than (A), it displays the same grammatical problem: the plural pronoun they does not show agreement with the singular child.

C. The plural pronoun they correctly agrees with the plural children in the second clause, as well as with the plural antecedent adult Olympic athletesin the first clause.

D. The pronouns he and she are both singular and do not agree with the plural antecedent adult Olympic athletes in the first clause.

E. The pronouns he and she are both singular and do not agree with the plural children in the second clause.

Page 135: Economist

The correct answer is C  Answer  Sh o w Answer  E x plain Answer

P revious  N ext

H elp  E nd Exam  R eview Section

The company is in bankruptcy and has no dividends to pay; however, if the merger with the larger company is successful, shareholders could receive from 10 cents on the dollar up to 40 cents on the dollar for their stock.

from 10 cents on the dollar up to 40 cents on the dollar

from 10 cents of the dollar to 40 cents of the dollar

from 10 cents on the dollar and 40 cents on the dollar

from 10 to 40 cents of the dollar

from 10 to 40 cents on the dollarA. The phrase from...up to is incorrect.

B. The phrase of the dollar is incorrect.

C. The phrase from...and is incorrect.

D. The phrase of the dollar is incorrect.

E. From...to is the correct use of a phrase indicating range. On the dollar is the correct use of a phrase to indicate a portion of currency. The sentence is also more concise than the original.

The correct answer is The study, prepared for the county's economic development council, represents the most thorough attempt to gauge the impact of increased food and beverage taxes in the southeastern region since they were raised six years ago.

the most thorough attempt to gauge the impact

the thorough attempt to gauge the impact

a most thorough attempt to gauge the impact

Page 136: Economist

a thorough attempt to gauge the impact

a most thorough gauge of the impactThe sentence requires a comparison of the current study with other possible studies implied by the context. Because of the phrase since they (taxes) were raised six years ago, the correct answer choice will display a superlative construction, indicated by the word most.

A. This choice correctly uses both the superlative word most and the demonstrative article the, further emphasizing the study’s unique character.

B. This choice lacks the word most, and the article the is vague and confusing.

C. This option includes the superlative most, but the word a does not indicate the study's unique character.

D. The word most is missing, and the word a does not emphasize the study's unique character.

E. Gauge is misplaced, making it sound as if the study itself is the measure of the increased taxes.

The correct answer is AThis process resulted in a total of 15 new townships: of which, to date, 7 are still in existence.

of which, to date, 7 are still in existence.

of which number, 7 of the 15 are still, to date, in existence.

of the number 15, 7, to date, are still in existence.

to date of these 15, 7 are still in existence.

to date, 7 of the 15 are still in existence.This question requires phrasing that will clearly relate the numbers 7 and 15 to each other, and correctly place the temporal marker to date.

A. The words of which at the beginning of the clause do not clearly refer to any antecedent.

Page 137: Economist

B. The words of which at the beginning of the clause do not clearly refer to any antecedent, and the parenthetical expression to date is awkward and unnecessary.

C. The word number in the first part of the clause is redundant, and the string of parenthetical expressions – 7, to date – makes the clause awkward and unclear.

D. The phrase to date is misplaced and seems to refer to these 15 rather than any appropriate temporal marker.

E. The phrase to date is correctly set off by itself, the phrase 7 of the 15 clearly positions the two numbers relative to each other.

The correct answer is EProponents of the Application Central website (ApCenWeb) claim that a single, standardized application valid for all graduate programs in the state will streamline the selection process by collecting student information into a single database. The proposed changes should not be made, however, because ApCenWeb is not yet flexible enough for students to present their strongest credentials to the varied programs specific to each university. A standardized application will, in fact, penalize students by making them unable to present their unique aptitudes for fields as divergent as engineering and visual arts.

Which of the following statements, if true, will most strengthen the argument against a standardized graduate school application?

The ApCenWeb interface is simple and straightforward, assuring that students interested in all programs will access the application easily.

Engineering and visual arts, though different from each other, are both popular graduate programs.

The board of directors of one of the largest state universities has expressed reluctance to join the Application Central organization.

ApCenWeb offers no option to submit additional materials, such as painting or photography samples for students applying to visual arts programs.

Page 138: Economist

The ApCenWeb software is still being developed, and programmers are concentrating especially on solving problems with uploading documents.Situation:An opponent of the proposed standardized online application for the state's graduate program is building an argument against its implementation.

Reasoning: Which point strengthens the argument against a standardized online application? The author of the passage acknowledges the goal of the website (to streamline the selection process), but offers a specific reason why the plan will not work: students do not have the flexibility to include materials specific to fields that have little in common, such as engineering and visual arts.

A. This is a potential argument in support of ApCenWeb

B. Discussion of the relative popularity of graduate programs does not directly address the benefits or shortcomings of ApCenWeb.

C. The reason for the board's reluctance is not included, and may have nothing to do with the author's argument regarding flexibility.

D. This is a specific example of the software's lack of flexibility preventing students from distinguishing themselves according to program. In this case, students applying to visual arts programs would be unable to submit samples of their work.

E. This option creates doubt in the trustworthiness of ApCenWeb software, but does not address the question of flexibility.

The correct answer is DBecause the time between the announcement of the program’s existence and its implementation was so short, many observers assumed that it had been hastily put together; in reality, however, it was based off of years of careful research.

in reality, however, it was based off of years of careful research.

in reality, however, it was based on careful years of research.

in reality, however, it was based on years of careful research.

however, in reality, it was based off of years of careful research.

Page 139: Economist

however, in reality, its existence was actually based on years of careful research.The original sentence contains the incorrect prepositional phrase “based off of” to refer to the time spent researching a new program. The words howeverand in reality are correct in either order, but the prepositional phrase must be based on in order to be correct.

A. Prepositional phrase based off of should be based on.

B. Careful should modify the word research, not years.

C. In this sentence, the prepositional phrase based on is used correctly, and careful correctly modifies research.

D. Prepositional phrase based off of should be based on .

E. Words its existence and actually are wordy and unnecessary.

The correct answer is C.Rene Descartes, a 16th century Frenchman often regarded as one of the founders of modern philosophy, penned one of its most famous slogans: “I think, therefore I am.” This statement was the conclusion to an approach of systematic doubt, the goal of which was to arrive at a new foundation for knowledge. He might have been able to doubt all the evidence of his senses, Descartes reasoned, but he could not doubt the proposition that he himself existed to do the doubting. His statement might almost be rephrased as: “I doubt, therefore I am.” Yet despite the fact that philosophers since the early 19th century have made heavy use of this conclusion, Descartes in his own day was most influential as a scientist, not a philosopher.

Throughout 16th century Europe, for example, Descartes was well known for his work in physics, geometry, and physiology, and modern scholars still consider him one of the precursors of the scientific revolution. His attempts to use algebra to solve problems of space and motion essentially created the field of analytic geometry, upon which modern applied mathematics is still largely based.Descartes discovered the law of refraction, which explains why straight objects appear bent when immersed partly in water, and was one of the first to theorize that the workings of human bodies can be studied in similar fashion to the workings of machines. Even his “Discourse on Method,” which contained the famous philosophical statement, was intended merely as the framework within which a collection of essays on optics, meteorology, and geometry was to be understood.

Page 140: Economist

Which of the following best describes the organization of the passage?

The career of a well-known historical figure is traced chronologically

Two opposing perspectives on a well-known historical figure are presented

Two fields of study are compared and contrasted

A common misconception in introduced, then put in wider context

A common misconception is introduced, then refutedThis question concerns the way the author has organized information in the passage. The first paragraph introduces Descartes as a well-known historical figure and explains that, because of a famous statement, he is most often associated with the field of philosophy. The second paragraph, however, argues that the idea of Descartes as primarily a philosopher is a misconception; details in the paragraph are meant to refute this by arguing for an alternate understanding of his importance.

A. Though certain time periods are mentioned, neither paragraph is primarily concerned with chronology.

B. The two perspectives are not merely presented; the author intends to use the second perspective to argue against the first.

C. The passage is concerned with Descartes in relation to each field of study, not the fields themselves.

D. The second paragraph is meant to argue an opposing perspective, not to give context.

E. A common misconception is introduced using details about its origin, and then is refuted using other details.

The correct answer is E.Information from the passage suggests that the author would most likely regard which of the following statements as INCORRECT?

Descartes’ contributions to the field of mathematics have largely been superseded.

Descartes’ work has greatly influenced philosophers since the early 19th century.

Page 141: Economist

Descartes’ curiosity about the natural world led him to explorations of many fields.

Descartes took a rational approach to the search for a new foundation of knowledge.

Descartes intended his scientific work and his philosophical work to be closely interrelated.The word suggests in the question indicates that the answer depends on making an inference. Additionally, the word incorrect in the question means that all of the answer choices will be correct EXCEPT ONE. To answer this question correctly, locate the lines or sections in the passage where the ideas present in each answer choice are discussed, and identify the only one with which the author would not agree.

A. The author does not believe that Descartes’ contributions to mathematics have been superseded, or outdated. See these lines.

B. The author does agree that Descartes’ work has had great impact on philosophers since the early 19th century. See these lines.

C. It can be inferred from the list of interests in paragraph 2 that Descartes was curious about the natural world, meaning that the author wouldagree with this statement.

D. This line  suggests that Descartes’ approach was rational, or based on reason, indicating that the author would agree with this statement.

E. This line  indicates that Descartes intended his philosophical work to serve as a framework for his scientific work, meaning that the author wouldagree with this statement.

The correct answer is AWhich of the following most accurately states the purpose of the passage?

To inform readers about the context in which Descartes’ famous statement was written

To review Descartes’ contributions to modern scientific fields

To offer an explanation for why Descartes’ philosophical works are more popular than his scientific ones

Page 142: Economist

To argue that Descartes should be considered more a scientist than a philosopher

To explain the relationship of analytic geometry to metaphysicsThis question requires an understanding of the passage as a whole, especially the author’s tone. The first paragraph introduces Descartes as an important historical figure most often associated with philosophy. The words yetand despite in the last sentence of the first paragraph indicate that the author disagrees with this association; the author also claims that Descartes is more properly understood as a scientist. The second paragraph provides details for this claim, and uses the language of argument to clarify the author’s position.

A. Context is provided only in the first paragraph, not the entire passage.

B. The words yet and despite in the last sentence of the first paragraph indicate that the author intends to draw a strong contrast, not merely to review.

C. The author is more concerned with general interests than specific works.

D. The contrast indicated by the words yet anddespite prepares the reader for the author’s conclusion in the second half of the sentence thatDescartes in his own day was most influential as a scientist, not a philosopher. This sentence begins an argumentative tone that continues in the second paragraph.

E. Outside the scope of the passage. This relationship is never discussed directly.

The correct answer is DIt can be inferred from the passage that Descartes came to be known as a philosopher rather than a scientist for which of the following reasons?

Because doubt is a very common problem for the modern individual

Because more of his philosophical than scientific writings have been preserved

Because of his popularity with philosophers since the early19th century

Because he himself considered philosophy to be more important than science

Because optics is no longer a significant scientific fieldThis question requires that a logical conclusion be drawn from information stated in the passage. Since the author discusses the misconception that Descartes is a

Page 143: Economist

philosopher when he should be known as a scientist, review the passage to discern what reasons the author may give for this.

A. The modern individual is not mentioned in the passage.

B. The passage does not mention which of Descartes’ works have been preserved.

C. In this line, the author implies that Descartes remains well-known as a philosopher today because of his popularity with philosophers in the early 19th century.

D. The passage does not discuss Descartes’ thoughts on the relative importance of the two fields.

E. The passage does not give information about the current state of optics as a scientific field.

The correct answer is CThe executive committee of the shopping mall announced yesterday it will partner with city police to add more officers to the mall security force for the holiday shopping season. The mall security team, which consists of 10 officers, was deemed too small to keep shoppers safe after there were 44 armed robberies in the parking lot during last year's season. At least two thirds of the officers will be patrolling the mall's north parking lot, which sees the most traffic from shoppers.

Which of the following, if true, is the best basis for criticizing the plan to deter crime by adding additional security officers?

Retailers expect this to be one of the busiest shopping seasons yet, drastically increasing the number of people in mall parking lots.

Police reports from the previous year show that most of the robberies occurred in the south parking lot.

Some officers in the north parking lot will be directing traffic rather than patrolling.

Security cameras have been installed on light posts in all parking lots, in the hopes of identifying armed robbers.

There are a large number of entrances and exits to the parking lots, making isolating suspect cars difficult.

Page 144: Economist

Situation: A shopping mall will increase its security force in the hopes of deterring parking lot robberies.

Reasoning: Which point most undermines the plan to deter crime in the mall parking lot? The executive committee plans not only to increase the number of security officers, but also to station most of the total force in the north parking lot, where there is the most traffic from shoppers. If, however, the north parking lot is not the most dangerous area around the mall, then the plan will most likely fail.

A. This statistic is relevant to the need for police protection for larger crowds, but does not address the soundness of the plan.

B. If the plan does not include increased police presence in the area that last year was most dangerous, then it most likely will not succeed.

C. The officers directing traffic will still constitute a police presence capable of deterring crime.

D. This point is an addition to the plan to add security officers and does not directly address the soundness of the plan.

E. This item may complicate the situation security officers must deal with, but it does not directly address whether the current plan is sound.

The correct answer is BAlthough there is a serious shortage of potable water in the area where the new sub-division is being built, the developer has contracted a company to deliver 400 gallons a day to assure that the workers remain fully hydrated.

Although there is a serious shortage of potable water

Because of a serious shortage of potable water

Because of a potable shortage of water

In the event of a serious shortage of potable water

Even though there is a serious shortage of potable waterIn this sentence, the first clause is dependent, and the second is independent. In order to choose the best introduction to the first part of the sentence, the correct relationship

Page 145: Economist

between the two clauses must be established. Here the first clause establishes the reasons for the action taken in the second clause; therefore, an introductory word demonstrating causation will best complete the sentence.

A. The word although does not make sense in this sentence because no contrast is being made.

B. The word because correctly demonstrates the relationship between the shortage of water and the need to have it delivered to workers.

C. The word potable is misplaced and modifies shortage rather than water.

D. In the event suggests a contingency plan and does not make sense in this sentence.

E. Even though draws an unnecessary contrast.

The correct answer is BPsychologists and marketers alike are aware that product labels can trigger brand loyalty and influence customers' perceptions of quality. A recent experiment, however, goes beyond brand loyalty and suggests that expectation can alter the experience of taste itself.

In the experiment, 300 men and women tasted two versions of a beverage each, one a regular cola, and the other the same cola with a few drops of white vinegar added. A previous survey had found that most cola drinkers thought vinegar would worsen the taste. The participants were told only that one of the beverages, called "cola extra," contained an unspecified secret ingredient.

Researchers found that in a blind taste test of 100 people, 60% actually preferred the vinegar-enriched "cola extra." A second group of 100 learned after they had tasted both beverages that vinegar was the secret ingredient. This group, surprisingly, preferred "cola extra" by the same percentage. A final group was told before the taste test that one of the colas contained vinegar. In this group, only about one third of respondents favored "cola extra.

The primary purpose of the passage is to

Critique the concept of brand loyalty.

Argue that taste preference is merely a matter of expectation.

Discuss an experiment studying the relationship of expectation to taste.

Page 146: Economist

Survey the taste preferences of participants.

Persuade readers to prefer cola with vinegar.To answer this question, look at the passage as a whole. The first paragraph introduces the results of a scientific experiment. The second paragraph details the method of the experiment, and the third paragraph discusses the results. Therefore, the overall purpose of the passage is to relay information about the experiment.

A. The concept is mentioned in the passage, but not critiqued.

B. Argue is too strong a word for the tone of the passage. Also, the word merely suggests a more strict correlation between expectation and taste than that discussed in the passage.

C. Each paragraph provides different information about the experiment's background, methods, and results.

D. The experiment concerns the influence of expectation on taste, not the original taste preferences of participants.

E. The passage does not seek to persuade.

The correct answer is C.The author mentions previous findings that "most cola drinkers thought vinegar would worsen the taste" primarily to

Offer interesting background to the most recent experiment.

Prove that all cola drinkers have taste preferences.

Contrast the negative expectation associated with vinegar and the neutral expectation of other taste tests.

Establish that taste testers were not expected to prefer the beverage containing vinegar.

Demonstrate the scientists' commitment to the project.To answer this question, examine the function of the phrase in the context of the passage as a whole. Since the experiment deals with taste preferences, mentioning the fact that participants were not expected to prefer vinegar establishes the related fact that they were also not expected to prefer the "cola extra" with vinegar additive. The fact that some of them did proves that expectation influences taste.

Page 147: Economist

A. The fact may be interesting, but it serves a more specific purpose in the passage.

B. This may be true, but does not directly address the question of how participants would react to the taste of vinegar.

C. The passage does not discuss other taste tests.

D. By establishing that participants were not expected to prefer the beverage containing vinegar, the passage is able to argue that expectation of a "secret ingredient" led some participants to prefer "cola extra."

E. The passage does not discuss the scientists' commitment.

The correct answer is D.Which of the following is the best sentence with which to begin a paragraph continuing the passage above?

The test group responses prove there may be a market for colas blended with vinegar.

The test group responses corroborate previous research with brain imaging, which had also shown that expectation can change the physical trace of neural activity.

This experiment illustrates the need for further research before the impact of expectation on taste can be stated conclusively.

This experiment establishes that labels indicating a "secret ingredient" will always sell more of a product.

This experiment offers useful information to marketers, who may adjust brand labels to indicate changes in product formula.Consider the intention and structure of the passage as a whole. The passage is concerned with discussing several aspects of the experiment with the influence of expectation on taste. The correct answer will continue in the same vein, and offer yet another piece of information for discussion.

A. The passage is not concerned with marketing, so a suggestion that cola with vinegar might be popular is misplaced.

B. Brain imaging scans are one more piece of information for discussion in the experiment.

Page 148: Economist

C. The passage is not designed to suggest further experiments, but to discuss information regarding this one.

D. Labeling is not the main concern of the passage, so will not logically be its extension.

E. The experiment deals with expectation and taste, not what is useful to marketers.

The correct answer is BIt can be inferred from the passage that which of the following, if true, would be most damaging to the conclusion that expectation influences taste?

A majority in all three test groups preferred standard cola to "cola extra," regardless of whether they were told it contained a secret ingredient.

The experiment was conducted again with vanilla as the secret ingredient and produced similar results.

Taste testers were told that a "bitter ingredient," rather than a "secret ingredient," had been added to the colas.

A previous survey had established that consumers thought vinegar would improve the taste of cola.

Both colas were given to taste testers in glasses that looked exactly alike.An inference is drawn from information stated in the passage. Since the experiment establishes a correlation between expectation and taste, the correct answer choice will disprove the correlation.

A. If members of all three test groups prefer the cola without vinegar, then the experiment has proven that expectation has no influence on taste.

B. Expectation would still be a factor in the experiment, though the preference for cola with vanilla might not be as surprising.

C. This version of the experiment would ask participants to identify a bitter flavor, not test the correlation between expectation and taste.

D. The experiment might yield higher percentages of participants preferring cola extra, but the taste test would still reveal a correlation between expectation and taste.

E. The appearance of glasses is irrelevant to an experiment testing taste.

Page 149: Economist

The correct answer is AA small orchard specializing in citrus fruits has been receiving fewer orders for oranges in the off season. Orchard owners expect demand to pick up in the coming weeks as harvest begins, but they are concerned about a neighboring orchard nearly three times their size. The neighboring orchard has announced plans to sell cases of oranges for 20% less than regular price in order to jump-start the season by attracting new business.

Which of the following, if true, points to the most serious flaw of the small orchard's plan to attract business by matching the prices of the competing orchard?

The small orchard has a dozen customers under contract to purchase the first oranges of the season and may anger them by not extending the discount to them.

The small orchard does not have time to advertise its oranges at lower prices.

The small orchard will reduce the number of oranges in each case to minimize losses incurred by matching prices.

The large orchard has a larger inventory and is more able to take losses on initially discounted oranges.

The large orchard will target a different market than the small orchard.Situation: A small orchard desires to compete with a larger neighboring orchard.

Reasoning: Which point reveals the most serious flaw in the small orchard's plan? It can be determined from the passage that the large orchard has three times the inventory of the small orchard, and that they are looking to attract new business. It can also be inferred from the passage that the owners of the large orchard are confident that they are able to sell oranges at a discount and still remain solvent.

A. The concern of the orchards is in attracting new business, not satisfying existing customers. Additionally, customers under contract have likely already agreed to a non-discount price.

B. This item may be problematic, but does not address the concerns of the passage as a whole.

Page 150: Economist

C. This approach is not a flaw in the plan, but a possible, potentially unethical, solution.

D. The large orchard's greater inventory will give them a flexibility that will outlast the small orchard. The small orchard could run out of stock before the large orchard has sold even a third of its oranges.

E. This item makes it sound as if the two orchards will not actually be in competition, but does not address the soundness of the small orchard's plan.

The correct answer is DWhich of the following best completes the passage below?

Two popular local restaurants, The Cancan Club and Chez Shay, use two different systems of valet parking. At the Cancan Club, valet parking is both optional and complimentary, and there is some parking around the restaurant. At Chez Shay, however, customers must pay $10 per car for valet parking; though this service is optional, there is no parking within 3 blocks of the restaurant. Because the Chez Shay valet lot is the busier of the two, one can reasonably conclude that Chez Shay customers.

Are wealthier than customers at The Cancan Club

Also use valet parking when they dine at The Cancan Club

Are not deterred from dining at Chez Shay by the expense of valet parking

Prefer Chez Shay to The Cancan Club

Prefer valet parking to self-parkingSituation The more expensive valet lot is busier than the less expensive.

ReasoningWhat conclusion about Chez Shay customers can be reached given information in the passage? According to the passage, both local restaurants are popular, and both offer optional valet parking. Chez Shay customers, however, must pay to have their cars parked by a valet, and will not find convenient parking around the restaurant. However, since the Chez Shay lot is busier than The Cancan Club lot, it can be concluded that the up-front expense of valet parking at Chez Shay does not deter customers from dining there.

Page 151: Economist

A. This may be true, but the passage does not mention the relative wealth of customers.

B. This may be true, but can not be stated conclusively based only on information in the passage.

C. Since the Chez Shay lot is busier than The Cancan Club lot, it can be concluded that customers do not avoid Chez Shay merely to bypass the expense of having their car valet parked.

D. This may be true, but the passage does not mention which restaurant is more popular, nor does it discuss the personal taste of customers.

E. This may be true, but the passage does not directly compare rates of valet and self-parking.

The correct answer is CAlthough the company’s founders certainly hoped that incorporating would increase their profit margin, no one, not even the founders, didn’t expect that the value of the stock would appreciate so quickly.

no one, not even the founders, didn’t expect

no one, not even the founders, expected

nobody, not even the founders, expected

nobody, not even the founders, didn’t expect

no one, even the founders, expectedA. No one...didn’t expect is a double negative and is incorrect

B. No one...expected is a grammatically correct phrase; not even is necessary to emphasize what the founders did not expect.

C. Nobody is incorrect.

D. Didn’t expect creates a double negative.

Page 152: Economist

E. The parenthetical phrase must be not even the founders to emphasize that the founders are included in the group of people who had expectations for the stock.

The correct answer is BThis information is potentially harmful to the company and is meant to stay only between a small group of investors.

between a small group of investors.

between a group of small investors.

among a small group of investors.

among a smaller than usual group of investors.

within the knowledge of a small group of investors.This question requires choosing the correct preposition to describe information that must be known only to a small group. Among is the only correct choice for a preposition.

A. The word between refers to two people only and inadequately refers to the group of investors.

B. Small is misplaced and incorrectly modifies investors, rather than group.

C. In this choice, the preposition among correctly describes the location of the information respective to a small group of investors.

D. The phrase smaller than usual group is wordy and not necessary in the context of the sentence.

E. Within expresses the idea of a group, but among is the only correct option for the sentence.

The correct answer is CThe state legislature has proposed a law that would require registered voters to bring photo ID with them to polling places. The law is intended to curb potential fraud by preventing anyone else from voting under a registered voter's name. A 2004 survey found that the names of over 1,500 people who had died were still on voter rolls, and

Page 153: Economist

another 672 people were still listed as registered voters even though they had either moved out of the state or lost the right to vote due to felony conviction.

Which of the following, if true, is the best piece of evidence to support the adoption of the proposed law?

A law from a neighboring state requiring citizens to show photo ID before they can register to vote, eliminating the need for them to bring ID to polling places.

Data from the latest census showing that only two-thirds of the people in the state are registered voters.

Evidence from a survey demonstrating that a percentage of minorities living in cities do not have photo ID.

Proof that absentee ballots from the last election had been misplaced.

A survey showing that over the five years previous to the survey votes were recorded for a percentage of people who had died or moved out of state.Situation: A proposed law would require people to bring photo ID to polling places when they vote, in order to curb potential fraud.

Reasoning: Which point supports the making of a law requiring people to confirm their identities before voting? Identify carefully that claim the passage is making: that the many names of people no longer eligible to vote remaining on the rolls is an indication that voter fraud is possible. Evidence that fraud has already occurred is the best support for the law's adoption.

A. The fact that a neighboring state simply has a law to counter a similar problem does not prove that the law presents an adequate solution.

B. The fact that   of the state's population are not registered to vote does not solve a problem with the other two thirds.

C. This is evidence against the proposed law, because it would demonstrate that some registered voters do not have the required accompanying photo ID.

D. This item includes irrelevant information because it does not directly address the question of whether people are voting under other names.

Page 154: Economist

E. This piece of evidence illustrates that the state has an actual, rather than potential, problem with voter fraud that requires action detailed in the proposed law.

The correct answer is EIn order to keep the competitive edge gained by our extensive research and innovative methods, information security must remain a high priority for our company. The information stored on our networks including everything from research data, to training materials, to company memos - could potentially be very valuable to competitors and foreign governments alike. All employees must work to protect company information, not only through passwords and data encryption, but also through the physical control of company assets, such as laptops.

If all of the above statements are true, which of the following statements can be most directly inferred?

Passwords are often inadequate forms of security, leading to the need for "physical control" over data storage objects such as laptops.

Employees are not allowed to discuss their research with anyone other than fellow company employees.

Company information carries a high monetary value, making the network attractive to hackers.

The company is willing to go to great expense to protect its information.

Employees must attend classes to learn to use the latest security software.Situation: A company prioritizes information security in order to protect its ability to compete.

Reasoning:What inference can be drawn from this information? The theme running through the entire passage is the importance of maintaining secrecy. The first and second sentences contain reasons for secrecy, and the third an insistence that employees use a variety of means to maintain it. The correct answer, then, will be an extension of the argument for secrecy.

A. Passwords and physical control are both methods recommended for maintaining secrecy, but one is not prioritized over the other in the passage.

Page 155: Economist

B. This sentence is a logical extension of the company's policies about information security. Not discussing research with anyone other than company employees is another method of maintaining secrecy.

C. Though the passage mentions the value of company information, it does not specify that the value is monetary, nor is the passage specific about fears of attracting hackers.

D. The passage does not discuss real or potential costs for maintaining information security.

E. The passage concentrates on the importance of information security, rather than the training of employees to maintain it.

The correct answer is BWhich of the following best completes the passage?

A study released by a major medical center has found that people experiencing stroke are frequently not treated in time to prevent major complications, such as brain damage and paralysis, because they do not recognize the symptoms, which are similar to symptoms of heart attack but may pass more quickly. Additionally, since stroke damages the brain, many people are not able to recognize the symptoms themselves and must depend on the quick reactions of those around them. Doctors affiliated with the study hope that a stroke education program will  

Explain the treatments available for stroke, so that victims and their families can make informed decisions.

Encourage members of the public to see stroke as a serious threat to their health

Shed light on the relationship between heart attack and stroke symptoms, since one is often confused with the other.

Raise awareness of stroke symptoms, so that victims and their families will recognize them and seek treatment more quickly.

Inform the public about risk factors for strokes, hopefully preventing a number of medical emergencies.

Page 156: Economist

Situation: A study has found that stroke victims frequently do not receive treatment in a timely manner.

Reasoning: Which point best states the goal of a stroke education program? The study found, not only that people do not receive treatment in a timely manner, but that they are delayed because they or those around them do not recognize their symptoms. The main concern of an education program, then, would be to help people recognize stroke symptoms so they can receive treatment quickly.

A. This item is outside the scope of the passage, which does not discuss treatments for stroke.

B. This concern is implied, but does not reach to the wider purpose of the passage.

C. The two illnesses may share symptoms, but the passage is not concerned with this.

D. This sentence correctly identifies the passage's concern with recognizing symptoms and having them treated in a timely manner in order to prevent major complications.

E. This item is outside the scope of the passage, which does not discuss risk factors for stroke.

The correct answer is DAccording to a recent publication by the city council, the most cost-effective solution to the problem of congestion on State Highway 1 is to build an elevated roadway above the existing six lanes. This solution makes the most sense because it will save the expense of surveying and breaking new ground for an alternate route, and will also mean that crews will need to construct half as many lanes, since the current lanes will remain in use after the elevated portion is completed.

The conclusion in the passage above is based on which of the following assumptions?

That the city council has already approved funding for an elevated roadway above the existing highway.

That the cost of surveying and breaking new ground for an alternate route is greater than the cost of constructing an elevated roadway.

Page 157: Economist

That the large number of people traveling State Highway 1 every day make easing congestion a priority for the city council.

That the city council can not afford to fund the construction of more than six lanes, and so seeks a solution that will require building as few lanes as possible.

That drivers on State Highway 1 are unwilling to pay a toll to use the road, and so the city council must provide all funds for improvement.Situation A city council has recommended an elevated roadway as the most cost-effective solution to a local traffic problem.

ReasoningWhat assumption is present in the argument supporting this recommendation? The city council believes that surveying and breaking new ground would be the most expensive option, and that constructing new lanes will be more expensive than modifying existing ones. If it could be shown, however, that an elevated roadway will be more expensive than surveying and breaking new ground, the argument would fall apart.

A. The passage does not say that the council has already approved money for the project.

B. The city council’s publication presents the elevated roadway plan as a way to avoid the expense of surveying and breaking new ground. The assumption is that those costs would be prohibitive, and that the construction of an elevated roadway would be less expensive.

C. While priority for the project is implied, the mere fact of its importance does not affect which solution the council recommends.

D. The passage does not mention explicitly whether or not it can afford to construct a certain number of lanes.

E. This is outside the scope of the passage, since the question of tolls is never raised directly.

The correct answer is BThe state legislature has proposed a law that would require registered voters to bring photo ID with them to polling places. The law is intended to curb potential fraud by preventing anyone else from voting under a registered voter's name. A 2004 survey found that the names of over 1,500 people who had died were still on voter rolls, and

Page 158: Economist

another 672 people were still listed as registered voters even though they had either moved out of the state or lost the right to vote due to felony conviction.

Which of the following is most like the passage in logical structure?

A local bank begins requiring additional identification before allowing access to safety deposit boxes. This decision is made after a series of robberies.

An elementary school institutes a rule requiring anyone picking up children after school to be issued a tag that will hang in their car, proving that they are either the child's parent or someone approved by the parents to transport the child. This decision is made after teachers notice children riding home in different cars on successive days.

A gym requires proof that patrons live within a 10-mile radius of the facility before issuing membership. This decision is made after the number of patrons increases beyond the gym's capacity.

An online auction site requires users to answer a "secret question" before completing checkout with a credit card. This decision is made after the site is forced to return money to patrons whose accounts were charged with unauthorized purchases after their credit card numbers were stolen.

Callers into a talk radio show are asked to identify themselves by first name and town of residence. This decision is made after the show's host becomes curious about the locations of his callers.Situation: A proposed law would require people to bring photo ID to polling places when they vote to curb potential fraud.

Reasoning: Which alternate situation is most like the attempt to deter potential fraud described in the passage? The study described in the passage reveals a weakness in record-keeping that could potentially allow someone to vote under the name of a person who should no longer be voting in the state. By requiring voters to prove their identities, the proposed law will apparently concentrate on stopping potential fraud before it starts.

A. The bank's decision involves a form of identification, but it is made only after a certain problem has already occurred.

Page 159: Economist

B. The elementary school's rule about proper identification is issued in response to the perception of a potential problem children being picked up by drivers other than their parents.

C. The gym's decision is made for reasons of inadequate space, not to prevent harm to a person or system.

D. The auction site's decision is made in response to theft that has already occurred.

E. In this example identification has the meaning of self-identification rather than proof. Additionally, the decision is made out of curiosity rather than caution.

The correct answer is BScientists have several rival theories on the causes of fibromyalgia, a disorder that causes body-wide chronic pain and fatigue. One leading theory holds that conditions such as depression, anxiety, drug use and serotonin deficiency can aggravate or even cause fibromyalgia by interfering with "stage 4," or "deep sleep." This theory is concerned with the function of neuropeptide substance P, which is released in the spinal cord in response to pain and causes nerve endings around the initiating nerves to also become more sensitive to pain. Normally, this mechanism is "reset" during deep sleep. If pain becomes body-wide, however, and the mechanism can not be reset, this process may run out of control.

The theory above on the cause of fibromyalgia rests on which of the following assumptions?

The body has a limited amount of control over neuropeptide substance P.

Fibromyalgia is a completely preventable disorder, if only other conditions are recognized and treated before it can develop.

Deep sleep is necessary to the healthy functioning of the nervous system.

The functions of neuropeptides released by the spinal cord must be better understood before a cure for fibromyalgia can be found.

Page 160: Economist

Because the causes of fibromyalgia are linked to depression, the disorder should be treated by mental health professionals.Situation: Doctors theorize that fibromyalgia is the result of a disruption of healthy sleep patterns.

Reasoning: Which assumption must be true in order to argue that fibromyalgia is caused by a disruption of sleep patterns? The passage describes a mechanism in the nervous system that, when disrupted, may aggravate or cause fibromyalgia. Other conditions depression, anxiety, etc. are mentioned as factors that may interfere with the root cause of fibromyalgia: the disruption of pain receptors normally reset during deep sleep.

A. A limited focus on neuropeptide substance P does not address the concerns of the passage as a whole.

B. Outside the scope. The passage does not argue whether or not fibromyalgia is preventable.

C. If deep sleep is not achieved, the substance P process can not reset, and fibromyalgia may result.

D. Outside the scope. The passage is concerned with the causes of fibromyalgia, not its cures.

E. The passage lists several conditions that can aggravate or cause fibromyalgia, but does not prioritize depression as the condition most linked to its onset.

The correct answer is CA recent article in a literary journal argues that having a career separate from publishing makes writers more likely to produce large volumes of work. The article uses Anton Chekhov, a 19th century Russian writer who was also a physician, as the flagship example, tracing the roots of recurrent themes in his stories and plays to his experience in medicine. Anyone who writes, the article claims, will find his or her productivity vastly stimulated by work in another field.

Which of the following would provide the best piece of evidence for the claim made in the article?

An in-depth biography of a single writer, such as Chekhov, tracing the influence of a previous career on eventual literary output.

Page 161: Economist

An interview with a panel of literary experts making the same claim.

A survey of the output of 100 writers, identifying those with highest output as the same ones with previous or separate careers.

A cross-reference of best seller lists from the last 20 years with a list of which of those writers had previous or separate careers.

An article by a psychologist discussing the emotional and intellectual factors involved in writing well.Situation: The article argues that writers with separate careers produce a larger volume of work.

Reasoning: Which piece of evidence best supports the argument that writers with separate careers have a larger output? The article mentioned in the passage asserts a statistical correlation between the amount of work a writer produces and whether that writer has a separate career. Search the answer choices for the option that will best establish concretely the correlation asserted in the article.

A. The biography would give more information about Chekhov, but would not provide insight about other writers in general.

B. The panel of experts would be able to make a similar claim, but the mere existence of the panel would provide no evidence.

C. A survey identifying the writers with separate careers as the ones who also wrote the most would establish a statistical correlation able to support the article's argument.

D. Though this item would establish a statistical correlation, it would be based on the popularity of an author's work (determined by sales), rather than sheer volume of output.

E. The psychologist's observations would not offer a comparison of output volume with separate career.

The correct answer is C.The entire community is hoping on the mayor to improve the water drain-off system.

hoping on the mayor to improve

Page 162: Economist

hoping of the mayor to improve

hoping that the mayor will be able to significantly improve

hoping that the mayor will improve

full of hope in the mayor’s ability to improveThis question tests the correct usage of the verb to hope. One can hope that, hope to, hope for, or have hope in.

A. The phrase hoping on is incorrect idiomatic usage.

B. Hoping of is incorrect usage.

C. Though the verb form is correct, the words be able to are wordy. Significantly is redundant and unnecessary.

D. In this sentence hoping that is the correct usage of the verb to hope. The rest of the sentence is concise and appropriate.

E. The phrase full of hope in the mayor’s ability, though technically correct, is wordy and unnecessary.

The correct answer is DIn most industries the rising cost of support services actually passes a greater financial burden to customers; in the legal profession, for example, a current climb in the cost of court reporting has forced many attorneys to raise their hourly rates.

has forced many attorneys to raise their hourly rates.

has made many attorneys raise their hourly rates.

has meant that many attorneys are being forced to raise their hourly rates.

is forcing many attorneys to raise their hourly rates.

is meaning that many attorneys are raising their hourly rates by force.A. The past tense has forced does not take into account the fact that the rising

costs are a current trend.

Page 163: Economist

B. Has made incorrectly uses the past tense; the phrase is also both informal and imprecise.

C. Has meant incorrectly uses the past tense. Has meant and are being forced use inconsistent verb tenses, and are unnecessary phrases.

D. The phrase is forcing correctly uses the present progressive tense, reflecting a current trend.

E. By force is a misplaced modifier that confuses the meaning of the sentence; is meaning is an imprecise phrase.

The correct answer is DThe Dear One Baby Carrier is the safest and most comfortable baby carrier available today. The hard plastic exterior retains its shape despite minor impacts, protecting baby from the outside, while the thick interior “comfort pad” acts as a shock absorber inside. The interior design has been comfort-tested by thousands of infants, and the exterior handles have been designed with the arms and hands of caring parents in mind. Perhaps best of all, the Dear One Baby Carrier converts easily from an at-large carrier to a car seat, reducing the risk to baby posed by being shifted from one carrier to another.

Which of the following, if true, would most undermine the claim to safety made on behalf of the Dear One Baby Carrier?

An infant safety commission announces that most dropping accidents involving infants occur when a parent is the primary caretaker.

A survey conducted by a popular magazine reveals that a large number of parents are still not aware of the danger posed to infants by putting their carriers in the front seat.

A parents’ group endorses the Dear One Baby Carrier as the most convenient baby carrier to take on vacation.

A manufacturer adds a “danger of suffocation” label to the bedding used for the interior of the Dear One Baby Carrier.

Page 164: Economist

An article written by a pediatrician states that at-large baby carriers do not safely double as car seats.Situation The manufacturer of a brand of baby carrier claims that it is the safest on the market.

Reasoning Which specific claims are made by the manufacturer, and how could they be undermined? The paragraph claims to protect infants in three ways: with a hard plastic exterior, with interior padding, and by reducing the need to transfer the infant from one type of carrier to another. First eliminate options that do not deal directly with one of these claims, and then test remaining answer choices to see which will undo one of the major claims.

A. The passage does not deal with accidents involving infants in general.

B. The passage does not compare the relative merits of front- and back-seat placement for the carrier.

C. The manufacturer’s claim deals with safety, not convenience.

D. Though this option deals with a safety concern, the concern remains between manufacturers, not between the public and the manufacturer of the Dear One, since the padding will likely change forms before being added to the baby carrier.

E. This option directly undermines the manufacturer’s claim that infants are safer remaining in the carrier than being transferred from carrier to car seat.

The correct answer is EOur understanding was that she would continue to cover the costs of school enrollment and room and board for the children, which she verbally agreed to pay with the rest of the family.

which she verbally agreed to pay with the rest of the family.

which she agreed verbally with the rest of the family to pay.

of which she agreed verbally with the rest of the family to pay.

both of which she verbally agreed to pay with the rest of the family.

Page 165: Economist

both of which she agreed to pay with the rest of the family verbally.This sentence requires the correct placement of the adverb verbally, indicating the way in which the agreement was made, and the correct use of phrases indicating with whom (the rest of the family) the agreement was made.

A. The phrase with the rest of the family is misplaced, making it sound as if the subject of the sentence and the rest of the family were planning to pay the fees together.

B. The adverb verbally comes immediately after the verb agreed, indicating their correlation, and the placement of the phrase with the rest of the family makes it clear that the agreement was between the two parties.

C. The word of is unnecessary and confusing.

D. The phrase with the rest of the family is misplaced, making it sound as if the subject of the sentence and the family were planning to pay the fees together; also the word both is unnecessary and confusing.

E. Verbally is misplaced, making it sound as if the promised payments will be made by speaking.

The correct answer is B.New coal-burning power plants are making moderate strides in environmental responsibility. Plants using an integrated gasification combined cycle (IGCC) utilize two distinct thermodynamic cycles, making them up to 15% more efficient than traditional coal-burning plants. In the first cycle, raw coal is converted into a synthetic gas, filtered to remove pollutants such as sulfur and nitrous oxide, and burned at high pressure to produce half the plant's electricity. Exhaust gases from the first cycle are captured and used to heat water, which turns a second turbine to generate another 125 megawatts. Newer plants are equipped with scrubbers to capture carbon dioxide and other greenhouse gas emissions, and older plants can be retrofitted to do the same.

Since the IGCC system yields more energy per ton of coal and is also capable of capturing harmful emissions, constructing more coal-burning plants using this technology seems to be an energy solution to satisfy both industrial and environmental groups. However, "cleaner burning" coal plants are actually short-sighted, inadequate solutions to our current energy crisis because they ignore two grave environmental dangers.

First, though coal may seem cheap economically, it is painfully expensive environmentally, as mining practices deplete land beyond recovery. In some eastern

Page 166: Economist

states, for example, mining companies have leveled entire mountains in their quest for coal. Experts estimate that strip-mining of sources in Appalachian forests is contaminating ground water and causing erosion that will eventually destroy several irreplaceable species.

Additionally, though carbon dioxide is not released into the atmosphere by IGCC plants, it remains a by-product, which scientists must either develop a way to process or, as in an ill-considered current trend, find a place to store. A team investigating storage possibilities in a series of caverns under the Arctic Ocean floor estimates that the caverns are large enough to store the planet's carbon dioxide emissions for the next 600 years; so far they have found no evidence that the rock is leaking. If, however, the stored carbon dioxide leaks after only 50 years, the damage to the environment will be greater than if coal plants had continued operating with no emissions-capturing provisions

The primary purpose of the passage as a whole is to

Account for the popularity of coal-burning power plants.

Argue that coal-burning power plants may cause more environmental problems than they solve.

Explain the dual thermodynamic cycles of IGCC coal plants.

Plead for more efficient methods of reducing carbon emissions.

Evaluate the economic feasibility of constructing more coal-burning power plants.This question requires a look at the passage as a wholein order to determine the author's purpose in writing. The first paragraph introduces the idea that IGCC coal plants are more energy efficient than traditional coal plants and explains how the process works. The word however, though, signals that the author of the passage is changing the direction of the argument. Paragraphs 3 and 4 discuss the grave environmental dangers of IGCC plants, making it clear that the author's main concern is pointing out potential problems.

A. The passage does not discuss the popularity of coal-burning power plants.

B. The passage begins with an explanation of what seems a possible environmental solution, then argues that what seems to be a solution may actually cause different kinds of problems.

Page 167: Economist

C. The first paragraph does explain the thermodynamic cycles of IGCC plants, but the explanation is not the focus of the passage as a whole.

D. The passage assumes the need for methods of reducing carbon emissions, but does not take up the argument directly.

E. The passage does not discuss the cost, oreconomic feasibility, of constructing more coal-burning power plants.

The correct answer is BThe passage above most likely appeared first in

The national section of a syndicated newspaper.

A journal devoted to new technologies.

A personal weblog.

A newsletter published by an environmental group.

A newsletter published by a coal plant.The question's use of the words most likely means that the answer depends on making an inference. Because the question does not reference a specific line or paragraph, the inference must be drawn from the passage as a whole. Phrases such as painfully expensive environmentally,in their quest for coal, and ill-considered current trendindicate that the author's sympathies lie with the environment.

A. Though the passage references the United States, its clear environmental bias makes it inappropriate for the news section of a syndicated newspaper.

B. The first paragraph explains IGCC technology, but the argument in the third and fourth paragraphs veers from the merely scientific.

C. Though it is possible the article appeared on a personal weblog, its journalistic tone means it is more likely it appeared in a publication of a group.

D. The argumentative tone in the second half of the passage makes this passage most likely to have been published in a newsletter explicitly concerned with environmental issues.

E. The passage actually questions the environmental soundness of coal-burning plants.

The correct answer is DThe author most likely uses the word "seems" in this line in order to

Page 168: Economist

Draw attention to an obvious solution.

Highlight the need for agreement among environmental and industrial groups.

Suggest that an apparent solution can not be trusted.

Make it clear that environmental experts must have the final say in the matter.

Avoid overstating the case for IGCC coal plants.To answer this question, look at the phrase seems in the context of the passage. The phrase occurs at the beginning of the second paragraph and signals a transition between the introduction and main argument of the passage. The first paragraph discusses the apparent efficiency of IGCC plants, but the third and fourth discuss the two grave environmental dangers the author believes coal-burning plants present.

A. The passage is questioning the soundness of IGCC plants as a solution.

B. The passage does not focus on agreement among environmental and industrial groups.

C. Since IGCC plants burn coal more efficiently, the passage suggests, they seem like a good solution to an environmental problem. The third and fourth paragraphs, however, argue that the plants will perpetuate other problems.

D. The passage does not discuss environmental experts.

E. The passage ultimately does not build a case at all for IGCC plants.

The correct answer is CIt can be inferred that the author details the workings of IGCC power plants in paragraph 1 primarily in order to  

Explain a more efficient coal-burning process, in order to create consensus among environmental and industrial groups.

Form a bridge between the first and fourth paragraphs, which both discuss the containment of carbon dioxide emissions.

Suggest that coal-burning power plants could be even more efficient if they integrated three thermodynamic cycles instead of two.

Page 169: Economist

Demonstrate the availability of new technologies that can be put in the service of environmental responsibility.

Acknowledge the reasons for viewing IGCC plants as a solution, before demonstrating that the environmental problems of coal-burning plants outweigh the solution of burning coal more efficiently.To answer this question, consider the role of the first paragraph in the passage as a whole. The first paragraph is the background to the argument constructed in paragraphs 2 and 3 that IGCC plants do not account for other environmental dangers. The word seems in the last sentence of paragraph 1 signals that the author believes the IGCC solution would be reasonable if not for other environmental factors.

A. The passage explains the process, but does is not specifically aimed at creating consensus.

B. Both paragraphs discuss carbon dioxide emissions, but for opposite purposes.

C. This is not discussed in the passage.

D. The passage does not directly address the availability of IGCC technology.

E. The phrase making moderate strides indicates the author's belief that IGCC plants are an improvement over traditional plants. The third and fourth paragraphs, however, state clearly that the author does not consider the moderateimprovement to be adequate.

The correct answer is E.

Gmat1

The new approach to poetry that emerged between 1900 and 1920 sought to put aside the sentimentality and strict obedience to established forms that had characterized the Victorian poets of preceding decades.Many poets in both the United States and Europe had begun to experiment with free verse – poetry that does not adhere to set standards of rhyme or rhythm – and several magazines were founded in order to publish this “new poetry.” It was not until 1912, however, that the first real school of new poetry emerged from the chaos of experimentation.

With only a handful of adherents, but access to the editorial department of a key American literary magazine, Ezra Pound created and sustained Imagism by mere declaration. Pound, an American poet living in London, had at first open lines of communication with a small group of English poets who combined deep appreciation for Japanese verse forms, such as haiku, with experiments in free verse.

Page 170: Economist

Although Pound was briefly aligned with this group, he soon found greater artistic affinity with Richard Aldington, a fledgling English poet, and Hilda Doolittle, who signed her poems “H.D.” One of H.D.’s first poems, “Hermes of the Ways,” so delighted Pound with its crystal clear images and lack of ornamentation that he assigned her the penname “H.D., Imagist” and sent the poem immediately to the U.S. for publication in the magazine Poetry, for which he was serving as foreign editor.

In a later issue of Poetry, Pound articulated the guiding principles of Imagism, stressing that poets should treat objects directly, use no words that do not contribute to the presentation, and write according to the rhythm of the words rather than an accepted poetic form. In addition to his work with literary periodicals, Pound also edited an anthology of Imagist poems in 1914. Ironically, however, his insistence that no explanation of the Imagist movement be offered in the anthology backfired; readers did not understand that Pound and his Imagists were experimenting with a new form of poetry, and many of the anthologies were returned to the publisher.

The primary purpose of the passage is to

convince the reader of the significance of a literary figure

explain the failure of an experimental type of poetry

trace the development of a literary movement

debate the merits of traditional forms of poetry

introduce important contributors to a literary movementThis question concerns the focus of the passage as a whole. The first paragraph provides background on Victorian and early twentieth century poetry and introduces Imagism as the first real school of new poetry. The remaining three paragraphs recount the sequence of events that introduced Imagism to the public through the influence of Ezra Pound. The author focuses on the contributors to and development of Imagism as a movement.

A. Though the passage mentions several significant literary figures, this is not the main purpose of the passage as a whole.

B. The passage does not state that Imagism failed as an experimental type of poetry.

C. The dates cited by the passage, as well as the details of a step-by-step development, are intended to trace the development of Imagism.

Page 171: Economist

D. The merits of traditional forms of poetry are not mentioned in the passage.

E. The passage mentions several literary figures, but this is not the primary purpose of the passage.

The correct answer is CEACH of the following is cited in the passage as a goal of Imagism EXCEPT

Challenge accepted standards for poetry

Promote new and talented poets

Concentrate on objects over ornate speech

Experiment with free verse

Praise Japanese verse formsThis question requires the selection of the one item in the answer choices that is not explicitly cited in the passage as a goal of the Imagist movement.

A. Imagism was part of the movement of new poetry that sought to put aside…strict obedience to established forms.

B. Pound’s excitement over new poets Richard Aldington and HD prompted him to send HD’s poem for publication as promotion of a distinctly Imagist style, which is mentioned here.

C. This answer choice combines two of Pound’sthree guiding principles for Imagism: poets shouldtreat objects directly and use no words that do not contribute to the presentation.

D. Imagism was part of the larger trend of new poetry that had begun to experiment with free verse.

E. Japanese verse forms are only mentioned here.

The correct answer is E.Many poets in both the United States and Europe had begun to experiment with free verse – poetry that does not adhere to set standards of rhyme or rhythm – and several magazines were founded in order to publish this “new poetry.” It was not until 1912, however, that the first real school of new poetry emerged from the chaos of experimentation.

The highlighted passage most clearly implies which of the following about new poetry movements before 1912?

Page 172: Economist

Although many poets had begun to experiment independently with free verse, there were no clearly defined movements before Imagism.

Movements adhered strictly to the established forms that had characterized Victorian poets.

Although many poets had begun to experiment with free verse, none was successful enough to achieve lasting recognition.

The sheer number of new poetry movements in the United States and Europe created chaos.

Many movements founded magazines in which to publish their experiments with free verse.To answer this question, review the context of the passage. The first paragraph describes an emerging desire in the early years of the twentieth century to break from poetic tradition. Poets began to work with free verse and, according to the passage, each poet’s different attempts to break with tradition resulted in the chaos of experimentation. In 1912, however, Imagism began to emerge as the first official movement of the new poetry.

A. According to the passage, Imagism was the first clearly defined movement in new poetry.

B. This is not true of new poetry, which, in general, sought to break from Victorian standards.

C. Recognition of individual poets is not addressed in the passage.

D. The chaos mentioned in the passage was caused by individual experimentation, not the number of movements.

E. The founding of magazines and experimentation with free verse were simultaneousdevelopments; the passage does not address whether movements founded magazines.

The correct answer is A.Which of the following, if added to the end of the last paragraph, forms the best conclusion to the passage?

Because of this misunderstanding, Imagism failed to gain the recognition as a valid literary movement that it needed to survive.

Page 173: Economist

Despite this setback, however, Pound’s articulation of the principles of Imagism had succeeded in presenting a viable alternative to traditional forms, and modern poetry set off in a new direction.

Angered by this rejection, Pound adjusted his ambitions for Imagism and began encouraging fellow poets to write according to the rhythm of words, rather than accepted forms.

On the other hand, even though most of the public did not understand the goals of Imagism, Pound’s anthology sold several thousand copies.

Additionally, several readers wrote letters to the editors of the magazine criticizing Pound and the Imagists for slavishly imitating traditional poetic forms.The last sentence in the first paragraph makes it clear that the passage is mainly concerned with presenting Imagism as the first distinct school practicing “new poetry,” and the conclusion of the passage is likely to return to this idea. Additionally, the best conclusion to the paragraph will comment on the final idea in the original passage, specifically, that Imagism as a movement had to overcome the challenge of being misunderstood by many in the public.

A. This directly contradicts the first paragraph, which states that Imagism did in fact gain recognition as a school of poetry.

B. This option continues the line of reasoning in the final paragraph and returns to the passage’s main concern with Imagism as an important movement in the development of modern poetry.

C. Pound insisted that poets write according to the rhythm of words before the anthology was published.

D. On the other hand does not make sense as a transition, because the paragraph does not suggest a contrast between public understanding and sales.

E. Traditional poetic forms were challenged by Imagism, not imitated.

The correct answer is B.Which of the following is cited in the passage as a historical reason that Ezra Pound played such a decisive role in the development of Imagism?

He clearly defined the principles of Imagism in the pages of an influential literary magazine.

Page 174: Economist

He himself was a successful poet and was able to help others follow his lead.

He was delighted by HD’s use of images and lack of ornamentation.

He was the foreign editor for an important American poetry magazine.

He capitalized on an emerging taste for free verse in the United States and Europe.A specific line or lines detailing a historical reason for Pound’s role must be identified in order to answer this question correctly. The passage states here that Pound created and sustained Imagism by mere declaration. In other words, his advocacy of Imagism was responsible for its development. The correct answer choice will identify the factor that made such direct and powerful influence possible.

A. Pound’s role in the development of Imagism began before he published its principles in Poetry magazine.

B. Pound’s success as a poet is not directly addressed in the passage.

C. Though this is true, it is not cited as a reason Pound’s role in the development of Imagism was so decisive.

D. These lines  state that, although Imagism originally had only a handful of adherents, Pound’s access to the editorial department of a key American literary magazine allowed him to exert great influence on the development of the movement.

E. The passage states that a taste for free verse was emerging among poets, not necessarily the public.

The correct answer is D.Because city council members in small towns are intimately acquainted with the public they serve, they clearly have a more difficult job than city council members in larger cities.

The conclusion above rests on which of the following assumptions?

Working directly with members of the public is the one of the most challenging aspects of serving on a city council.

Managing the affairs of a large city requires a more significant time commitment than working directly with the public.

Page 175: Economist

City council members in small towns are often related to a certain percentage of the population, making them intimately acquainted with the people they serve.

Residents of small towns often attend city council meetings to present their concerns.

City council members in large cities rarely engage the public directly.Situation: City council members in small towns have a difficult job because they are intimately acquainted with the public.

Reasoning:Which identifies the assumption made in the passage? The claim that city council members in small town have a more difficult job than council members in larger cities is linked directly to the author’s unstated definition of the word difficult, which is found in the first clause of the sentence and is signaled by the word because. Difficult means being more intimately acquainted with the public.

A. The author of the passage considers a council member’s job in a small town to be more challenging than a council member’s job in a city, because there is the necessity of working more directly with the public.

B. The passage does not address the time commitment required.

C. This fact can also be true in larger cities, where even a single relative accounts for a certain percentage of the population.

D. Information about residents attending city council meetings is not provided in the passage.

E. This claim is not made in the passage.

The correct answer is ADuring a single hour of a pledge drive for a public radio station, anyone making a pledge of a stated amount was given a free gift. Pledges were encouraged by the announcement that the retail cost of the gift was equal to the amount of the pledge. Yet, at the end of the hour, the total money raised from pledges accounted for a larger dollar amount than the amount organizers had paid for all the free gifts.

Page 176: Economist

Which of the following, if true, is the best explanation for the fact that funds raised from pledges during the hour accounted for more money than the cost of the free gifts?

The cost of postage was included in the total cost assigned to the gifts, making them seem more expensive to potential donors.

Organizers underestimated the amount of money that would be raised during the hour and were surprised by the actual total of pledges.

Organizers overestimated the number of donors who would respond to the offer and were forced to offer gifts at half price when there were fewer pledges than expected.

Free gifts were donated by a sponsor, eliminating the need to subtract the cost of them from the total money raised through pledges.

More money was raised during this hour than during the previous three hours, driving down the average out-of-pocket cost of the free gifts.Situation: Organizers of a public radio fund drive find that they have received more money in pledges than they must pay for free gifts they send to donors.

Reasoning:Which is the best explanation for the excess in money? Because the retail cost of the gift and the amount each donor’s pledge are assumed to be equal, it is expected that organizers will not see a net gain in funds during this hour. It is important to note, however, that the amounts of money being compared are the pledge amount and the retail cost of the free gifts. If organizers do not spend money on the free gifts they send to donors, all of the pledges received during the hour represent a net gain.

A. Whether the gifts seem more expensive has no impact on the net amount raised during the hour.

B. The organizers’ surprise may influence their perceptions of the amount of money raised, but would not influence an actual count.

C. Free gifts are not being sold in the passage, and therefore can not be offered at half price.

Page 177: Economist

D. If the cost of the gifts need not be subtracted from the dollar amount raised during the hour, all pledges count as net gain and represent a larger dollar amount than the total retail price of all gifts.

E. The amount of money raised during previous hours has no bearing on the situation described in the passage.

The correct answer is D.Although genetic recombination ensures that only half the genetic material of a parent is represented in the offspring, some genetic lines may eventually die out entirely, even though there are numerous progeny.

Although

Conclusively

When

Because

IfThe underlined word must establish the proper logical relationship between the first part of the sentence and the later clauses. The facts among which a relationship must be stated are, first, that genetic recombination permits only half of a parent’s genetic material to be represented in the offspring; and, second, that some genetic lines may eventually not be passed on, despite a large number of progeny. If only half of a parent’s genetic material is present in any single offspring, it follows that some genetic lines may not be reproduced at all and could eventually die out entirely. The relationship that should be stated in the sentence is, therefore, a causal relationship.

A. Although makes the dying out of some genetic lines seem like an exception to a rule, and does not establish the appropriate cause-and-effect relationship between the parts of the sentence.

B. Conclusively does not make sense in this context.

C. The word when makes genetic recombination seem like a process that happens sometimes, and so does not establish the appropriate logical relationship among parts of the sentence.

Page 178: Economist

D. This option correctly identifies the fact that genetic recombination is given as the reason that some genetic lines may eventually die out entirely.

E. If makes genetic recombination seem like a conditional event and does not establish the appropriate logical relationship among parts of the sentence.

The correct answer is DAll business owners invited to bid for the lucrative construction project realize that, in order to secure the contract, they must have presented an exceptionally well proposal.

have presented an exceptionally well proposal

have presented their proposal exceptionally well

present an exceptionally well proposal

present an exceptionally good proposal

present a good proposal exceptionallyThe underlined portion of the sentence requires adjustment in two areas: verb form and diction in terms of the word well. The words invited in the first clause and the phrase in order to in the second indicate that the third clause of the sentence must be in future tense, not past perfect as in the question stem. Also, the word well in the question stem is an incorrectly used adverb, when the sentence calls for the adjective good to modify the noun proposal.

A. Well is an adverb and incorrectly modifies proposal. Also, must have presented is in past perfect tense, when the sentence calls for future tense.

B. This option uses well as an adverb that modifies exceptionally, but the meaning of the original sentence has been changed. Also, the first two clauses of the sentence indicate that future tense is necessary for the verb.

C. Must present identifies changes that should be made to the verb in the original sentence, but well incorrectly modifies proposal.

D. This option makes needed changes to both verb form and the word well as a modifier. They must present is correct future tense, and the adverbwell has been changed to the correct good, and adjective that modifies proposal.

Page 179: Economist

E. This option changes and confuses the meaning of the sentence by changing exceptionally from an adjective modifying good proposal to an adverb modifying present.

The correct answer is DWhich of the following best completes the passage below?

A new children’s book by a prominent invertebrate expert features detailed profiles and photographs of the thousands of bee species common to North America, most of which, children are often surprised to learn, are comprised of solitary insects. The author hopes not only to expand recognition of diversity in the invertebrate world, but also to.  

teach children about different kinds of bees.

surprise other invertebrate experts with previously undiscovered information.

convince the public that bee species should be studied in greater detail.

counter the common misconception that all bees are social insects.

detail what makes bees unique among other insects.Situation: A children’s book about bees gives information on species in North America.

Reasoning:Which best completes the passage? The goals mentioned in the second sentence are connected to the selection of information contained in the book. Aside from statistics and photographs, the passage informs us, the book also contains information that surprises children: most bee species arecomprised of solitary insects. The author of the book’s goal, then, is to expand knowledge of the invertebrate world in a way that is surprising because it challenges what children think they know about bees.

A. This is understood by the passage as a whole, and would be redundant as a completion to the passage.

B. Children, not other invertebrate experts, are the intended audience.

C. The passage does not make this claim.

Page 180: Economist

D. Since children are often surprised to learn that bees can be solitary insects, it follows that one of the author of the book’s goals is to challenge the common misconception that all bees are social insects.

E. Other insects are not mentioned in the passage.

The correct answer is DThe university library offers most of the resources Ronald will need. Except for Spanish translations of certain books, which he will have to request from a neighboring university.

need. Except for Spanish translations of certain books, which he

need, except for Spanish translations of certain books, which he

need, accept for Spanish translations of certain books, which he

need, with the exception of certain Spanish translations of books which he

need, but there are few books in Spanish which heThe second sentence is incomplete and must corrected by the addition of a comma and appropriate subordinating conjunction.

A. Except is a subordinating conjunction that makes the second sentence incomplete.

B. Except works properly as a subordinating conjunction here, joining the fact that Ronald has access to most of the resources he will need to the fact that Spanish translations will not be available.

C. Accept sounds like except, but is not correct.

D. The phrase certain Spanish translations of books is wordy and unnecessary.

E. The conjunction but in this option changes the meaning of the sentence, as does the adjusted phrase there are few books in Spanish which he.

The correct answer is BControversial mortgage laws passed last year state that, if at any point during the maturation period of the loan the person in whose name it is taken is not able to meet

Page 181: Economist

the payments, that person’s spouse will have responsibilities that include paying the balance.

will have responsibilities that include

will be responsible to pay

is responsible to pay

will be responsible for paying

is responsible for payingThis sentence is confusing because it contains many prepositional phrases nestled within each other. At the core, however, the sentence is testing the ability to recognize the conditional tense created by the word if that begins the series of phrases after the comma; this if causes the underlined portion to need future tense in order to agree. The sentence also tests the correct use of the idiomatic verb to be responsible for.

A. This phrase is wordy and unnecessary.

B. The preposition for should be used with the –ing form of the verb to pay

C. If in the first phrase after the comma creates a conditional tense that requires a future-tense form in the underlined portion.

D. This option correctly uses the preposition for and the –ing form of the verb to pay, as well as the future tense signaled by the conditional word if.

E. The word if in the sentence signals a conditional tense that requires a future-tense form in the underlined portion.

The correct answer is DA cultural study examining smiling as a submissive gesture that signals lower social status examined photographs in 200 newspapers and magazines and found that, the more wealthy or influential a person was, the less likely that person was to smile for the photographer. Researchers conclude that advertisements for high-end products will use unsmiling models to convey status and exclusivity.

The conclusion that models in advertisements for high-end products will not smile rests on which of the following assumptions?

Page 182: Economist

Unsmiling models adequately lend status to the products, even if the models are not wealthy or influential themselves.

Advertisements in which the models do not smile will not be as successful as advertisements in which they do.

Models in the advertisements for high-end products will have the same high social status as the people toward whom the advertisements are directed.

Advertisements will run in the same newspapers and magazines used for the survey.

Advertisements featuring a mix of smiling and unsmiling models will be most successful in selling mid-priced products.Situation: Following the results of a survey on smiling and social status, researchers predict that print ads will use unsmiling models to make their products more attractive to the wealthy and powerful.

Reasoning:Which assumption underlies the researchers’ conclusion that ads for high-end products will use models who do not smile? The survey used published photographs to determine that wealthy and influential people are less likely to smile because smiling is a submissive gesture that signals lower social status. Advertisements for products directed toward the wealthy and influential, the survey concludes, will use models who do not smile as a signal that the products themselves are suited for those with high status.

A. If the lack of a smile is a sufficient signal of high social status, then models who mimic this gesture will lend status to a product, regardless of whether they themselves are wealthy or powerful.

B. This is the opposite of the conclusion reached in the passage.

C. Since the focus of the passage is on the power of the gesture of not smiling, the actual social status of the models is irrelevant to the success of the advertisements.

D. This assumption is not made in the passage.

E. The passage is not concerned with selling mid-priced products.

Page 183: Economist

The correct answer is A> Tests > v01

F lag for Review

Chec k   Answer  Sh o w Answer  E x plain Answer

P revious  N ext

H elp  E nd Exam  R eview Section

Most people who run for an elected office do so because they want to be of service to the public. If, however, an official publishes a book revealing information that had not previously been made public and profits from the publication, that person’s political career has inarguably come to an end because he or she will never again be trusted by other members of the government.

The conclusion that a public official who publishes a book has ended his or her political career rests on which of the following assumptions?

Most government officials seek office in order to serve the public.

Public officials must often keep information secret.

Trust is essential for a successful career as a public official.

Members of the government frequently do not trust authors of books.

Public officials will seek to profit from their access to privileged information if given the opportunity.Situation: The author argues that public officials who publish books containing information not previously made public end their careers because other members of the government will no longer trust them.

Reasoning:Which identifies the assumption necessary to the author’s conclusion? The first sentence introduces the idea of running for elected office as a way of serving the public. A contrast to this idea appears in the second sentence, which states that public officials sometimes publish books for their own profit. This publishing for profit undermines confidence and trust, since colleagues expect the official’s first priority to be serving the public. Without this trust, the passage argues, the official is unlikely to further progress in his or her career.

A. This is stated directly in the passage and can not be the underlying assumption.

Page 184: Economist

B. Though this option is tempting, information...not previously made public is not necessarily secret.

C. According to the passage, a public official’s career ends when actions taken for personal profit undermine the trust placed in them by their colleagues.

D. This claim is not made in the passage.

E. Though the author discusses possible consequences when this does occur, the passage does not make the claim about most public officials.

The correct answer is CThe farmer is planning to reinforce the windows on the eastern side of his barn, even though most wind storms develop to the west and approach toward the opposite directions.

approach toward the opposite directions

approach toward the opposite direction

often approach from opposite directions

approach from the opposite direction

approach from opposite directionsThe meaning of this sentence is unclear as it is written, and so the correct answer will be a clarification. The sentence require that a contrast be made between east and west: storms usually approach from the west, so it is strange that the farmer is reinforcing his eastern windows. From, then, is the correct preposition to use with approach when referring to a specific direction. Also opposite direction should be singular because there is only one directionfrom which the storms will approach.

A. Toward is not the correct preposition to use with approach, and the plural directions creates confusion in the sentence.

B. Though direction is correctly singular, toward is not the correct preposition to use with approach.

Page 185: Economist

C. This option makes it sound as if storms approach from both east and west simultaneously, and does not satisfy the contrast between east and west required by the sentence. Also, the word often is unnecessary.

D. This option correctly identifies from as the preposition to be used with approach, and also correctly makes direction singular.

E. This option makes it sound as if storms approach from both east and west simultaneously, changing the intended meaning of the sentence.

The correct answer is DThe constellation that includes the North Star has been known by many names among different cultures, called “The Bear” in ancient Greece, “The Drinking Gourd” in parts of Africa, and “Star Girl and Her Seven Sky Brothers” among the Cheyenne people of North America.

among different cultures, called

among different cultures; it was called

in different cultures, being called

in different cultures; including the title

in different cultures; it was calledThe sentence tests the idiomatic version of the preposition in, which should be used to refer to different cultures. Among is not appropriate here, because the sentence is concerned with the use of names inside each culture. Secondly, the creation of a grammatically correct sentence using a semicolon is being tested.

A. Among would indicate terms common to each culture and is not appropriate in this sentence. Also, the sentence as written is incomplete.

B. Though the sentence is complete, among does not correctly modify different cultures.

C. This option creates a complete sentence, but is unnecessarily wordy.

D. Both a semicolon and conjunction are used in this option, making the sentence incomplete.

Page 186: Economist

E. This option correctly uses the preposition in to modify different cultures and creates a complete sentence by inserting a semicolon.

The correct answer is ECandidate for governor: The outgoing governor and his administration have made a mess of our state’s tax laws. Although they have offered necessary tax cuts to small business, they have so complicated and increased personal income taxes that it is almost better financially to be unemployed than to own a business.

Incumbent governor: On the contrary, the changes to the tax code my administration and I have made have undoubtedly brought new business to our state, thereby increasing overall prosperity. In fact, last year alone three major new manufacturing plants began operations.

The incumbent governor’s response to the candidate’s criticism is flawed because it  .

assumes that new manufacturing plants will contribute significantly to revenues in the state.

does not assign a dollar amount to the relationship between tax incentives and the attraction of new business.

fails to address the candidate’s concern about the tax burden placed on the individual.

denies the candidate’s accusation without providing evidence for an alternative conclusion.

perpetuates the unfounded assumption that the state will prosper if new business is encouraged.Situation: A candidate for governor accuses the outgoing governor and his administration of making mistakes with the state’s tax laws. The outgoing governor denies that his administration has made mistakes, and he argues that his administration has rather brought increased prosperity to the state.

Reasoning:Which represents the flaw in the incumbent governor’s response? The candidate’s complaint is that personal income taxes have both become more complicated and increased by an unacceptable percentage. While acknowledging tax cuts offered to small business, the candidate apparently does not consider that these

Page 187: Economist

advantages outweigh the disadvantages of the administration’s policies on income tax. The governor’s response, however, focuses on prosperity generated from new business and does not directly engage the candidate’s criticism about personal income taxes.

A. The incumbent’s argument actually shares this assumption with the candidate’s argument.

B. While this is true, it is not a flaw in the governor’s argument, because the candidate has not specified a dollar amount himself, nor has he asked the governor to do so.

C. The candidate’s main criticism of the governor’s tax policies is that they have complicated and increased personal income taxes to the extent that unemployment may be preferable.

D. On the contrary, the governor’s response does provide evidence: the attraction to the state of three new manufacturing plants.

E. This option is similar to option (A); in fact, both the candidate and the governor share the assumption that new business should be encouraged for the overall prosperity of the state.

The correct answer is C.Hyperkalemia is a potentially fatal condition found most often in kidney dialysis patients that causes pain, severely erratic pulse, and tissue swelling throughout the body. Hyperkalemia is the result of extremely high levels of potassium in the blood and, in an emergency, can be treated with an injection of calcium chloride. However, since calcium chloride destroys surrounding tissues if it escapes from a vein, doctors overwhelmingly prefer preventative treatments.

Which of the following can most logically be concluded based on the passage above?

Potassium is poisonous except in very small amounts.

Since the symptoms of hyperkalemia resemble those of heart attack, the condition is often misdiagnosed.

Calcium supplements are the preferred preventative treatment for hyperkalemia.

Page 188: Economist

The kidneys are at least partially responsible for removing excess potassium from the bloodstream.

Most cases of hyperkalemia result in permanent tissue damage.Situation: Hyperkalemia is a potentially fatal condition resulting from extremely high levels of potassium in the bloodstream.

Reasoning:Which conclusion can be drawn from the passage? The passage lists several facts concerning hyperkalemia, or high potassium, including options for both treatment and prevention. The correct answer will be a conclusion drawn directly from one of these stated facts.

A. According to the passage, potassium is fatal when it reaches extremely high levels.

B. Misdiagnosis is not addressed in the passage.

C. The passage does not give specifics of preventative treatments.

D. Since hyperkalemia is most often found in kidney dialysis patients and is the result of high levels of potassium, it is reasonable to conclude that the kidneys play a role in eliminating potassium from the bloodstream.

E. Though tissue damage is possible, the passage dos not claim that it is an outcome of most cases.

The correct answer is DEven in places in which the average temperature is as warm as Florida, snowfall is sometimes possible.

places in which the average temperature is as warm as

a place with an average temperature as warm as

a place as warm as

places as warm as it is in

a place that is as warm a

Page 189: Economist

This sentence is testing mainly concision, but the wordiness in the original sentence also causes a misplaced modifier, making it sound as if temperature is intended as a direct comparison to Florida. The correct answer will be the most concise version of the underlined portion that keeps the sense of the sentence and eliminates ambiguity.

A. This phrase is unnecessarily wordy, and temperature is incorrectly paralleled to Florida.

B. Temperature is incorrectly parallel to Florida, and the phrase is wordy.

C. This option both provides a more concise rewording of the original underlined portion and eliminates ambiguity, making warm an appropriate modifier for Florida.

D. Ambiguity has been eliminated, but the phrase as warm as it is in is unnecessarily wordy.

E. The words that is are redundant and unnecessary.

The correct answer is CA major film studio announced the release date of a movie based on a novel that, though it was a bestseller when first published, has been out of print for nearly fifteen years. Hoping to capitalize on the anticipated success of the film, the publisher who owns the copyright on the novel plans to print a new edition to be made available the same week the film premieres.

EACH of the following, if true, supports the soundness of the publisher’s plan to capitalize on the success of the film EXCEPT

The publisher has received permission from the film studio to stamp the words “Now a major motion picture” on the cover of each book.

Last year a new edition of a novel that had been out of print hit the bestseller lists two weeks after a movie biography of its author was released.

The publisher received two hundred thousand dollars after selling the production rights to the film studio.

The actress playing the lead in the film has discussed her love for the novel in nationally televised interviews.

Page 190: Economist

Last year a new edition of an unpopular novel was adapted into a top-grossing film, and sales of the book spiked.Situation: A publisher hopes a film based on an out-of-print novel will prompt sales of a new edition of that novel.

Reasoning:Which one does not support the soundness of the publisher’s plan? The publisher hopes the film will renew demand for the out-of-print novel and plans to supply a new edition to meet that demand. The soundness of this plan can be supported by evidence that films have previously created demand for the novels on which they are based, and also by any agreements the publisher has already made with the film studio. Opinions of participants in film production, however, have no direct bearing on the soundness of the plan.

A. The publisher has made an agreement with the film studio that links the movie with the book, thereby supporting the soundness of the plan.

B. This piece of evidence supports the soundness of the plan by proving that the link the publisher intends to make between book and film has previously been successful.

C. The fact that the publisher has already received a form of profit from the link of novel and movie supports the soundness of the plan.

D. Although the actress’s high opinion of the novel may add interest in the eyes of the public, it has no direct bearing on the soundness of the publisher’s plan.

E. Proof that a popular film can increase sales of the book on which it is based supports the soundness of the publisher’s plan

The correct answer is DAs part of continuing efforts to control illegal immigration, the US government should institute a system of internal passports similar to those in European nations. Drivers’ licenses currently serve a kind of internal-passport function, but this system is severely limited because all states do not require the same documentation before issuing photo ID. Illegal workers can get a driver’s license in one state with lax regulations and use it as identification in another state with stricter regulations but better-paying jobs. Internal passports will keep illegal workers from slipping through the cracks by standardizing forms of photo ID.

Which of the following is most similar to the passage above in logical structure?

Page 191: Economist

National standardized college entrance exams replace exams written independently by test administration services affiliated with groups of universities.

A national association of doctors approves standards for diagnosis and treatment of childhood development disorders, and these standards are used in private clinics across the country.

Resale values of used cars are discussed, standardized, and published in an industry manual also available to independent buyers and sellers.

The distance between rails of train tracks is standardized across the country, allowing trains built by any manufacturer to travel along any rail line.

Employment applications and hiring processes for a national restaurant chain are standardized in order to ensure uniform quality among employees.Situation: A proposal is made to introduce a system of internal passports to the United States to prevent illegal workers from falsifying identity records.

Reasoning:Which is most like the passage in logical structure? The first sentence introduces the idea of an internal passport system in the US, and the second states the reason that the current system of using drivers’ licenses as photo ID: not all states require the same documentation. The author of the passage worries that illegal workers will use photo IDs gained by taking advantage of lax documentation rules in one state to gain employment in other states. The overall logical structure of the passage has to do with standardizing regulations in order to make sure those given access to certain resources, whether jobs or college admission, are properly certified.

A. College entrance exams that are both national and standardized will unify a certification process in the same way that a system of internal passports would. In the passage, workers can gain employment with ID received in another state, and in this answer choice students can gain admission to a college with scores administered by another university.

B. A standardized list for diagnosis and treatment will not permit the transfer of credentials described in the passage.

Page 192: Economist

C. Even though an industry manual will make it easy for buyers to find the costs and resale values of used cars, this option does not use the wordstandardized in the same way as the passage.

D. This will actually make movement across boundary lines easier, not more difficult, as is the goal of the argument in the passage.

E. Employment is a topic discussed in the passage, but uniform standards in a national chain will make it simpler for employees to change jobs, not more difficult.

The correct answer is AWhich of the following best completes the passage below?

A preliminary survey of online networking sites revealed that over 2,000 female users under age 18 are members of pro-anorexia discussion communities. This survey may overestimate the number of girls diagnosed with anorexia, however, since  ________.

many girls who are very thin could not be medically diagnosed as anorexic.

a large number of people with anorexia do not join online communities.

the survey was conducted according to a keyword search that included only the titles of registered communities, not their actual discussions.

doctors are unable to agree about the symptoms that constitute anorexia.

community members need not be diagnosed with anorexia in order to join.Situation: A survey reveals that over 2,000 girls are members of pro-anorexia discussion communities online.

Reasoning:How could the survey overestimate the number of girls with anorexia? The number quoted in the passage refers to the number ofmembers of pro-anorexia discussion communities. Since membership in a discussion community is the only criteria for being counted in the survey, it is possible that some community members are not themselves diagnosed anorexics.

A. This may be true, but it has nothing to do with the count taken by the survey.

Page 193: Economist

B. The survey was concerned only with those girls who are members of online discussion communities.

C. This option actually hints that the survey may have underestimated the number of girls diagnosed with anorexia by suggesting that it was not sufficiently thorough.

D. This option does not directly address whether the survey results accurately reflect the number of girls who have been diagnosed with anorexia.

E. This option correctly identifies the possibility that the number of girls who are members of pro-anorexia discussion communities may be larger than the number of girls who actually have anorexia, since a diagnosis of anorexia is not necessary for membership in the community.

The correct answer is EThe announced relocation of the hospital has been welcome news for the city, where economic growth has been stagnant for the past several years.

city, where economic growth has been stagnant

city whose economic growth has been stagnant

city where economic growth has been stagnated

city, whose economic growth as been stagnated

city, the site of stagnated economic growthThe proper use of the words where and whose is being tested in this sentence. Whose refers to a being capable of making a decision, whereas whererefers to a place. The sentence is also testing the correct use of a comma to join a subordinate clause to an independent clause.

A. The sentence correctly uses the word where to modify city, and also properly joins a subordinate clause to an independent clause using a comma.

B. This option lacks a comma and creates a run-on sentence; it also misuses the word whose.

C. A comma is missing in this sentence, and a run-on sentence has been created.

Page 194: Economist

D. This option misuses the word whose to modify city.

E. The phrase the site of stagnated economic growth is wordy and confusing.

The correct answer is AA government watch group has publicly accused members of the city’s waterworks board of wasting taxpayer money by accepting large salaries for weekly meetings that rarely last more than an hour. In response to this criticism, the president of the waterworks board has offered to reduce the number of meetings per month.

Which of the following, if true, represents the most significant flaw in the president’s proposed method of saving taxpayer money by reducing the number of board meetings per month?

Members of the board are routinely able to complete all necessary business within an hour.

Members of the board receive a monthly salary, regardless of how many times per month they meet.

Board members’ salaries are already less than the salaries of waterworks board members in neighboring cities.

Water is relatively scarce in the area, and water-related matters require frequent attention.

Board members’ salaries are determined by the city council, not by the board members themselves.Situation: The president of the waterworks board has offered to reduce the number of board meetings per month as a response to public accusations that the board is wasting taxpayer money on members’ salaries.

Reasoning:Which is the largest flaw in the president’s proposal? The government watch group has accused the waterworks board of wasting tax payer money, specifically by receiving large salaries for what seems a relatively small amount of work. The president of the board has offered to reduce the number of board meetings per month. If, however, a change in the number of meetings will not reduce the amount of money spent on board members’ salaries, then the president’s plan will not result in a net savings for taxpayers.

Page 195: Economist

A. This item is irrelevant to the president’s plan.

B. This item correctly identifies the flaw in the president’s argument: if fewer meetings a month will not lessen the salaries of board members, then the proposed change will result in no net savings of taxpayer money.

C. The size of board members’ salaries relative to neighboring cities does not have direct bearing on the president’s plan.

D. Though this may suggest a need for more frequent waterworks board meetings, it is not the most serious flaw in the president’s plan.

E. The president’s plan has to do with reducing the number of meetings – not directly reducing the salaries of board members. It is possible within the limits of the passage that the city council could determine a pay-per-meeting rate, while allowing the board members themselves to determine the frequency of the meetings.

The correct answer is BAlthough the researchers agreed unanimously on their expectations for the outcome of the experiment, their surprise was evident when the actual data started coming in.

their surprise was evident when

they were evidently surprised when

their surprise was in evidence when

they were evidently surprised as

they were surprised asThis sentence tests the use of the idiomatic phrase surprised when, which indicates surprise at a particular point in time, rather than surprised as, which is a comparison. The underlined portion of the sentence is also a passive construction that should be made active in order to emphasize the reactions of the scientists.

A. This option uses when correctly, but is a passive construction and not the best revision of the sentence.

Page 196: Economist

B. This option uses the idiomatic phrase surprised when correctly and creates an active construction by changing surprise from a noun to an adjective and evident to the adverb evidently.

C. In evidence is awkward and unnecessary.

D. The word as is incorrect in the idiomatic use of the word surprise.

E. The word as suggests a process rather than a specific point in time and is not correct in this sentence.

The correct answer is BMicrolending as a form of foreign aid first became popular in the 1970s as a way to bypass bureaucracy and administration costs that frequently, though unintentionally, prevented money from reaching individuals and families in struggling countries. In contrast to traditional lending, which tenders large sums to lendees who have strong credit histories and steady employment, microloans are generally made for less than $1,000 and are available without collateral to individuals with questionable credit histories who may or may not be employed.The central qualification for approving a microloan recipient is that the individual have a clearly defined plan for a small business, whether that be a bakery, dairy, tailor shop, or retail store. Recipients are bound to use profits from their business to repay the loan, and lenders since the inception of microloan programs have reported surprisingly high returns on their investment: up to 96% of microloans are repaid on time.

Though there are several administrative options for microloan programs, one of the earliest has remained the most common. According to this approach, a branch of an established bank or a bank specially formed to issue microloans will locate in an area of need and begin issuing loans to local entrepreneurs. In the early years of microloan programs, banks frequently set up village committees, composed of financial advisors and bank staff, to host weekly progress meetings. This proved a difficult administrative strategy to maintain, however, when villagers began to default on their loans just to avoid the meetings and what they often perceived as interference in their businesses. Though most banks quickly revised this approach when they realized its negative potential, the trust vacuum created when they could not offer a return to investors led many banks to seek other forms of administration

According to the passage, which of the following best describes the contrast in qualifications between traditional lending and microloans?

Recipients of traditional loans must complete a rigorous evaluation process, whereas recipients of microloans need only suggest an idea.

Page 197: Economist

Traditional lending qualifies candidates who are already financially independent, whereas microlending prioritizes potential.

Recipients of microloans must prove the viability of small businesses, while traditional loan recipients are responsible for predicting the success of corporations.

Traditional lending is structured so that the lender receives the bulk of the benefit, whereas microlending benefits the recipient.

Microloans are made to qualified individuals who plan to start a business, whereas traditional loans are made to businesses already in operation.The information needed to answer this question can be found in these lines. According to the author, traditional lending is available to those who have strong credit histories and steady employment, or, in other words, are financially independent. These qualifications do not hold for recipients of microloans, however, who usually have questionable credit histories and are not necessarily employed. These recipients qualify for loan funds by presenting a clearly defined business plan that testifies to their potential.

A. The application processes for each type of loan are not detailed in the passage.

B. According to the passage, traditional loans are made to recipients with strong credit histories and steady employment, whereas recipients of microloans qualify for funding on the basis of a clearly defined business plan.

C. A contrast between small businesses and corporations is not made in the passage.

D. This answer choice actually addresses the benefits, not the qualifications, of each type of lending.

E. The passage does not state that recipients of traditional loans must own a business.

The correct answer is BIt can be inferred from the passage that the author makes which of the following assumptions about traditional lending in the form of foreign aid?

Traditional lenders are more interested in earning profit than in giving assistance.

Leaders of struggling countries are dissatisfied with the approach of traditional lending.

Page 198: Economist

Traditional lending is not flexible enough to loan amounts less than $1,000.

A complicated administration process often limited the money available to individuals and families.

Traditional lending is outdated and will eventually be replaced by microloan programs.To answer this question correctly, it is necessary to make an inference. Traditional lending is discussed in the first paragraph, and it is here that the author claims that, when it came to foreign aid, bureaucracy and administration costs frequently…prevented money from reaching individuals and families in struggling countries. Given this statement, it is reasonable to assume that the author believes that administration costs and protocols often limit the amount of actual aid that reaches individuals.

A. The passage does not directly discuss whether traditional lenders are interested in earning profit, and so this assumption can not underlie the discussion of foreign aid.

B. Opinions of leaders of struggling countries are not discussed in the passage.

C. It is stated in the passage that microloan amounts are often less than $1,000, but the relative flexibility of traditional lending is not discussed.

D. The first sentence of the passage identifies this as the very problem that led to the creation of microloan programs.

E. The passage neither discusses relevance of traditional lending, nor speculates that one form of lending will eventually replace another.

The correct answer is D. This proved a difficult administrative strategy to maintain, however, when villagers

began to default on their loans just to avoid the meetings and what they often perceived as interference in their businesses. Though most banks quickly revised this approach when they realized its negative potential, the trust vacuum created when

they could not offer a return to investors led many banks to seek other forms of administration.

The author suggests that the rise of the “other forms of administration” mentioned in the highlighted text was due primarily to  

input from villagers gained during weekly progress meetings.

Page 199: Economist

disappointment in the ability of the village committee approach to insure repayment of loans.

a desire to provide villagers with more direct feedback than that available during weekly progress meetings.

the perception that weekly progress meetings were interfering in the businesses of villagers.

the development of trust between villagers and bank staff, making weekly progress meetings unnecessary.Because a specific line is referenced in the question stem, it is clear that the correct answer to this question depends on an appropriate understanding of supporting details. First, review for context. The passage describes a situation in which villagers reacted negatively to the weekly progress meetings required by the banks who issued their loans. This reaction led to a climb in the default rate for the loans and, in turn, caused confidence in the banks to plummet. The other forms of administration sought to regain the trust of investors caused by the failure of the village committee approach.

A. According to the passage, villagers did not give input about the administration of their loans.

B. The committee approach was intended to secure repayment of the loans, but, because many villagers felt the banks were interfering with their business, they defaulted on their loans. This led many banks to seek other forms of administration in order to regain the trust of investors.

C. Actually, the villagers considered that the banks were interfering in their businesses and demandedless direct feedback.

D. This was the perception of the villagers, not the banks who sought other forms of administration.

E. The passage does not state that trust developed between villagers and bank staff.

The correct answer is BThe cost of the El Paso tour includes admission to two museums, a play, and two other cultural attractions, but anyone wishing to cross the border should bring their passport.

but anyone wishing to cross the border should bring their passport

Page 200: Economist

and everyone who wishes to cross the Mexican border should be bringing their passport

but anyone wishing to cross the border should bring his or her passport

and everyone wishing to cross the border should bring his or her passport

to such an extent that anyone who wishes to cross the border should bring his or her passportThe word anyone is a singular antecedent requiring singular pronouns, his or her specifically, in order to maintain rules of agreement. The sentence also includes an implied contrast between the cultural events in El Paso and potential unpaid-for events across the border. This contrast requires the conjunctionbut.

A. The pronoun their requires a plural antecedent, not the singular anyone.

B. This option is more wordy than the original, and does not correct the fundamental disagreement between anyone and their. Additionally, the conjunction and does not show the necessary contrast between El Paso events and events across the border.

C. This option clarifies pronoun-antecedent agreement so that singular words his or her correctly modify singular antecedent anyone, and the conjunction but supplies appropriate contrast between parts of the sentence.

D. This option is more wordy than the original, does not correct the fundamental disagreement between anyone and their, and does not show appropriate contrast between parts of the sentence.

E. His or her correctly modifies the singular antecedent anyone, but this option is unnecessarily wordy.

The correct answer is CScientists studying climate change have found that one of the overall effects of global warming is not only a higher planetary temperature, but also a universal change in seasons, specifically earlier springs and later autumns. Though some argue that this is an advantage because it presents a longer summer growing season, others fear that too many animal species will be unable to adapt and will become extinct.

Page 201: Economist

Which of the following is the best piece of evidence to support the conclusion that a longer growing season will hasten the extinction of some animal species?

Squirrel and chipmunk species are unable either to consume or to store all the nuts and berries produced during a longer growing season.

Fungus species typically produce spores only once a year and are unable to produce enough spores to release spores again before the end of a longer growing season.

Butterfly species that respond quickly to changes in temperature hatch caterpillars and undergo metamorphosis into butterflies earlier, flying away before bird species that feed exclusively on them have hatched their young.

Salmon species that rely on changes in temperature to signal the start of annual migrations leave their freshwater birthplaces earlier, before they have fed sufficiently on freshwater insects to reduce their number.

Reptile species that hibernate during winter months must emerge from hibernation sooner and spend a longer time searching for food before mating in mid-summer.Situation: Some scientists argue that climate change will result in a longer growing season and cause the extinction of some animal species.

Reasoning:Which supports the conclusion that a longer growing season will result in some animal species becoming extinct? The first sentence states the general context for the passage: concern over climate change. More specifically, scientists are concerned that changes in the timing of the spring, summer, and autumn will have dire consequences for animal species who do not adapt quickly enough. The correct answer choice will provide an example of a species negatively affected by a change in the timing of seasons.

A. Squirrels and chipmunks will not be adversely affected by a surplus of food.

B. Fungus species will not necessarily be harmed because their reproductive cycle will still produce spores once a year.

Page 202: Economist

C. This option correctly identifies the danger to bird species whose food supply disappears when caterpillars turn to butterflies and fly away before the bird young have hatched.

D. This situation leaves a greater number of insects, but does not negatively affect either the salmon or the insects.

E. This option does not provide sufficient detail to suggest that reptile species are threatened.

The correct answer is CIt was at the American School for the Deaf that influences from both French Sign Language and various “home sign” systems would interact, intermingle, andwhat would become American Sign Language was born.

what would become American Sign Language was born

give birth to what became American Sign language

what would become American Sign Language was given birth

and American Sign Language, or what would eventually become it, was born

American Sign Language was birthedThe underlined portion as written is awkward and its meaning unclear. The correct answer will clarify the underlined portion in a way that maintains parallelism with the list of verbs begun by the helping word would.

A. This option is awkward and does not continue the list of verbs begun by the helping word would.

B. This option correctly places the verb phrase give birth first, so that the list of verbs is continued; this placement also helps to clarify the meaning of the underlined portion within the context of the sentence.

C. The words given birth create a passive construction that is even more unclear and awkward than the original.

Page 203: Economist

D. The words or what would eventually become it create and awkward, unclear construction and do not satisfy the parallelism required in the sentence.

E. American Sign Language should be placed after the verb, so that the list of verbs begun by the word would is continued in the sentence.

The correct answer is BAlthough they were both employed by the Federal Writers’ Project in the 1930s, unlike Ralph Ellison in New York and Massachusetts, Zora Neale Hurston lived and wrote almost exclusively in the South.

unlike Ralph Ellison in New York and Massachusetts, Zora Neale Hurston lived and wrote almost exclusively in the American South.

unlike Ralph Ellison, Zora Neale Hurston lived and wrote almost exclusively in the South.

Zora Neale Hurston lived and wrote almost exclusively in the South, unlike Ralph Ellison in New York and Massachusetts.

Zora Neale Hurston lived and wrote almost exclusively in the South, whereas Ralph Ellison worked in New York and Massachusetts.

Zora Neale Hurston did not live and write in New York and Massachusetts, like Ralph Ellison, but rather was in the South.This sentence compares and contrasts two participants in the Federal Writers’ Project. According to the sentence, the two were alike in their employment, but different in their places of residence. The original sentence is awkward and confusing, because it is not clear which idea unlike is referring to. The correct answer choice will both maintain and clarify the contrast between the writers.

A. Unlike is misplaced, making its antecedent unclear.

B. This version is shorter than the original, but the antecedent of unlike remains unclear.

C. The subordinate clause that begins with unlike does not contain a verb and is, therefore, both incorrect and unclear.

Page 204: Economist

D. This option correctly substitutes the conjunction whereas for unlike, and it creates a correct subordinate clause by adding the verb worked.

E. This option successfully contrasts the writers, but it is wordy and confusing.

The correct answer is DA recent cultural history of the art and architecture of the European Baroque Period, which spanned roughly the entire seventeenth century, uses well-known works to sketch a fresh and surprising perspective on the impact of these works on the imaginations of their original viewers. The author, an emeritus professor at an Ohio university, does not bother to challenge the customary list of Baroque characteristics–opulence, grandeur, movement, large scale, emotion, bold colors, gold–but students of art history, far from entertaining suspicions of insufficient academic rigor due to this oversight, will ultimately find themselves grateful to be left with this piece of familiar ground.

Although the first chapter is spent detailing the debt owed by Baroque style to the theology of the Roman Catholic Church, discussion of the devotional uses of religious pieces thereafter is strictly limited. The prime goal of the following chapters is to define Baroque art’s function within the power structure of the sixteenth century Church in such a way as to make religious devotion beside the point. More specifically, the author is concerned with the role art played in re-centralizing the Church’s power in the wake of the Protestant Reformation, as well as with the significance of the Council of Trent, during which, as part of a cohesive opposition to groups of believers who had broken away, the practice of using art objects as aids to religious devotion was encouraged and codified.

From the author’s perspective, then, the Church’s advocacy of grand and ornate pieces of religious art served the very practical purpose of reminding the faithful that real power was to be found within the Church. St. Peter’s Basilica in Rome, for example, with its almost unimaginatively lofty ceilings, marble floors, and special chapels, solidified the Church’s power by inspiring the viewer’s awe. Likewise, paintings and sculptures produced by most of the period’s Church-sponsored artists depicted human figures, most often saints or martyrs, with a startling realism intended to emphasize presence and immediacy

The passage above is most likely taken from which of the following?

An art history textbook

A personal weblog

A periodical discussing religion and the arts

Page 205: Economist

The book review section of a magazine

The back cover of the book being discussedThe correct answer to this question depends on making an inference about the tone and context of the passage as a whole. It is important to note, first of all, that the passage is a book review for a cultural history. The focus of the passage is on highlighting the content of the book and giving an overview of its objectives in such a way that potential readers can decide whether or not to examine the book more closely. The correct answer will identify the context in which these goals would be appropriate.

A. Textbooks do not deal with current events, such as newly published books.

B. Though this is possible, it is unlikely that a personal weblog would use such technical language.

C. Religion and the arts are both mentioned in the passage, but the main focus is the new cultural ideas offered in the book.

D. Not only is the passage a book review, but the specific content of the discussion best situates it in a periodical.

E. The passage is both too long and too opinionated to appear on the book itself.

The correct answer is DThe author most likely mentions the ceilings of St. Peter’s Basilica in order to

Provide evidence supporting the idea that art can be used to solidify power.

Offer readers a visual image of the work of art in question.

Demonstrate that works of art often inspire awe.

List the distinctive characteristics of a famous building.

Present an explanation of the building’s visual power.The line reference in the question stem indicates that familiarity with the context of the reference is required to answer the question. The words for example in the sentence indicate the continuation of a preceding idea; specifically, that grand and ornate pieces of religious art reminded churchgoers that real power was found in the Church. Additionally, the end of the sentence re-emphasizes this idea by stating that awe over the lofty ceilings solidified the Church’s power.

Page 206: Economist

A. Both the preceding sentence and the sentence from which the reference is taken focus on the idea of art as a reminder of the power of the Roman Catholic Church.

B. A visual image may be produced, but this is not the function of the reference to the ceilings.

C. Though this is true, it is only the first step toward identifying the author’s purpose in this sentence.

D. Characteristics are listed, but only because they help to prove a certain point.

E. The building may have visual power, but this is not the goal of discussing the ceilings.

The correct answer is A.students of art history, far from entertaining suspicions of insufficient academic rigor due to this oversight, will ultimately find themselves grateful to be left with this piece

of familiar ground.The author would most likely agree with which of the following statements concerning academic rigor?

Academic rigor is often due to oversight.

Students of art history find common ground in academic rigor.

Academic rigor often requires scholars to question received knowledge.

Students of art history, though entertaining, often display insufficient academic rigor.

The study of art history requires academic rigor.Although the question stem contains a specific line reference, the words would most likely agree indicate that the correct answer to this question depends on making an inference. First, it is worth noting that the beginning of the sentence relays a fact about the author’s approach to the book: the author does not...challenge the customary list of Baroque characteristics. The author of the passage suggests in the second part of the sentence that some students of art history would perceive this refusal to question custom as insufficient academic rigor. The correct answer, then, will identify what, to the author of the passage, would constitute sufficientacademic rigor.

A. This is a corruption of the meaning of the sentence.

Page 207: Economist

B. Students of art history are the group perceiving the academic rigor applied by the author of the book; their use of academic rigor is not the focus of this sentence.

C. The sentence suggests that art history students may accuse the author of the passage ofinsufficient academic rigor for failing to challenge the customary list of Baroque characteristics. It can be reasonably concluded, then, that sufficient academic rigor would include a challenge to this received knowledge.

D. Entertaining in the passage is used as a verb, not as an adjective as it is in this answer choice.

E. While the author of the passage might agree with this in a different context, the meaning ofacademic rigor in this passage is more specific.

The correct answer is C.The primary function of the second paragraph is to

Prove that religious devotion is not a valid criterion for judging Baroque art.

Introduce the idea that Baroque art played a significant role in centralizing power.

Define a connection between political power and religious devotion.

Challenge the assumption that art is useful for religious devotion.

Trace the development of the author’s understanding of Baroque art.The first paragraph introduces the book being reviewed and suggests there is something unique and exciting about it. The third paragraph offers details of the specific works of art used by the Roman Catholic Church to centralize power. The second paragraph bridges the ideas in the first and third paragraph by introducing the theoretical framework within which specific pieces of Baroque art are analyzed in the remainder of the book.

A. The passage states that the book’s discussion ofreligious devotion is limited, but does not evaluate the concept’s use as a criterion for judging art.

B. The second paragraph serves to explain the framework the book uses for examining each piece of Baroque art discussed, specifically, that art served to centralize the power of the Roman Catholic Church.

Page 208: Economist

C. These two ideas are contrasted, not connected, in the passage.

D. This assumption is not challenged in the passage.

E. The author’s understanding of Baroque art is not the subject of the passage.

The correct answer is B.Scientists working on a number of separate projects made contributions to the invention of the radio, such as Nikola Tesla, who was one of the first to receive a patent in the United States for what was then called “wireless transmission of data.”

radio, such as Nikola Tesla

radio; including as Nikola Tesla

radio, including Nikola Tesla

radio; not least among them being Nikola Tesla

radio; especially Nikola TeslaThe words scientists, separate projects, and contributions all indicate that there is a group of people under discussion in the sentence. The wordincluding, then, is the most appropriate way to indicate Tesla’s participation in the group. Also, since the second clause of the sentence is subordinate, a comma is necessary before the conjunction.

A. Such indicates an example, not participation in a group.

B. Including is a subordinating conjunction and should be preceded by a comma rather than a semicolon.

C. The word including correctly indicates that Tesla was part of a group of scientists working on radio, and the comma before including correctly joins a subordinate to an independent clause.

D. The words not least among them being are redundant and unnecessary.

E. A semicolon is not correct in this sentence, and the word especially is inappropriate, given the context of Tesla being one among a number of scientists working on radio.

The correct answer is C

Page 209: Economist

If the building’s contractor does not agree to repair the damages caused during renovation, we maybe have no choice, but to bring a lawsuit.

we maybe have no choice, but to

it may be we have no choice but to

we maybe have no choice but to

we may have no choice, but to

we may have no choice but toIn this sentence the adverb maybe is inappropriate for expressing the possibility of bringing a lawsuit; may is correct. Additionally, a comma before the conjunction but is incorrect, because but does not join two independent clauses.

A. Maybe is incorrect, and the comma before but is unnecessary.

B. This option breaks up the word maybe, but is awkward and wordy.

C. The adverb maybe is inappropriate for expressing conditional tense.

D. A comma before the conjunction but is incorrect.

E. This option identifies the correct use of the word may to express the possibility of a lawsuit, and the comma before but has been eliminated.

The correct answer is E.Community spokesperson: After a recent surge of foreclosures, many homes in our community are vacant and falling into disrepair. Property values are already falling, and, if action is not taken quickly, will fall even further when panicked residents begin to leave. Clearly the only way to prevent a snowball effect is to make it easy to purchase these vacant homes by offering potential buyers special mortgages with a low interest rate for the first two years.

Which of the following, if true, represents the most significant potential problem with the plan to attract buyers through special mortgage offers?

Interest rates in the area are already at a 5-year low, and it would be foolish of lenders to lower them even more.

Page 210: Economist

Crime rates in the area have risen by 1% in the last year, making it less likely that potential buyers will be interested in moving to the area.

Most residents have lived in the area for over 20 years and do not want to leave their homes.

Low interests rates will likely attract buyers who will be unable to make payments when the interest rate goes up after the first two years.

There are few mortgage brokers in the area, making it unlikely that there will be enough personnel to assist a rush of potential home buyers.Situation: A community spokesperson proposes offering easy mortgages to home buyers to attract them to a community where foreclosures have left many houses vacant.

Reasoning:Which identifies the biggest flaw in the plan to attract potential buyers? The community spokesperson is concerned that vacant homes will cause property values in the neighborhood to fall, and is looking for a permanent solution. The proposed solution would attempt to reverse the problem by making it easier for potential buyers to purchase vacant homes. If, however, mortgages are given to people who will be unable to make payments when interest rates rise after the first two years, the neighborhood will see even more foreclosures and vacant houses, and the snowball effect will only have been delayed.

A. This may be true, but is not the most significant potential problem.

B. Though crime rates are a concern, this option does not directly address a potential problem with mortgages.

C. Whether or not residents want to leave their homes is irrelevant to the soundness of the plan.

D. If vacant homes in the area are purchased primarily by people who will only be able to afford their mortgage payments for the first two years, then after than time period, another series of foreclosures will leave the homes vacant once again.

E. This option does not directly address the soundness of the plan to offer low-interest mortgages for the first two years.

Page 211: Economist

The correct answer is DCommunity spokesperson: After a recent surge of foreclosures, many homes in our community are vacant and falling into disrepair. Property values are already falling, and, if action is not taken quickly, will fall even further when panicked residents begin to leave. Clearly the only way to prevent a snowball effect is to make it easy to purchase these vacant homes by offering potential buyers special mortgages with a low interest rate for the first two years.

The plan to attract potential buyers by offering low-interest rate mortgages for the first two years rests on which of the following assumptions?

Vacant homes will always fall into disrepair.

Homes in the area remain vacant mostly because it is too difficult for potential buyers to get mortgages.

Lenders have enough confidence in their knowledge of interest rates that they can predict what will be considered a low rate for the next two years.

Some home owners may stay in the community, but their property values will decrease.

Vacant homes will cause residents to panic.Situation: A community spokesperson proposes offering easy mortgages to home buyers to attract them to a community where foreclosures have left many houses vacant. Reasoning:Which is the assumption underlying the spokesperson’s plan? The spokesperson is concerned that vacant homes will cause property values in the neighborhood to fall, and is looking for a permanent solution. The proposed solution aims to make it easier for potential buyers to purchase vacant homes, revealing the spokesperson’s assumption that the homes have not already been bought because people could not afford to take on mortgages at prevailing rates.

A. The spokesperson attributes falling property values to the fact that homes are both vacant and falling into disrepair; a permanent connection between the two is not assumed.

Page 212: Economist

B. The plan to attract potential buyers through bargain mortgages rests on the assumption that vacant homes remain vacant mostly because no one can afford to buy them.

C. The success of the plan does not depend on lenders’ ability to predict what will be considered a low rate.

D. Though it is true that some home owners may choose to stay in the community, the spokesperson’s argument does not rest on this assumption.

E. According to the passage, falling property values, not vacant homes, may cause residents to panic.

The correct answer is BThe adoption of either method of extraction, open-pit mining or solution mining, will require miners to be certified in the use of safety equipment.

open-pit mining or solution mining

open-pit mining and solution mining

open-pit and solution minings

open-pit mining, including solution mining

mining by open-pit or by solutionThis sentence requires the grammatically correct extension of the phrase either method of extraction. The underlined portion must show contrast and agree with the singular word either by using the conjunction or.

A. This option correctly extends the phrase in the first part of the sentence to read either…open-pit mining or solution mining.

B. And is an incorrect conjunction, because it does not agree with either.

C. This option may seem more concise, but and is incorrect because it does not agree with either, and minings should not be plural.

D. In this option, the meaning of the sentence has been changed to incorrectly suggest that solution mining is a kind of open-pit mining.

Page 213: Economist

E. Mining is redundant in this context, because it repeats the words method of extraction. This word order also makes the sentence awkward and confusing.

The correct answer is A